Download as pdf or txt
Download as pdf or txt
You are on page 1of 140

Solutions to

Mathematics
PULLOUT WORKSHEETS
FOR CLASS X
First Term

By

Surender Verma
P.W - 288 pages / 30-12-2014

M.Sc. (Maths), B.Ed


Delhi Public School,
Dwarka, New Delhi

New Saraswati House (India) Pvt. Ltd.


New Delhi-110002 (INDIA)

Published by

New Saraswati House (India) Pvt. Ltd.


Second Floor, MGM Tower, 19 Ansari Road, Daryaganj, New Delhi-110002 (India)
Phone : +91-11-43556600
Fax
: +91-11-43556688
E-mail : delhi@saraswatihouse.com
Website : www.saraswatihouse.com
CIN
: U22110DL2013PTC262320
Import-Export Licence No. 0513086293

Branches

Ahmedabad  (079) 22160722 Bengaluru  (080) 26619880, 26676396 Chennai  (044) 28416531
Dehradun  09837452852 Guwahati  (0361) 2457198 Hyderabad  (040) 42615566
Jaipur  (0141) 4006022 Jalandhar  (0181) 4642600, 4643600 Kochi  (0484) 4033369
Kolkata  (033) 40042314 Lucknow  (0522) 4062517 Mumbai  (022) 28737050, 28737090
Patna  (0612) 2570403 Ranchi  08294693413

Author
All rights reserved. No part of this publication may be reproduced, stored in a retrieval system, or
transmitted, in any form or by any means without the prior written permission of the publishers.
Printed at Vikas Publishing House Pvt. Ltd., Sahibabad (Uttar Pradesh)

CONTENTS
1. Real Numbers
Worksheets (1 to 6) ..................................................................................................... 6
Assessment Sheets (1 and 2) ................................................................................... 11
Chapter Test ............................................................................................................. 13
2. Polynomials
Worksheets (10 to 15) ............................................................................................... 15
Assessment Sheets (3 and 4) ................................................................................... 23
Chapter Test ............................................................................................................. 26
3. Pair of Linear Equations in Two Variables
Worksheets (18 to 29) ............................................................................................... 29
Assessment Sheets (5 and 6) ................................................................................... 45
Chapter Test ............................................................................................................. 48
4. Triangles
Worksheets (33 to 45) ............................................................................................... 51
Assessment Sheets (7 and 8) ................................................................................... 68
Chapter Test ............................................................................................................. 71
5. Introduction to Trigonometry
Worksheets (50 to 58) ............................................................................................... 73
Assessment Sheets (9 and 10) ................................................................................. 84
Chapter Test ............................................................................................................. 86

6. Statistics
Worksheets (62 to 65) ............................................................................................... 88
Assessment Sheet 11 ................................................................................................ 93
Chapter Test ............................................................................................................. 97
PRACTICE PAPERS (1 to 5) ..................................................................................... 99

Solutions to

PULLOUT WORKSHEETS

AND
PRACTICEPAPERS

[SummativeAssessments]
[FIRST TERM]

Chapter

REAL NUMBERS
WORKSHEET 1

1. (C) We know that the factors of a prime are


1 and the prime itself only.
Therefore, the common factor of p and q
will be 1 only. Hence, HCF (p, q) = 1.
2. (A) As prime factors of 1005 are:
1005 = 5 3 67.
7 is not a prime factor of 1005.
3.

125
53
1
=
=
= 0.0625
4. 3
16
2 5
16 53
Clearly, the decimal form of
nates after four places.

125
termi24 . 53

4. Terminating
Hint:

24
192
=
= 0.192 .
125 1000

5. LCM =
=

First number Second number


HCF

96 404
= 24 404 = 9696.
4

6. (i) 660 (ii) 330


Hint: Going in opposite direction to the
factor tree, we obtain
2 165 = 330 (ii) and 2 330 = 660 (i).

10. Let a be any odd positive integer and b = 4.


By Euclids lemma there exist integers q
and r such that
a = 4q + r, 0 r < 4
.. .
a = 4q or 4q + 1 or 4q + 2 or 4q + 3.
Therefore, for a to be odd, we have to take
a = 4q +1 or 4q +3.
11. The maximum capacity (in kg) of a bag will
be the HCF of 490, 588 and 882. Let us find
out the required HCF by prime factorisation
method.
490 = 2 5 72
588 = 22 3 72
882 = 2 32 72

HCF = 2 72 = 98
Thus, the maximum capacity of a bag is
98 kg.

WORKSHEET 2
1. (A) HCF (p, q) = 1 p and q are coprime.
If p and q are coprime with q 0 and

p
is
q

7. HCF = 3; LCM = 420


Hint: 12 = 22 3; 15 = 3 5; 21 = 3 7.

a rational number, then q has only 2 and 5


as prime factors.

8. (i) Terminating

i.e., q = 2m 5n where, m and n are nonnegative integers.

Hint:

543
543
= 1 3.
250 2 5

(ii) Non-terminating repeating


Hint:

9
1
1
=
=
.
108 12 22 31

a
9. Hint: Let 5 2 3 = ; b 0
b
5b a

3 =
2b

As RHS of this equation is rational, but


LHS is irrational so a contradiction.

2. (B) Going to opposite direction to the


factor tree, we obtain
3 7 = 21 (ii) and 2 21 = 42 (i).
3. 2 = 1.414... and

3 = 1.732...

3
Therefore, we can take 1.5 =
2
3
as 2 < < 3 .
2
M A T H E M A T

I C S X

4. Required number =

231449
161
=

1449
= 207.
7

5. Hint: As 12576 > 4052


...
12576 = 4052 3 + 420
Further
4052 = 420 9 + 272
Further
420 = 272 1 + 148
Further
272 = 148 1 + 124
Further
148 = 124 1 + 24
Further
124 = 24 5 + 4
Further
24 = 4 6 + 0.
In the last equation, remainder is zero.
Hence, the required HCF = 4.
6. First given number is composite as
5 3 11 + 11 = 11 (15 + 1) = 11 16
= 11 2 8
But second given number is prime as
5 7 + 7 3 + 3 = 35 + 21 + 3 = 59.
7. No. Prime factors of 6n will be of type 2n 3n.
As it doesn't have 5 as a prime factor, so 6n
can't end with the digit 5.
8. Hint: Let a be any positive integer
.. .
a = 3q or 3q + 1 or 3q + 2
.
..
a2 = 9q2 = 3m; m = 3q2
or
a2 = (3q +1)2 = 3m + 1, m = q (3q + 2)
or
a2 = (3q + 2)2 = 3m + 1, m = 3q2 + 4q + 1.
9. We represent 6, 72 and 120 in their prime
factors.
6 =2 3
72 = 23 32
120 = 23 3 5
Now, HCF = 2 3 = 6
And LCM = 23 32 5 = 360.
10. Hint: Let

2  5 = x, a rational number

2 =x+ 5
Squaring both sides, we get
2 = x2 + 5 + 2x 5
x2 3

5 =
2x
RHS of this last equation is rational, but
LHS is irrational which is a contradiction.
R E A L

N U M B E R S

11. Length = 6 m 30 cm = 630 cm


Breadth = 5 m 85 cm = 585 cm
Height = 3 m 60 cm = 360 cm
The required length (in cm) of the tape will
be the Highest Common Factor (HCF) of
the numbers 630, 585 and 360.
Let us find out the HCF.
630 = 2 32 5 7
585 = 32 5 13
360 = 23 32 5
HCF = 32 5 = 45
Hence, the length of the tape will be 45 cm.

WORKSHEET 3
1. (C)

43
43 5
215
=
= 4 = 0.0215
3
4
2 5
(2 5)
10
4

Hence, the number terminates after four


places of decimal.
2

2. (A) 2  3
2 3
 2

= 2 3 = 1.
1 is a rational number.
3. 128 = 27; 240 = 24 3 5.
Now, HCF (128, 240) = 24 = 16.
4. 232
HCF LCM
Hint: First number =
.
Second number
5. No.
Hint: Prime factors of 15n does not contain
2p 5q in factor, p, q being positive integers.
6. Rational number = 0.27
Irrational number = 0.26010010001... .
7. (i)
(ii)

145
29
8
232
=
=
= 0.232 .
625 125 8 1000
7
125
875

=
= 0.0875 .
80 125 10000

8. Let us assume, to the contrary that 2 is


rational. We can take integers a and b 0
such that
a
3 = , where a and b are coprime.
b

3b2 = a2

a2 is divisible by 3


a is divisible by 3
...(i)
We can write a = 3c for some integer c

a2 = 9c2

3b2 = 9c2
(... a2 = 3b2)
2
2

b = 3c
2

b is divisible by 3

b is divisible by 3
...(ii)
From (i) and (ii), we observe that a and b
have atleast 3 as a common factor. But this
contradicts the fact that a and b are coprime. This means that our assumption is
not correct.
Hence,

3 is an irrational number.
9. As:
1032 = 408 2 + 216
...(i)
408 = 216 1 + 192
...(ii)
216 = 192 1 + 24
...(iii)
192 = 24 8 + 0
...(iv)

HCF = 24
From (iii)

24 = 216 192
= 216 [408 216] [Use (ii)]
= 2 216 408
= 2[1032 2 408] 408
[Use (i)]

24 = 1032 2 5 408

m = 2.
10. Hint: Let x be any positive integer.
Then it is of the form 3q or 3q + 1 or 3q + 2.
If
x = 3q, then
x3 = (3q)3 = 9m; m = 3q3
If
x = 3q + 1, then
x3 = (3q + 1)3
= 9m + 1; m = q(3q2 + 3q + 1).
If
x = 3q + 2, then
x3 = (3q + 2)3
= 9m + 8; m = q (3q2 + 6q + 4).
11. The maximum number of columns must be
the highest common factor (HCF) of 616
and 32. Let us find out the HCF by the
method of Euclid's division lemma.
Since 616 > 32, we apply division lemma
to 616 and 32, to get
616 = 32 19 + 8

Since the remainder 8 0, we apply the


division lemma to 32 and 8, to get
32 = 8 4 + 0
The remainder has now become zero, so
our procedure stops. Since the divisor at
this stage is 8, the HCF of 616 and 32 is 8
Hence, the maximum number of columns
is 8.

WORKSHEET 4
1. (B) Non-terminating repeating.
Hint: Denominator is not in the exact form
of 2m 5n, where m, n are non-negative
integers.
2. (C) 0 r < b.
2

3. 6  5
6 5
 6
 5

= 6 5 = 1 = Rational number.
4. Terminating decimal form as denominator

107
is of the form 2n 5m.
4
Here n = 2, m = 0
4 of

5. (i) 1001 (ii) 91


Hint: 7 13 = (ii) and (ii) 11 = (i).
6. Let us represent each of the numbers 30,
72 and 432 as a product of primes.
30 = 2 3 5
72 = 23 32
432 = 24 33
Now, HCF = 2 3 = 6
and LCM = 24 33 5 = 2160.
7. Here, 396 > 82.

396 = 82 4 + 68
Further 82 = 68 1 + 14
Further 68 = 14 4 + 12
Further 14 = 12 1 + 2
Further 12 = 2 6 + 0
In the last equation, the remainder is zero
and the divisor is 2.
Hence, the required HCF = 2.
a
8. Hint: Let 3 + 2 5 = ; b 0
b
a 3b

= 5 = Rational
2b
M A T H E M A T

I C S X

Which is a contradiction as
tional number.

5 is an irra-

n = 1, 3, 17 5, 33 , ...
n = An odd integer is the right
Answer.

9. (i) The given fraction can be written as


43
43 5
=
= 0.0215
4 3
2 5
10 4

Hence, the given number terminates after


four places of decimal.
(ii) The given fraction can be written as

359
2 4 359
5744
=
5
5
5 =
25
2 5
100000
= 0.05744
Hence, the given number terminates after
five places of decimal.
10. The required number of students will be
the highest common factor (HCF) of 312,
260 and 156. Let us find out the HCF by
the method of prime factorisation.
312 = 23 3 13
260 = 22 5 13

HCF = 22 13 = 52
Number of buses required

Total number of students


Number of students in one bus

312 + 260 + 156


= 14
52
Thus, the maximum number of students in
a bus and number of buses required are
52 and 14 respectively.

11. Hint: Let x = any positive integer


x = 5m, 5m + 1, 5m + 2, 5m + 3 or 5m + 4
Now take square of all these form.

WORKSHEET 5
1. (C) Let the quotient is m when
divided by 8.

4. Prime factors of numbers 1 to 10 are:


1 = 1; 2 = 1 2; 3 = 1 3; 4 = 1 22
5 = 1 5; 6 = 1 2 3; 7 = 1 7;
8 = 1 23; 9 = 1 32; 10 = 1 2 5
Now,
LCM = 1 23 32 5 7
= 8 9 5 7 = 2520 is required
number.
5. 2.
Hint:

156 = 22 3 13

2. Hint: HCF (65, 117) = 13


Now,
65m 117 = 13.

m = 2 will satisfy this equation.


3. Hint: LCM of 18, 24, 30, 42 = 2520
Required number = 2520 + 1 = 2521.

n2

1 is

N U M B E R S

= 2x 15

4 15 = 2x 15

5 18  6 10 =

R E A L

x = 2, which is a rational number.


6. Hint: Any odd positive integer will be
type of 4q + 1 or 4q + 3

(4q + 1)2 = 16q2 + 8q + 1


= 8 (2q2 + q) + 1
= 8n + 1
2
Also, (4q + 3) = 16q2 + 24q + 9
= 8 (2q2 + 3q + 1) + 1
= 8n + 1.
7. 35 cm
Hint: Find HCF.
a
8. Hint: Let 5  3 2 =
b
where a, b are integers and b 0
Squaring on both sides,

n2 1 = 8 m
n2 1 = 0, 8, 16, 24, 32, ... .
n2 = 1, 9, 17, 25, 33, ...

5

23 

a2
= 6 10
b2

23b 2  a 2
6b 2

a2
b2

= 10 ... a contradiction.

9. (i) Terminating

(ii) Terminating.

10. The required number of burfis will be the


highest common factor of 420 and 130.
Let us find out the HCF using Euclid's
division lemma.
It is clear that 420 > 130. We apply Division
lemma to 420 and 130, to get
420 = 130 3 + 30
Since the remainder 30 0, so we apply
Division lemma to 130 and 30, to get
130 = 30 4 + 10
Again the remainder 10 0, so we apply
Division lemma to 30 and 10, to get
30 = 10 3 + 0
Now, the remainder is zero. So the HCF of
420 and 130 is the divisor at the last stage
that is 10.
Hence, the required number of burfis is 10.
11. Let n = 3q, 3q + 1 or 3q + 2.
Case I. If n = 3q, then
n = 3q divisible by 3
n + 2 = 3q + 2 Not divisible by 3
n + 4 = 3q + 4 = 3(q + 1) + 1
Not divisible by 3.
Case II. If n = 3q + 1 then only
n + 2 = 3q + 1 + 2 = 3q + 3
= 3(q + 1) is divisible by 3.
Case III. If n = 3q + 2 then only
n + 4 = 3q + 6 = 3(q + 2)
is divisible by 3.

WORKSHEET 6
1. (C) 3825 = 52 32 17
So, 11 is not a prime factor of 3825.
2. (C) As p and p + 1 are two consecutive
natural numbers, HCF = 1 and
LCM = p (p + 1).

51
17
or
1500
500
Denominator = 500 = 22 53
Clearly, the denominator is exactly in the
form 2m 5n, where m and n are nonnegative integers; so the given number
has a terminating decimal expansion.

3. Hint: The given number is

10

4. 1800
Hint: ... 8 = 23; 9 = 32; 25 = 52
HCF (8, 9, 25) = 1
LCM (8, 9, 25) = 1800.
5. 19
Hint:

6. Irrational
Hint:

HCF (210, 55) = 5


210 5 + 55y = 5
55y = 5 1050
1045
= 19.
y=
55
2 3
2 3

x
3

x
3
7 3 12 = x = Irrational.
74 3 =

7. Rational Number = 0.55


Irrational number = 0.5477477747... .
8. 15
Hint: HCF (1380, 1455, 1620) = 15.
9. (i) 0.052.
(ii) 5.8352.
10. We know that any positive integer is either
of the form 3q, 3q + 1 or 3q + 2 for some
integer q.
Now, three cases arise.
Case I. When p = 3q,
p + 2 = 3q + 2 and p + 4 = 3q + 4
Here,
p = 3q is exactly divisible by 3
p + 2 = 3q + 2 leaves 2 as remainder when it is divided by 3
p + 4 = 3q + 4 or 3 (q + 1) + 1 leaves
1 as remainder when it is
divided by 3.
Case II. When p = 3q + 1,
p + 2 = 3q + 3 and p + 4 = 3q + 5
Here,
p = 3q + 1 leaves 1 as remainder
when it is divided by 3
p + 2 = 3q + 3 or 3 (q + 1) is exactly
divisible by 3
p + 4 = 3q + 5 or 3(q + 1) + 2 leaves 2
as remainder when it is
divided by 3.
M A T H E M A T

I C S X

Case III. When p = 3q + 2, p + 2 = 3q + 4


and p + 4 = 3q + 6
Here, p = 3q + 2 leaves 2 as remainder when
it is divided by 3.
p + 2 = 3q + 4 or 3(q + 1) + 1 leaves 1 as
remainder when it is divided by 3
p + 4 = 3q + 6 or 3(q + 2) is exactly divisible
by 3.
Hence, in all the cases, one and only one
number out of p, p + 2 and p + 4 is divisible
by 3, where p is any positive integer.
OR
Any positive odd integer is type of 2q + 1
where q is a whole number.
(2q + 1)2 = 4q2 + 4q + 1 = 4q (q + 1) + 1
...(i)
Now, q(q + 1) is either 0 or even
So it is 2m, where m is some number.
From (i) (2q + 1)2 = 8m + 1.
11. Since, height of each stack is the same,
therefore, the number of books in each stack
is equal to the HCF of 96, 240 and 336.
Let us find their HCF
96 = 24 2 3
240 = 24 3 5
336 = 24 3 7
So, HCF = 24 3 = 48.
Now, number of stacks of English books
96
=2
=
48
Number of stacks of Hindi books

HCF LCM = Product of the two numbers


40 252 p = 2520 6600
2520 6600

p=
= 1650.
40 252
4. No; because HCF must divide LCM and here
HCF = 18 which doesn't divide LCM which
is 380.
3.

5. True, If the number 3n ends with the digit 0,


then its prime factorisation contains the
primes 2 and 5. But by the Fundamental
Theorem of Arithmetic, there is no prime other
than 3 in the factorisation of 3n.
6. The required number would be the HCF of
967 7 = 960 and 2060 12 = 2048.
Let us find the HCF of 960 and 2048 by
using Euclids algorithm.
Since 2048 > 960

2048 = 960 2 + 128


960 = 128 7 + 64
128 = 64 2 + 0
Since the remainder becomes zero and the
divisor at this stage is 64, the HCF of 960
and 2048 is 64.
Hence, the required number is 64.
7.

240
=5
48
Number of stacks of Mathematics books

336
= 7.
48

ASSESSMENT SHEET 1
1. (D) The denominator of each fraction in the
options (A), (B) and (C) can be expressed in
the form 2n 5m, where m, n being nonnegative integer.
2. (A) Let x be any positive integer then it is
of the form 3q or 3q + 1 or 3q + 2. So, x2 can
be written in the form 3m or 3m + 1.
R E A L

N U M B E R S

456 = 23 3 19
360 = 23 32 5
= 23 3 = 24
456 360
Hence
LCM =
= 6840.
24
8. (i) Time taken by Ram to complete one cycle
= 180 seconds.
Time taken by Shyam to complete one cycle
= 150 seconds.
Consider LCM of 180 and 150.
Clearly,
and
HCF

11

180 = 22 5 32

2. (C) LCM (p, q) = x3 y2 z3.

150 = 2 52 3

3. HCF LCM = Product of the two numbers.

LCM of 180 and 150 = 22 52 32


= 4 25 9
= 900 seconds
=

900
= 15 minutes
60

They both will again meet after 15 minutes.


(ii) Since they started at 6 a.m. and they will be
meeting again after 15 minutes.
The time will be 6:15 a.m.
(iii) L.C.M. of real numbers.
(iv) Since Ram and Shyam go for morning walk
daily. So, it depicts their discipline and
health consciousness.
9. Let a be any odd positive integer. Then, it is
of the form 6p + 1, 6p + 3 or 6p + 5.
Here, three cases arise.
Case I: When a = 6p + 1,

a2 = 36p2 + 12p + 1
= 6p(6p + 2) + 1 = 6q + 1,
where q = p(6p + 2).
Case II: When a = 6p + 3,

a2 = 36p2 + 36p + 9
= 36p2 + 36p + 6 + 3
= 6(6p2 + 6p + 1) + 3
= 6q + 3,
where q = 6p2 + 6p + 1.
Case III: When a = 6p + 5,

a2 = 36p2 + 60p + 25
= 36p2 + 60p + 24 + 1
= 6(6p2 + 10p + 4) + 1
= 6q + 1,
where q = 6p2 + 10p + 4.
Hence, a is of the form 6q + 1 or 6q + 3.

ASSESSMENT SHEET 2
14587
= 11.6696.
1250
Clearly, the decimal expansion terminates
after four decimal places.

1. (D)

12

9 LCM = 306 657

306 657
= 22338.
9
4. As given number can be written as 2525
which is product of prime numbers: 5 5
101. Hence it is a composite number.
5. The maximum number out of 3, 5, 15, 25,
75 is 75. Therefore, the HCF of 525 and 3000
is 75.

LCM =

6. The denominator of

257
514
or
is 10000.
5000
10000

10000 = 104 = 24 54
Further,

514
257
=
= 0.0514.
5000
10 4

7. Let
x = 2p + 1 and y = 2q + 1
2
x + y2 = (2p + 1)2 + (2q + 1)2
= 4p2 + 4p + 1 + 4q2 + 4q + 1
= 4(p2 + p + q2 + q) + 2
=S+T
where
S = 4(p2 + p + q2 + q) and T = 2
S is divisible by 4 and so an even integer.
T is not divisible by 4 but an even integer.
Therefore, S + T is even, as sum of any two
even number is even, and not divisible by 4.
8. Let us assume the contrary that 5 is a
rational number.
We can take coprime a and b (say) such that
a
5 = ;b0
b

b 5=a
Square both the sides to get
5b2 = a2
2
a is divisible by 5
a is divisible by 5 because if square of a
number is divisible by a prime, then the
number is divisible by the prime.
Let us take some integer c such that
a = 5c
Squaring both sides, we get
a2 = 25c2
M A T H E M A T

I C S X

Substitute

a2 = 25c2 in 5b2 = a2 to get


5b2

25c2

= 5c2
b2 is divisible by 5
b is divisible by 5
Therefore, both a and b are divisible by 5.
This contradicts the fact that a and b are
coprime that is a and b have no common
factor.
Our assumption is false.
So, we conclude that 5 is an irrational
number.

4. Yes.
2 3 5 13 17 + 13

b2

9. (i) First we will find HCF of 732 and 942 by


using Euclids lemma:

942 = 732 1 + 210


732 = 210 3 + 102
210 = 102 2 + 6
102 = 6 17 + 0
The last divisor = 6

HCF = 6
The least number of students in which
732 apples can be distributed such that
732
each student will get 6 apples =
= 122.
6
Similarly, the least number of students in
which 942 oranges can be distributed such
that each student will get 6 oranges
942
=
= 157.
6

Total least number of students required =


122 + 157 = 279.
(ii) HCF of two real numbers.
(iii) Harmony and love.

CHAPTER TEST
1. (D) Since 32844 = 2 2 3 7 17 23
So, 11 is not prime factor of 32844.
2. (C)
As 8q is even and 6 is even, 8q + 6 is even.
3. ... LCM =
R E A L

306 1314
= 22338.
18

N U M B E R S

= 13 (2 3 5 17 + 1)
= 13 511
= a composite number.
5.

2 9
= 2 2 18 9
= 11 2 18
= irrational.

6. No.
Hint: Prime factors of 9n will be type of
32n, i.e., 3 3 ... 3
 
Even no.
of times.
7. ...

0.56125 =
=

...

8.

56125
449
=
100000
800
449
449
= 5
32 s 25
2 s 52

2n 5m = 25 52
n = 5, m = 2.
120 = 23 3 5
105 = 3 5 7
150 = 2 3 52

HCF = 3 5 = 15

And

LCM = 23 3 52 7
= 8 3 25 7
= 4200.

9. Hint:
Let

2 3 3 = x, where x is rational.
2

2 3 3
= x2

2 + 27 6 6 = x2

29 x2 = 6 6
29 x 2
=
6

6.

Since 6 is not a perfect square. So


always irrational.

6 is

It's a contradiction.

13

10. We know that any positive integer is of the


form 3q or 3q + 1 or 3q + 2.
n = 3q

Case I:

n3 = (3q)3 = 9 3q3 = 9m
n3

+ 1 = 9m + 1, where m = 3q3.
n = 3q + 1

Case II:

n3 = (3q + 1)3
= 27q3 + 1 + 27q2 + 9q

= 9q (3q2 + 3q + 1) + 1
= 9m + 1
n3 + 1 = 9m + 2, where
m = q(3q2 + 3q + 1).

Case III:

where

n = 3q + 2
n3 = (3q + 2)3

= 27q3 + 8 + 54q2 + 36q


n3 + 1 = 27q3 + 54q2 + 36q + 9
= 9(3q3 + 6q2 + 4q + 1)
= 9m,
m = 3q3 + 6q2 + 4q + 1.

Hence, n3 + 1 can be expressed in the form


9m, 9m + 1 or 9m + 2, for some integer m.
11. (i) We will find HCF of 96 and 112 by using
Euclids lemma:

112 = 96 1 + 16
and
96 = 16 6 + 0
the last divisor = 16

HCF = 16
The minimum number of boxes required
96
for apples =
=6
16
and the minimum number of boxes
112
=7
required for oranges =
16
Total minimum number of boxes required
= 7 + 6 = 13.
(ii) Concept used is HCF of two real
numbers using Euclids lemma.
(iii) By distributing fruits in orphanage his
kindness and concern towards the
needful has been reflected.

qq

14

M A T H E M A T

I C S X

Chapter

POLYNOMIALS
WORKSHEET 10

1. (C) Since the given graph of y = p(x) cuts


x-axis at three points, so the number of
zeroes of p(x) are 3.
b ( 5) 5


2. Sum of zeroes =
a
3
3
c 1
Product of zeroes = 
a 3
3. 1
Hint:

   B C
.
B C BC

8. Solving + = 3 and = 1,
we get = 1, = 2
Polynomial is x2 ( + ) x +
p(x) = x2 3x + 2.
9. According to the division algorithm,
p(x) = g(x) q(x) + r(x)
3
2
x 3x + x + 2 = g(x) (x 2) + ( 2x + 4)
(As given in question)

g(x) =

x 3 3x 2 + 3 x 2
x2

To find g(x), we proceed as given below.

4. Let one zero be , then the other one will be


1
.

5.

k
15
k = 15.

Sum of zeroes (S) =


=

2
3

4
3

38
5
=
4 3
4 3

2
3
1
=

2
4
3
Now, required polynomial will be
1
5
x2 Sx + P, i.e., x2 +
x
2
4 3
Product of zeroes (P) =

or 4 3 x2 + 5x 2 3 .
6. Let f (x) = 2x2 + 2ax + 5x + 10
If x + a is a factor of f (x), then f ( a) = 0
Therefore, 2a2 2a2 5a + 10 = 0

a = 2.
7. x3 4x2 + x + 6
Hint: If the zeroes are a, b and of a cubical
polynomial, then the polynomial will be
(x ) (x ) (x )
= (x 3) (x 2) (x + 1) = x3 4x2 + x + 6.
P O L Y N O M

A L S

Thus, g(x) = x2 x + 1.
1 3
10.  ;
3 2
Hint: 6x2 7x 3
= 6x2 9x + 2x 3
= 3x(2x 3) + 1(2x 3)
= (2x 3) (3x + 1)
2x 3 = 0 gives
3

x=
2

1
3
1 3 7 b
+=
+ = =
a
2
6
3
1 3
1
c
.=
.
=
= .
a
3
2
2
3x + 1 = 0 gives x =

15

11. Let p(x) = x4 + x3 34x2 4x + 120


Given zeroes of p(x) are 2 and 2
(x 2) (x + 2) = x4 4 is a factor of p(x).
We divide p(x) by x2 4,
x2 + x 30
x2 4 x4 + x3 34x2 4x + 120
x4
4x2

+
x3 30x2 4x +120
x3
4x
+

30x2 + 120
30x2 + 120

+
0

p(x) = (x2 4) (x2 + x 30)


Other zeroes of p(x) are given by
x2 + x 30 = 0
2
x + 6x 5x 30 = 0
x(x + 6) 5(x + 6) = 0

(x 5) (x + 6) = 0
x = 5, 6
Hence, all the zeroes are 2, 2, 5 and 6.

WORKSHEET 11

5. p = 2
Hint: (2)3 3(2)2 + 3(2) p = 0

8 12 + 6 p = 0
2 p =0
p = 2.

6. Let a and b be the two zeroes of


f(x) = ax2 + 2x + 3a

3a
2
and ab =
=3
a
a
According to the question,
Then, a + b =

2
=3
a

a=

2
.
3

7. Let the third zero be a, then

coefficient of x 2
coefficient of x 3

sum of the zeroes =

2+3+a=

6
1

a= 1
Hence, the third zero is 1.

8. Let us divide 6x4 + 8x3 + 17x2 + 21x + 7 by


3x2 + 4x + 1.

1. (D) Let us take option (D)


p(x) = (x2 2) (x2 + 3x) = 3x 2
This is a linear polynomial.
2. Q p(x) = 2x2 2x + 1
Sum of zeroes = 1
1
Product of zeroes = .
2
3. Let a = 5 and b = 5, then the quadratic
polynomial will be x2 (a + b)x + ab
or x2 25.
4. p(x) = 4x2 4x + 1
= 4x2 2x 2x + 1
= 2x (2x 1) 1(2x 1)
= (2x 1) (2x 1)
For zeroes, 2x 1 = 0 and 2x 1 = 0
1 1

x= , .
2 2

16

Clearly, the remainder is x + 2.


Now, ax + b = x + 2
Comparing the coefficients of like powers
of x both the sides, we obtain
a = 1, b = 2.
9. We know:
Dividend = (Divisor Quotient) + Remainder
4x3 8x2 + 8x + 1 = g(x) (2x 1) + x + 3

g(x) (2x 1) = 4x3 8x2 + 7x 2

g(x) =

4x 3 8x 2 7 x 2
2x 1

M A T H E M A T

I C S X

Now,

Hence, the number of sweets which was


distributed amoung the slum children was
2x 3.

2x2 3x + 2
2x 1 4x3 8x2 + 7x 2
4x3 2x2
+
6 x2 + 7 x 2
6x2 + 3x

+
4x 2
4x 2
+
0
Hence, g(x) =
10.

2x2

(ii) Helping one another, fair division.

WORKSHEET 12
1. (C)

3x + 2.

3 and 1

Hint: x2 3 x x + 3 = (x
For zeroes, x

3 = 0 and x 1 = 0

Now,

3 ) (x 1)

x=

3, 1

sum of zeroes =

3+ 1

=
And product of zeroes =
=

Coefficient of x
Coefficient of x 2

3
Constant term
.
Coefficient of x 2

11. (i) To find the number of sweets which was


distributed among the slum children, we
divide the total number of sweets by number
of children Mr. Vinod has. Remainder thus
obtained is the required number of sweets.
x2 + 6x + 8
x2 4x + 3 x4 + 2x3 13x2 12x + 21
x4 4x3 + 3x2
+

3
6x 16x2 12x + 21
6x3 24x2 + 18x

2
8x 30x + 21
8x2 32x + 24

+
2x 3

P O L Y N O M

A L S

Sum of zeroes =

( 5)
= 15
1

3

3
9
2
Product of zeroes = = .
2
1
3
c
2. (A) Let the zeroes be , , . Then =
1
If = 1, then = c
...(i)
3
2
Further, ( 1) + a ( 1) + b ( 1) + c = 0
1+ab+c=0
c=ba+1
...(ii)
From equations (i) and (ii), we have
= b a + 1.
3.
Sum of zeroes = 6
3k

6=
1
6

k = = 2.
3

4. Let one zero be , then the other one will


1
be .

4a
a2 4
a2 4a + 4 = 0

(a 2)2 = 0

a = 2.
5. Given polynomial is:
f (x) = x2 px 2p c

+= p
and
. = 2p c
( + 2) ( + 2) = + 2 ( + ) + 4
= 2p c + 2p + 4
= (4 c).
So,

B=

17

6. = 6
Hint: ( + )2 = ( )2 + 4.
7. x = 1 or 3; f(x) = x2 2x 3
Hint:
x = 1 or 3,
Sum of zeroes = 2
Product of zeroes = 3

p(x) = x2 ( + )x +
= x2 2x 3.

47
4
Hint: f (x) = {x2 (sum of zeroes) x + (product
of zeroes)}

8. x2 x

9. The number which to be subtracted is the


remainder when 4x4 + 2x3 8x2 + 3x 7 is
divided by 2x2 + x 2. To find the remainder,
we proceed as following.
2x2 2
2x2 + x 2 4x4 + 2x3 8x2 + 3x 7

4
4x2 2x +
+
+
5x 11

Hence, 5x 11 must be subtracted from


4x4 + 2x3 8x2 + 3x 7 so that it becomes
exactly divisible by 2x2 + x 2.
10. g(x) = x2 + 2x + 1
Hint:

p(x) = g(x) q(x) + r(x)

g(x) =

p ( x ) r( x )
q( x )

p(x) = 3x3 + x2 + 2x + 5
q(x) = 3x 5

and

r(x) = 9x + 10.

5
5
and x =
are zeroes of
3
3
p(x) = 3x4 + 6x3 2x2 10x 5, so p(x) is

11. Since x =

divisible by x

18

Hence, all the zeroes of p(x) are

5
, 1 and 1.
3

4x4 + 2x3 4x2

+
4x2 + 3x 7

where,

Here, other two zeroes of p(x) are the two


zeroes of quotient 3x2 + 6x + 3
Put
3x2 + 6x + 3 = 0

3(x + 1)2 = 0
x = 1 and x = 1

5
5
5
x +
, i.e., x2 3 .

3
3

5,
3

WORKSHEET13
1. (B) Sum of zeroes = 99 = ve
Product of zeroes = 127 = + ve
If the sum of both zeroes is negative, then
the zeroes would be either both negative or
one negative and other one positive. If the
product of both the zeroes is positive, then
the zeroes would be either both positive or
both negative.
Consequently, we obtain that both the
zeroes are negative.

3
11
,
2
2
Hint: Given polynomial can be written as:

2.

p(x) = 2x2 + 3x 11

b
a
c
Product of zeroes = .
a
Sum of zeroes =

3. We know that the degree of the remainder


is less than the degree of divisor or does't
exist.
M A T H E M A T

I C S X

Here, degree of the divisor is 3, therefore,


the possible degree of the remainder
according to the options can be any out of
0, 1 and 2.
4. k = 0.
Hint: Substitute x = 2 in x2 +
5. Since a, b are the zeroes of
a + b = p; ab = q

2x + k = 0.

+ px + q, then

p
1 1 +
+ =
=

Now,

and

x2

1 1
1
1
=
=
q

So the polynomial having zeroes


will be



and
B
C

 
 
q(x) = x2 x +
B C
B C

= x2 +

p
1
x+
q
q

a2 a 12
= (a 4) (a + 3)
For other zeroes, put a 4 = 0 and a + 3 = 0
a = 3, 4
Thus, the other two zeroes are 3 and 4.
10. g(x) = x + 1.
Hint: Applying division algorithm, we get
x4 + 1 = g(x) (x3 x2 + x 1) + 2
x4 1
g(x) = 3
x x2 + x 1
=

or q(x) = qx2 + px + 1.
x2

6. g(x) = + 2x + 7.
Hint: Divide x3 + 3x 14 by x 2.
7. One example is:
p(x) = 3x2 3x + 12.

q(x) = 3

1
,
7

= x + 1.
11. b 2ac
c2

B 2 C2
2
B C
 2BC
=
B2C2

r(x) = 0.
8.

x 1
x 2 + 1

Hint:

g(x) = x2 x + 4

x + 1
x 1
x 2 + 1

1
7

Let us divide x4

B 2 C2
B 2C 2

b2 2ac
.
c2

OR
+ 2x3 + 8x2 + 12x + 18 by x2 + 5.

Hint: For zeroes: 21x2 3 = 0


1
x2 =
7
x= p

1
.
7

9. Since a = 2 is a zero of a3 3a2 10a + 24,


therefore a3 3a2 10a + 24 is divisible by
a 2. Further the obtained quotient will
provide the other two zeroes.
P O L Y N O M

A L S

19

Clearly, the remainder is 2x + 3.


Now, px + q = 2x + 3
Comparing the coefficients of like powers of
x both the sides, we get
p = 2, q = 3.

(a - b)2 = 62 4 4 = 20 a b, = 2 5
Thus, the difference of zeroes is 2 5 .
5.

B C B 2 C2
=
C B
BC

WORKSHEET 14
1. (C) Let each of two equal roots be .
b
c
Then + = and . =
a
a

b
c
a(4ac b2) = 0
=
a
2a

But a 0
So,
b2 4ac = 0

b2 = 4ac
This last equation holds iff a and c have
same sign.
2. (A) a + b =

3
1
, ab =
2
2

(a b)2 = (a + b)2 4ab

9
1
2=
4
4
1
ab=
2
1
1
a = , b = 1 or a = 1, b =
2
2
=

5
, b + 2 = 3 or a + 2 = 3,
2
5
b+2= .
2
Hence, the required polynomial can be

a+2=

x2

3
2

x+

15
5
11
3, i.e., x2
x+
.
2
2
2

3. p(x) = x2 (sum of zeroes) x + product of


zeroes
1
1
= x2 ( + 2)x +
2
2
2
5
= x2 x + 1
2
4. Let zeroes be a and b.
a + b = 6, ab = 4
Using (a b)2 = (a + b)2 4ab, we get

20

B C
2  2BC
BC

13
25  12
=
.
6
6

6. x2 1 = (x + 1) (x 1)
x = 1 or 1, both will satisfy with the
given polynomial.

We get, p + q + r + s + t = 0
...(i)
and
pq+rs+t= 0
...(ii)
From (ii),
p+r+t=q+s
2(q + s) = 0 q + s = 0 [From (i)]

p + r + t = q + s = 0.
7. No.
Hint: Divide q(x) by g(x). If the remainder
obtained is zero, then the g(x) is a factor of
q(x) otherwise not.
8. a = 1, b = 7
Hint: Put remainder = 0 and equate coefficient
of x in the remainder and constant term
with zero.
9. According to division algorithm,
p(x) = g(x) q(x) + r(x)
(i) p(x) = 6x2 + 3x + 2, g(x) = 3
q(x) = 2x2 + x, r(x) = 2
(ii) p(x) = 8x3 + 6x2 x + 7, g(x) = 2x2 + 1
q(x) = 4x + 3, r(x) = 5x + 4
(iii) p(x) = 9x2 + 6x + 5, g(x) = 3x + 2,
q(x) = 3x, r(x) = 5.
10. Given quadratic polynomial is
5 5 x2 + 30x + 8 5

= 5 5 x2 + 30x + 8 5
= 5 5 x2 + 20x + 10x + 8 5
= 5x

)
(
5 ) ( 5x + 4)

5x + 4 + 2 5

= 5x + 2

M A T H E M A T

5x + 4

I C S X

To find its zeroes, put 5 x 2 5 = 0 and


5 x + 4 = 0.

i.e.,

x=
x=

2
5

and x =

2x2 10x + 8

x2

4
5

3 2x4 10x3 + 5x2 + 15x 12


2
2x4
3x2

+
10x3 + 8x2 + 15x 12

2 5
4 5
and x =
5
5

10x3
+
8x2
8x2

2 5 4 5
6 5

=
5
5
5
And product of zeroes
So, sum of zeroes =

Also,

2 5 4 5 8
=

= .
5
5 5

Coefficient of x
sum of zeroes =
Coefficient of x 2

=
And product of zeroes =
=

30
6 5
=
5
5 5

Constant term
Coefficient of x 2
8
8 5
= .
5
5 5
Hence verified.

OR
q(x) = 3x2 2x + 1
Hint:

Let S =
P=

B1 C1

B 1 C 1

B  1 C  1

B 1 C 1

Required polynomial q(x) = x2 Sx + P.


3 and 3 are the zeroes of the given
2
2

3
quadratic polynomial, so x
and
2

3
x +
will be the factors of that. Conse2

11. As

3
3
3
2
quently, x
x +
, i.e., x
2
2
2

must be the factor of that. Let us divide


3
2x4 10x3 + 5x2 + 15x 12 by x2 .
2

P O L Y N O M

A L S

+ 15x

2x4

Now,

10x3

5x2

12
12
+

+ 15x 12

3 2
(2x 10x + 8)
2
By splitting 10x, we factorise 2x2 10x + 8
as (x 4) (2x 2). So, its zeroes are given
by x = 4 and x = 1.

= x2

Therefore, all zeroes of the given polynomial are

3,
2

3 , 1 and 4.
2

WORKSHEET 15
1. (D) Let zeroes be a and b, then
(a b)2 = 144

a b = 12
...(i)
a+b=p
...(ii)
ab = 45
...(iii)
Also, we have
(a b)2 = (a + b)2 4ab

144 = p2 180

p = 18.
2. 1 c
Hint: f (x) = x2 px (p + c)
( + 1) ( + 1) = + ( + ) + 1.
p
3.
r



B C H
Hint:
.



BC
CH
HB
BCH
4. Let the given linear polynomial be
y = ax + b
...(i)
This passes through points (1, 1), (2, 1) and
3
,
2

21

\ 1= a+b
...(ii)
1 = 2a + b
...(iii)
3
0=
a+b
...(iv)
2
Solving equations (ii) and (iii), we get a = 2,
b = 3 which satisfy to equation (iv).
Consequently, using equation (i), we get
y = 2x 3
Polynomial is p(x) = 2x 3
3
Since p(x) = 0 if x =
2
3

x=
is zero of p(x).
2
5. Let us divide ax3 + bx c by x2 + bx + c by
the long division method.
x

2+

ax ab
bx + c ax3 + bx c
ax3 + abx2 + acx

abx2 + (b ac) x c
abx2 ab2 x
abc
+
+
+
(ab2 + b ac)x + abc c

Put remainder = 0
(ab2 + b ac)x + (abc c) = 0
ab2 + b ac = 0 and abc c = 0
Consider abc c = 0 (ab 1) c = 0
ab = 1 or c = 0. Hence, ab = 1.
6. Hint: Let f (x) = x3 mx2 2npx + np2
(x p) is a factor of p(x)

f (x) = 0 at x = p.

p3 p2m p2n = 0

p2 [(p (m + n)] = 0
p = m + n since p 0.
7. x3 4x2 + x + 6
Hint: The required cubic polynomial is given
by (x 3) (x 2) (x + 1) or x3 4x2 + x + 6
This is the required polynomial.
8. 2, 3, 4
Hint:
++=5
+ + = 2
= 24
Let = 12

22

Solving

=2
+ =7
( )2 = 1
=1
= 1 or = 1
+ = 7 and = 1, we get
= 4, = 3
And solving + = 7 and = 1
we get
= 3, = 4.

9. f(x) would become exactly divisible by g(x)


if the remainder is subtracted from f(x).
Let us divide f(x) by g(x) to get the remainder.

x2 + 6x + 8
x2 4x + 3 x4 + 2x3 13x2 12x + 21
x4 4x3 + 3x2

+
3
6x 16x2 12x + 21
6x3 24x2 + 18x

2
8x 30x + 21
8x2 32x + 24

+
2x 3
Hence, we should subtract 2x 3 from f(x).
10. If 2

3 are zeroes of p(x), then x 2 + 3

and x 2 3 are factors of p(x).


Consequently

{x (2 + 3 )} {x (2 3 )}

i.e., (x 2)2 3, i.e., x2 4x + 1 is factor of


p(x).
Further,

x2 2x 35
x2 4x + 1 x4 6x3 26x2 + 138x 35
x4 4x3 + x2

+
3
2x 27x2 + 138x 35
2x3 + 8x2 2x
+

+
35x2 + 140x 35
35x2 + 140x 35

+
+
0
M A T H E M A T

I C S X

Clearly x2 2x 35 is a factor of p(x)


(x 7)(x + 5) is a factor of p(x)
x 7 and x + 5 are factors of p(x)
x 7 = 0 and x + 5 = 0 give other zeroes
of p(x)
x = 7 and x = 5 are other zeroes of p(x).
Hence, 7 and 5 are required zeroes.

+
+ 2 =
2

2 2
2

11. Hint:

{( + )2 2}2 2 22
.
2 2

OR
Given polynomial is:
f (x) = pqx2 + (q2 pr)x qr
= pqx2 + (q2 pr)x qr
= pqx2 + q2x prx qr
= qx(px + q) r(px + q)
= (px + q)(qx r)
px + q = 0 and qx r = 0 provide the zeroes
q
r
of f (x). So zeroes are and .
q
p
q
r pr q2
Sum of zeroes = + =
p q
pq

Coefficient of x
Coefficient of x 2

So,

b2 4ac = 0

b2 = 4ac
This last equation holds iff a and c have
same sign.
2. Required quadratic polynomial
= x2 (sum of zeroes)x + product of zeroes
= x2 2 3x 5 3 .
3.

p(x)= x2 ax (a + 1)
At x = 1, p(x) = (1)2 a( 1) (a + 1)
=1 + a a 1 = 0
q(x) = ax2 x (a + 1)
At x = 1, q(x) = a( 1)2 ( 1) (a + 1)
=a + 1 a 1 = 0
Therefore, x + 1 is the common factor of p(x)
and q(x).

4. Correct,
f (x) = x2 p(x +1) c= x2 px (c + p)

+ = p; = (c + p)
Now, ( + 1) ( + 1)= + ( + ) + 1
= (c + p) + p + 1
=cp+p+1
= 1 c.
5. Let f(x) = 6x3 + 2 x2 10x 4 2
As 2 is a zero of f (x), (x 2 ) is a factor
of f (x).
Let us divide f(x) by x 2 .

q r
qr
Product of zeroes = p q = pq
=

Constant term
.
Coefficient of x 2

ASSESSMENT SHEET 3
1. (C) Let each of two equal roots be .
Then + =

b
c
and . =
a
a

c
b
2

= a a(4ac b ) = 0
a
2

But a 0.
P O L Y N O M

A L S

(
)
= (x 2 )
= (x 2 )

f (x) = x 2

7 2 x 4

6x

3 2 x 4 2x 4

2 x 4
2 x 1

6x

23

Hence, 3 2 x 4
2 x 1
gives x =

2 2
3

1
or x =
2

8. Let us divide p(x) by 2x2 5.

3x2 + 4x + 5
2x2 5 6x4 + 8x3 5x2 + ax + b
6x4

2 2
Therefore, other two zeroes are
and
3
2

.
2

3 5
y5
6. p(y) = y2 +
2
1
= (2y2 + 3 5 y 10)
2
1
= (2y2 + 4 5 y 5 y 10)
2
1
= [2y(y + 2 5 ) 5 (y + 2 5 )]
2
1
= (y + 2 5 ) (2y 5 )
2
y + 2 5 = 0 and 2y

5 = 0 give the required

zeroes, that are 2 5 and

5
.
2

= 2(1) + 2 [Using (i)]


...(iii)

And (2 + 1) (2 + 1) = 4 + 2 + 2 + 1
= 4 + 2 ( + ) + 1
= 4 ( 2) + 2 (1) + 1
[Using (i) and (ii)]
=5
...(iv)
Now, required polynomial can be given by
x2 {(2 + 1) + (2 + 1)}x + (2 + 1)(2 + 1)
i.e., x2 4x 5.

24

10x2 + (20 + a)x + b


10x2
25

+
(20 + a)x + b + 25
Here, remainder is (20 + a)x + b + 25.
If the polynomial p(x) is exactly divisible by
2x2 5, the remainder must be zero.
(20 + a)x + (b + 25) = 0
Comparing the coefficients of like powers of
x between both the sides, we have
20 + a = 0 and 25 + b = 0
a = 20 and b = 25.

ASSESSMENT SHEET 4

7. and are zeroes of f (x) = x2 x 2


1
Sum of zeroes = + =
=1
...(i)
1
2
Product of zeroes = =
=2
...(ii)
1
(2 + 1) + (2 + 1) = 2( + ) + 2

=4

15x2
+
8x3 + 10x2 + ax + b
8x3
20x
+

[Using (iii) and (iv)]

1. (C) Sum of zeroes =


Product of zeroes =

3 2
=
3

1
.
3

2. At x = 2, p(x) = 0, i.e., p(2) = 0

a (2)2 3 2 (a 1) 1 = 0

4a 6a + 6 1 = 0
5

a= .
2
3.
Sum of zeroes = + = 5
Product of zeroes = = 4
Now,

1
+
1
+
2 =
2

5
24
4
27
=
.
4
4. Using division algorithm, we have
g(x) (x 2) 2x + 4 = x3 3x2 + x + 2

M A T H E M A T

I C S X

g(x) =

x 3 3x 2 3x 2
x2

Thus, the zeroes are =


Sum of zeroes = +

Here, at x = 2,

x3

3x2

x3 3x2 + 3x 2

= 8 12 + 6 2 = 0
+ 3x 2
= (x 2) (x2 x + 1)

3
5 3
2 3
+
=
4
12
3

5
5 3
=
4 3
43

Coefficient of x
Coefficient of x 2

( x 2)( x2 x 1)
( x 2)

g(x) =

g(x) = x2 x + 1.

5. Given s = 2 and p =

Product of zeroes = =

3
2

The required polynomial is given by


k[x2 sx + p]
i.e.,

k x2

3
2 x , where k is any real
2

number.
Let us find zeroes of this polynomial.
k(x2

2x

k
3
) = (2x2 2 2 x 3)
2
2

=
2 x 3 = 0 and
zeroes.

Hence

3
2 3
and =
4
3

k
( 2 x 3)( 2 x + 1)
2

2 x + 1 = 0 provides the

3
2 3
.
4
3

Constant term
2 3
=
.
4 3
Coefficient of x 2

Hence verified.
7. (i) Let

y = p(x)
y = x2 + x + 6

The table for some values of x and their


corresponding values of y is given by
x

Let us draw the graph of p(x) using this


table.

2
3 2
and
are the required
2
2

zeroes.
6. Let f(x) = 4 3 x2 5x 2 3
= 4 3 x 2 8 x 3x 2 3
= 4x( 3x 2) 3 ( 3x 2)
= ( 3x 2)(4x 3)
To find zeroes of f (x), put

3x 2  0

x

2
3

and 4 x 3  0
2 3
and x 
3

P O L Y N O M

A L S

3
4

25

From the graph, it is clear that the zeroes of


p(x) are 2 and 3.

(ii)

Let y = p(x)

y = x3 4x

Since, x4 + x3 + 8x2 + ax + b is divisible by x2 + 1,


therefore remainder = 0
i.e.,(a 1)x + (b 7) = 0 or (a 1)x + (b 7) = 0.x + 0
Equating the corresponding terms, we have

The table for some values of x and their


corresponding values of y is given by

a1=0

and

b7=0

a=1

and

b=7

i.e.,

(ii) Common good, Social responsibility.

CHAPTER TEST

Let us draw the graph of p(x) by using this


table.

2
Zeroes are given by

1. Let p(x) = x x

x = 0 and x

7
= 0.
2

Hence zeroes are 0 and


2. Q + =

7
.
2

5
1
, =
2
2
+ + = 2.

= x2 ( + )x +

3. p(x)

= x2 + x 2.
4. Let p(x) = 2x3 + 4x2 + 5x + 7
Now, p(x) = g(x) 2x + (7 5x)

p( x) (7 5x )
2x
2x 3 4 x 2 5x 7 7 5x
=
2x
= x2 + 2x + 5.

From the graph, it is clear that the zeroes of


p(x) are 2, 0 and 2 .
8. (i) First we divide x4 + x3 + 8x2 +ax + b by
x2 + 1 as follows:
x2 x 7
x 1 x x 8 x ax b
2

g(x) =

5. 6 5 , 9
5
4

x4 x2

Hint:

 

x 3 7 x 2 ax b
x3 x

 

7 x 2 ( a  1)x b
7 x2 7

( a  1)x (b  7)

26

+=

6.

1
3
Hint:

Coefficient of x
Coefficient of x 2

Constant term
Coefficient of x 2

+= 0

b
= 0.
a

M A T H E M A T

I C S X

7. f (x) = ax3 + bx2 + cx + d

respectively the quotient and the remainder, the


division algorithm is
p(x) = g(x) . q(x) + r(x)
...(i)

g(x) = ax2 + bx + c
q(x) = x
r(x) = d.
8. If , and are the zeroes of a cubic
polynomial f(x), then
f(x) = x3 ( + + ) x2
+ ( + + ) x
Here, + + = 4, + + = 1

According to the question,


p(x) = 3x3 + x2 + 2x + 5
q(x) = 3x 5
r(x) = 9x + 10
Substituting these values of p(x), q(x) and r(x) in
the equation (i), we get

= 6

and

9. We have

3x3 + x2 + 2x + 5 = g(x) . (3x 5) + 9x + 10

f(x) = x3 4x2 + x + 6.

4 3 x 2 + 5x 2 3 =

)(

3x + 2 4x 3

So, the value of 4 3 x 2 5x 2 3 is zero


when,

3 x + 2 = 0 or 4 x 3 = 0,

i.e., when x =

(3x 5) g(x) = 3x3 + x2 + 2x + 5 9x 10


= 3x3 + x2 7x 5

To find g(x), we proceed as following:

3
or x =
.
4
3

x2 2x 1
3x  5 3x3 x2  7 x  5
3x3  5x2

Therefore, the zeroes of 4 3x 2 + 5x 2 3


are

2
3

and

3x 3 x 2  7 x  5
3x  5

g(x) =

3
.
4

6x2  7 x  5
6 x 2  10 x

5
3
+
=
4
4 3
3

Now, sum of zeroes


=

Coefficient of x

Coefficient of x 2

3x  5
3x  5

Product of zeroes =

=
10. Let p(x)

s
3

3
2 3
=
4
4 3

Constant term
.
Coefficient of x 2

= Total Relief Fund

g(x) = Number of families who


received Relief Fund
q(x) = Amount each family received
r(x) = Amount left after distribution
When the polynomial p(x) is divided by a
polynomial g(x) such that q(x) and r(x) are

P O L Y N O M

A L S

Thus, g(x) = x2 + 2x + 1.
(ii) Common good, Accountability, social
responsibility.
11. Since

1
1
and
are zeroes.
3
3

1
1

Therefore, x
x +
will be a factor
3
3

of p(x), i.e., x 2

1
is a factor of p(x).
3

27

3x2 15x + 18

x2

1 3x4 15x3 + 17x2 + 5x 6


3
3x4
x2
+

15x3 + 18x2 + 5x 6
15x3
+ 5x

+
18x2 6

Here, 3x2 15x + 18 = x2 5x + 6


= (x 3) (x 2)
Other zeroes are given by x 3 = 0 and
x 2 = 0.
So, other zeroes are 3 and 2.
1
1
Hence all the zeroes are
,
, 3 and 2.
3
3
qq

18x2 6

+
0

28

M A T H E M A T

I C S X

Chapter

PAIR OF LINEAR EQUATIONS IN TWO VARIABLES


WORKSHEET 18

1. (B) Since (3, a) lies on the line 2x 3y = 5,


therefore, (3, a) must satisfy this equation.

2 (3) 3 (a) = 5

a=

3a = 1

1
.
3

2. k = 6
Hint:

3
2
.
=
k
9

3. k = 6
Hint: The condition of inconsistency of two
equations a1x + b1y = c1 and a2x + b2y = c2 is
given by

a1 b1 c1
=
x .
a2 b2 c2

4. Adding the given equations, we get


80x + 80y = 240
or
x + y= 3
...(i)
Subtracting given first equation from other
one, we get
6x 6y = 6
or
x y = 1
...(ii)
Solving equations (i) and (ii), we obtain
x = 1, y = 2.
5. x = 3, y = 2
Hint: Let

= u,

x y

1
xy

6. False.
Let us substitute c = 40, The given equations
become
x 2y = 8
or
5x 10y = 40

P A

2
1
8
=
=
10
5
40
R

O F

N E A R

y
1
x
=
=
1
5
2

x = 2 and y = 5
Hence, x = 2, y = 5 is the solution of the
given system of equations.
8. To draw a line, we need atleast two
solutions of its corresponding equation.
x + 3y = 6; at x = 0, y = 2 and x = 3, y = 1.
So, two solutions of x + 3y = 6 are:

= v.

Given equations become


10u + 2v = 4 and 15u 5v = 2.

Here,

The equations represent a pair of


coincident lines.
The equations have infinitely many
solutions at c = 40 and no solutions at
c 40.
For no value of c, the given pair has a
unique solution.
7. The given equations are
4(2x + 3y) = 9 + 7y
and
3x + 2y = 4
or
8x + 5y 9 = 0
3x + 2y 4 = 0
By cross-multiplication, we have
y
1
x
=
=
 32 + 27
16  15
20 + 18

2x 3y = 12; at x = 0, y = 4 and at x = 6, y = 0
So, two solutions of 2x 3y = 12 are:
x

Now, we draw the graph of given system


of equations by using their corresponding
solutions obtained in the above tables.

E Q U A T

I O N S

...

29

2. x 5y = 5.
(2, k) lies on it.

2 5(k) = 5

5(k) = 3

3
.
5
3. Condition for parallel lines is

k=

b
a1
c
= 1 1
b2
c2
a2
1
=
3

3
2

1
k

k = 6.

4. The given lines to be coincident, if


From the graph, the two lines intersect the
y-axis at (0, 2) and (0, 4).
9. Let the fixed charges and change per km
be ` x and ` y respectively.
x + 10y = 105
...(i)
x + 25y = 255
...(ii)
Subtracting equation (i) from equation (ii),
we get
15y = 150
y = 10
...(iii)
From equations (i) and (iii), we get
x= 5
Now, the fare for travelling a distance of
35 km
= x + 35y
= 5 + 35 10
= ` 355.
Fixed charge = ` 5
Charge per km = ` 10
Total charge for 35 km = ` 355.

WORKSHEET 19
1. (D) As y = 2 and y = 7,
both represent straight lines parallel to
x-axis
y = 2 and y = 7 are parallel lines.
Hence, the given pair of equations has no
solution.

30

( k 3)
3
k
=
=
k
k
12
III
II
I

Taking I and II, we have


k2 = 36 k = 6.
Taking II and III, we have

...(i)

k2 3k = 3k k(k 6) = 0
k = 0 or 6
Using (i) and (ii), we obtain

... (ii)

k = 6.
5. x = 5, y = 2
Hint: Adding the given equations,
we get 2x + y = 12

...(i)

Subtracting the given equations,


we get 3x + y = 17

...(ii)

6. Yes.
Applying the condition
c1
a1 b1
=
= c ,
a2 b2
2

we have

3
2
1
=
=
9
6
3
That is true.
Therefore, the pair of equations is consistent
with infinitely many solutions.
M A T H E M A T

I C S X

6y = 3000 y = 500
Substituting y = 500 in equation (ii),
we get
x = 13000
Hence, starting salary was ` 13000 and
annual increment was ` 500.
The value imbibe by the man are:
consistency, hard work and sincerity

615
255
, y=
104
52
Hint: The system:
9x 10y + 15 = 0
5x + 6y 60 = 0
By cross-multiplication, we have
y
x
1
=
=
.
600  90  540  75 54 + 50

7. x =

WORKSHEET 20

8. For equation 3x + y 2 = 0,
x
y

x
y

1. (D) Line x = a is parallel to y-axis and the


line y = b is parallel to x-axis. These lines
intersect each other at (a, b).
2. As the lines are intersecting each other,
3
2
3

a
.
a
1
2
3. 3x y 5 = 0 and 6x 2y k = 0 have no
solution
These equations represent a pair of
parallel lines.
5
1
3
=

k
2
6

k 10.

2
1
For equation 2x 3y 5 = 0,

4. No.
For infinitely many solutions, the following
condition must be satisfied.
3
7
=
=
2 6
14
1
7
1
3
But, here

as

2
6
14
2
Hence, no value of provides the pair of
infinitely many solutions.
As the lines corresponding to the given
equations intersect each other at (1, 1),
the required solution is x = 1, y = 1.
9. Let the man's starting salary and fixed
annually increment be x and y respectively.
According to the question,
x + 4 y = 15000
...(i)
x + 10 y = 18000
...(ii)
Equations (i) and (ii) form the required pair
of linear equations. Let us solve this pair.
Subtracting equation (i) from equation (ii),
we get
P A

O F

N E A R

5. The given system of equations can be written


as
ax + by (a b) = 0
bx ay (a + b) = 0
By cross-multiplication,
y
x
=
a( a + b) + b ( a b)
b( a + b) a ( a b)
(ii)
(i)

1
a2 b 2
(iii )

E Q U A T

I O N S

...

31

Taking (i) and (iii), we get


x=

ab b2 a2 + ab
a2 b 2

(l 7)(b + 5) = lb
From equation (i),

=1

Taking (ii) and (iii), we get


y=

a2 ab + ab b2
a2 b 2

...(ii)

=1

Hence x = 1, y = 1 is the solution of the


given system of equations.
6. x = 6, y = 4, m = 0
1
1
Hint: Take
= u and
= v.
x
y
7. No; (6, 0) , (4, 0)
Hint: For x + 3y = 6
x

WORKSHEET 21

For 3x + 9y = 12
x

lb + 7b 3l 21 = lb

7b 3l = 21
...(iii)
From equation (ii),
lb 7b + 5l 35 = lb

7b + 5l = 35
...(iv)
Adding equations (iii) and (iv), we get
2l = 56 l = 28 m
Putting the value of l in equation (iii), we get
b = 15 m.
l = 28 m, b = 15 m.
(ii) Solution of system of linear equations in two
variables.
(iii) Love for environment and human beings.

Let us draw the graph of lines using the


tables obtained above.

1. (D) Let unit's and ten's digit be x and y


respectively.
x + y= 9
...(i)
10y + x + 27 = 10x + y
...(ii)
Solving equations (i) and (ii), we have
x = 6, y = 3
Hence, the required number is 3 10 + 6,
that is, 36.
2. Given equation is 5(x y) = 3
5x 5y 3 = 0
Let a2 = 10, b2 = 10 and c2 = 6
a1 b1 c1
For coincident; a  b  c
2
2
2
a1
5 1 b1
5 1
Hence a  10  2 ; b  10  2
2
2

In the graph, lines are parallel. So, the pair


of equations is not consistent.
The lines intersect the xaxis at (4, 0) and
(6, 0).
8. (i) Let l = length of the rectangle
b = breadth of the rectangle
According to question,
(l + 7)(b 3) = lb

32

c1 3 1
and c  6  2
2
a1 b1 c1
a b c
2
2
2

So required equation which can coincide is


10x 10y 6 = 0
3. p = 6
Hint:

...(i)

3
5
p=6
=
p 10

M A T H E M A T

I C S X

Note: At p = 6, the given system has both


zero and non-zero solutions.
4. a = 5, b = 1
Hint: According to the condition of infinitely
many solutions, we reaches at
a+b
2a b
21
=
=
.
2
3
7

5. x = 1, y = 1
Hint: Simplifying the given linear
equations, we have
7
2
8
7

= 5, + = 15
y
x
y
x
1
1
= u,
= v; and solve.
y
x

Now take

The lines intersect y-axis at (0, 5) and


(0, 5).
9.
Total distance = 300 km
Let speed of train = x km/h
and speed of bus = y km/h

a + 4b
4ab
, y=
5b
5a
x
Hint:
3b (2 a + b) + 2b ( a + 2b)

6. x =

y
=
2 a (2 a + b) + 3 a ( a + 2b)

1
2 a 2b 3 a 3b

Take first and third terms as well as second


and third terms and solve.
7. a = 7, b = 3
Hint: For infinitely many solutions,
(a 4 )
2b 1
2
=
=
5b  1
4
( a 1)

O F

N E A R

...(i)

15
60
+
=1
x
y
From equation (ii),

...(iii)

4 8
1
+ =
x y
6

1 2
1
+ =
...(iv)
x y
24
We will solve equations (iii) and (iv) by
elimination method.
Applying (iii) 15 (iv), we get:

Let us draw the graph of lines using the


tables obtained above.
P A

60 240
+
= 4 hours
y
x

25
100 200
+
=
hours
6
y
x

Similarly table for equation 2x y 5 = 0 is


x

According to question,

200
100
+
= 4 hours and
y
x
10 minutes. ...(ii)
From equation (i),

8. Table for values of x and y as regarding


equation 3x + y 5 = 0 is
x

distance
time
distance
time =
speed

speed =

and

1
1
a4
2b 1
=
and
=
.
2
5b  1
a 1
2

Take

As

E Q U A T

I O N S

...

33

60
15
+ y =1
x

6
8
3
4

x

12
6
6
3

15
30
15
+
=
x
y
24

Required value of k, can be 6 or 6.

3. Let the required equation be ax + by + c = 0.


a
b
c
Then,
=

2
3
5
b
a

=
= k (say)
2
3

30
15
y = 1 24
30
9
=
y
24

(True)

30 s 24
= 10 8 = 80 km/h
9
Putting the value of y in equation (iv), we get,
1
2
1
+
=
x
80
24
1
1
1
40  24
16

=
=
x
24
40
24 s 40
24 s 40
1
=
x = 60 km/h
60
(ii) Solution of system of linear equations in
two variables.
(iii) By opting for public transport it depicts that
she is a responsible citizen, so her
responsibility and rationality have been
depicted here.

y=

WORKSHEET 22

a=

2 k, b = 3 k, k

c
any real
5

number
Then, 2 kx

3 ky + c = 0
c

2x 3 y + = 0
k
Putting k = c, we have

2x

3 y = 1.

4. For infinite number of solutions, we have


3
2
7
=
=
p+q
( p q 3)
(4 p q)
2
3
On solving,
=
and
p+q
( p q 3)

7
3
=
,
(4 p + q)
( p + q 3)
we obtain p = 5, q = 1.

1. (D) The condition to be coincident for lines


ax + by + c = 0 and dx + ey + f = 0 is given
by
a b c
= =
f
d e

ae = bd; bf = ce.

5. x = 1, y = 2
Hint: Adding and subtracting the given two
equations, we have
x+y=3
...(i)
and
xy=1
...(ii)
Now, solve equations (i) and (ii).

Note: Two lines are coincident if both the


equations follow the condition of infinitely
many solutions.

6. x = a2, y = b2
Hint: Given system of linear equations may
be written as
bx + ay ab (a + b) = 0
b 2x + a 2y 2 a2 b 2 = 0
Solve these two equations by the method of
cross-multiplication.
7. Let the two digits number be 10x + y.
Since ten's digit exceeds twice the unit's
digit by 2

2. (A) For no solutions,


(k 2)
k
3
=

12 k
k
If

If

34

k=6

k= 6
6
3
62
4
2

x
 
12
6
6
6
3
k= 6

(True)

M A T H E M A T

I C S X


x = 2y + 2

x 2y 2 = 0
...(i)
Since the number obtained by interchanging the digits, i.e., 10y + x is 5 more
than three times the sum of the digits.
...
10y + x = 3 (x + y) + 5
2x 7y + 5 = 0
...(ii)
On solving equations (i) and (ii), we obtain
x = 8 and y = 3
.. .
10x + y = 83
Hence, the required two-digit number is 83.
8. Tables for equations 3x + y 11 = 0 and
x y 1 = 0 are respectively
x

perpendicular AM from A on the y-axis to


intersect it at M.
Now, in ABC,
base BC = 11 + 1 = 12 units,
height AM = 3 units.
1
... ar(ABC) = base height
2

1
12 3 = 18 sq. units
2
Hence, x = 3, y = 2; area = 18 sq. units.
=

9. Speed of boat = 6 km/hr,


Speed of stream = 2 km/hr
Hint: Let the speed of boat in still water =
x km/h and the speed of stream = y km/h
12
40
+
= 8
xy
x+y

and

...(i)

Distance
Using Time 

Speed
16
32
+
= 8
xy
x+y

Let us draw the graph.

...(ii)

Put x y = u, x + y = v and solve further to


find x and y.
OR
Let each boy receives ` x and the number of
boys be y. Then sum of money which is
distributed is ` xy.
Had there been 10 boys more, each would
have received a rupee less,
...
(y + 10) (x 1) = xy

10x y = 10

...(i)

Had there been 15 boys fewer, each would


have received ` 3 more,
...
(y 15) (x + 3) = xy

From the graph, it is clear that the lines


intersect each other at a point A(3, 2). So
the solution is x = 3, y = 2.
The line 3x + y 11 = 0 intersects the
y-axis at B(0, 11) and the line x y 1 = 0
intersects the y-axis at C (0, 1). Draw the
P A

O F

N E A R

5x y = 15
Solving (i) and (ii), we get

...(ii)

x = 5 and y = 40
...
xy = 200
Hence, sum of money = ` 200
And number of boys = 40.

E Q U A T

I O N S

...

35

WORKSHEET 23

1. (B) As the given equations are homogeneous


so only solution will be x = 0, y = 0.
2. (C) x = 4, y = 9

1
1
=u,
= v and solve.
y
x
3
2
9
3. Here,

x
6
9 5
.
. . Lines are parallel.

Hint: Put

4. The given equations have a unique solution


a
b

am bl.
m
l
5. The given equation can be written as
6ax + 6by = 3a + 2b
...(i)
and 6bx 6ay = 3b 2a
...(ii)
Multiplying equation (i) by a and (ii) by b
and adding the results, we have
6(a2 + b2)x = 3(a2 + b2)
1

x=
2
1
Substituting x = in equation (i), we have
2
6a
+ 6by = 3a + 2b
2
1

6by = 2b
y=
3
1
1
Thus, the solution is x = , y = .
3
2
6. a = 5, b = 1
Hint: Two linear equations a1 x + b1 y + c1 = 0
and a2 x + b2 y + c2 = 0 have infinite number
of solutions if
a1
b
c
= 1 = 1.
a2
b2
c2
7. Given system of linear equations can be
written as
(a + 2b) x + (2a b) y 2 = 0
(a 2b) x + (2a + b) y 3 = 0
By cross-multiplication,
x
3 (2 a b ) + 2 (2 a + b)
(i )

36

y
3 ( a + 2b) + 2 ( a 2b)
(ii )

1
( a + 2b) (2a + b) (2a b) ( a 2b)
(iii)

Taking (i) and (iii), x =

5b 2a
10 ab

Again taking (ii) and (iii), y 

Thus, x =

a + 10 b
10 ab

5b 2a
a + 10 b
, y =
is the
10 ab
10 ab

solution of the given system of equations.


8. Speed of rowing = 6 km/hr,
Speed of current = 4 km/hr
Hint:

2 (x + y) = 20
[Time Speed = Distance]

Where,

2 (x y) = 4
x = speed of rowing and,
y = speed of current.
OR

Let fare from A to B and from A to C be


` x and ` y respectively.
According to the given conditions,
2x + 3y = 795
...(i)
3x + 5y = 1300

...(ii)

Solving eqn. (i) and (ii), we obtain


x = 75, y = 215
Hence, fares from A to B is ` 75 and from A
to C is ` 215.
9. Let us make the table for the values of x and
corresponding values of y to the equation
2x + y 8 = 0
x

M A T H E M A T

I C S X

WORKSHEET 24

Similarly, for the equation x y 1 = 0


x

1. (A) In the case of no solution,

1
5
3
=

k 10.
k
6
2

Let us draw the graph.

2. (D) x = 80, y = 30
Hint: x + 2y = 140, 3x + 4y = 360.
3. For unique solution,
p
4

p 4.
2
2
4. True.
According to the conditions of consistency,
2
2
5
5
1
3
either
x
or 3 =
=
3 5
3
5
3
2
2

Clearly, the first condition holds. Hence,


the system of equations is consistent with
a unique solution.
5. For infinitely many solutions,
p+q
=
3

4p + 4q =
12p 12q =
2p 10q =

From the graph, the lines intersect each other


at the point A(3, 2). Therefore, the solution
is x = 3, y = 2.
The lines intersect the y-axis at B(0, 8) and
C(0, 1).
To find the area of the shaded portion,
that is, ABC, draw perpendicular AM
from A on the y-axis to intersect it in M.
Now, AM = 3 units and BC = 8 + 1 = 9 units.
...

ar(ABC) =
=

O F

Given equation can be written as:


3
u+v =
4

4u + 4v = 3

1
93
2

N E A R

1
1
= u,
=v
3x  y
3x y

Hint: Take

and

p = 5, q = 1.

6. x = 1, y = 1,

1
BC AM
2

27
=
sq. units
2
Hence, x = 3, y = 2; area = 13.5 sq.units.

P A

(5p 1)
2( p q)
=
12
4
6p 6q and
15p + 3
0 and 3p 12q = 3

7. x =

1
1
1
u v =
2
2
8

4u 4v = 1.

1
2

,y=

Hint: Put

E Q U A T

1
3

1
1
= u and
= v.
x
y

I O N S

...

37

Distance travelled by the car that starts


from A
AC = 5 x
Similarly distance for other car
BC = 5 y
...
AC BC = 5x 5y

5x 5y = 100

x y = 20
...(i)
Case II: When two cars travel in opposite
directions.
Let the cars meet at D

8. Table for values of x and y corresponding


to equation 4x 5y 20 = 0 is
x

Similarly for the equation 3x + 5y 15 = 0


x
y

5
0

0
3

Let us draw the graphs for the two equations.

Distance travelled by the car that starts


from A
AD = 1 x
Similarly distance for other car
BD = 1 y
...

AD + BD = x + y

x + y = 100

...(ii)

Solving equations (i) and (ii), we get


x = 60 and y = 40
Hence, speeds of two cars that start from
places A and B are 60 km/h and 40 km/h
respectively.
As the graphs of the two lines intersect
each other at the point A(5, 0), the required
solution is x = 5, y = 0.
The graphs intersect the y-axis at B (0, 3)
and C(0, 4). Therefore, the coordinates of
vertices of the triangle ABC are A(5, 0),
B(0, 3) and C(0, 4).
Hence the answer: x = 5, y = 0 and (5, 0),
(0, 3), (0, 4).
9. Let speeds of two cars that start from places
A and B be x km/hr and y km/hr respectively.
Case I: When two cars travel in same direction.
Let the cars meet at C

WORKSHEET 25
1. (B)

...(i)
...(ii)

(Subtracting)
.. .
x = 2.
Further y = x = 2.
2. The given equations represent to be parallel
lines if
1
2( k 1)
1
=

1
3
1

38

x y = 0
2x y = 2
+

x = 2

3
2
1
k = .
2

k 1 =

M A T H E M A T

I C S X

3. m 4

Solving equations (i) and (ii), we get

m 2
.
x
2
1
4. For the point of intersection of any line with
x-axis, put y = 0
...
3x + 7 (0) = 3

x =1
So the required point is ( 1, 0).

Hint:

5. For inconsistency,
k+2
6
(3k 2)2
=

3
2
4

Hence,

Hence, Meena received 10 notes of ` 50 and


15 notes of ` 100.
OR
Let the length and breadth of rectangle be
x units and y units respectively.
Then area of rectangle = xy sq. units
Case I. The length is increased and breadth
is reduced by 2 units.
...
(x + 2) (y 2) = xy 28

k + 2 = 4 and (3k + 2)2 8

1
2 2 2
k = 2 and k
3
k = 2.

x = 10, y = 15

6. Given system of equations can be written


as
2x + 3y 18 = 0
...(i)
x 2y 2 = 0
...(ii)
2
3
x
Now,
1 2
Hence the system has unique solution.
Now, by cross-multiplication on (i) and (ii),
we get
y
x
1
=
=
6 36
4 + 18
43

1
1
 = u,
 = v and solve.
x+y
xy
8. Let Meena received x notes of ` 50 and y
notes of ` 100
Since total number of notes is 25
...
x + y = 25
...(i)
Since the value of both types of notes is
` 2000.
...
50x + 100y = 2000

x + 2y = 40
...(ii)

O F

N E A R

...(i)

Case II. The length is reduced by 1 unit


and breadth increased by 2 units.

(x 1) (y + 2) = xy + 33

xy 2 y + 2x = xy + 33
2x y = 35

...(ii)

Solving equations (i) and (ii), we get


x = 23 and y = 11
Hence, the length of the rectangle is 23 units
and the breadth is 11 units.
9. The given linear equations are
and

Hint: Take

x y = 12

4x y 8 = 0
2x 3y + 6 = 0

...(i)
...(ii)

To draw the graphs of the equations (i) and


(ii), we need atleast two solutions of each of
the equations. These solutions are given
below:

x = 6, y = 2
Thus, the solution of given system is
x = 6, y = 2.
7. x = 5, y = 1

P A

xy 4 2 x + 2 y = xy 28

y = 4x 8

x
2x 6
y=
3

Plot the points A(0, 8), B(2, 0), C(0, 2) and


D( 3, 0) on graph paper and join them to
form the lines AB and CD as shown in
figure.

E Q U A T

I O N S

...

39

4. False.
As

a1
2 b1
5 c1
= ,
=
,
=6
a2
4 b2
10 c 2

...

a1
b1
c1
=

a2 b2 c2

They are parallel.


5. a = 1, b =

5
2

2
(2a + 5)
5
=
= .
2b + 1
9
15
1
1
6. Put
= u and
= v in given system of
y
x
equations.
u + v 7= 0
...(i)
2u + 3v 17 = 0
...(ii)
By cross-multiplication,
u
v
1
=
=
17 + 21
17 + 14
32
u = 4, v = 3
1
1
x= ,y=
4
3
1
1
Hence, x = , y =
is the solution of the
3
4
given system of equations.
Hint:

The graphs of these lines intersect each


other at P(3, 4). So, unique solution:
x = 3, y = 4.
Also, the graphs meet the x-axis at D( 3, 0)
and B(2, 0).
Hence, the triangle formed by the lines and
the x-axis is PBD with vertices P(3, 4), B(2,
0) and D( 3, 0).

WORKSHEET 26
1. (C) For coincident lines,
7
1 2
= =
k
14
2

k = 4.
2. (A) The given system of equations can be
written as
x + 2y = 140, 3x + 4y = 360
Solving this system, we obtain
x = 80, y = 30.
3. Adding the given equations, we have
3x = 0 x = 0
Substituting x = 0 in any of the given
equations, we get y = 0
Hence, the required solution is x = 0, y = 0.

40

7. x = 2, y = 5 and m = 1
Hint: 2x + 3y = 11 y =

11 2x
3

Substitute this value of y in 2x 4y = 24


and solve for x.
8. The given system of linear equations is
2x y 5 = 0
...(i)
3x + y 5 = 0
...(ii)
To draw the graph of equations (i) and (ii),
we need atleast two solutions of each of the
equations, which are given below:
x

y = 2x 5

y = 3x + 5

M A T H E M A T

I C S X

WORKSHEET 27

Using these points, we are drawing the


graphs of lines as shown below:

1. (A) A = 70, B = 53,


C = 110, D = 127.
Hint: In a cyclic quadrilateral ABCD,
A + C = 180 and B + D = 180.
2. (C) x = 0, y = 0
Hint: Both lines are passing through the
origin.
3. For infinite number of solutions,
p+q
2 p q 21
=
=
2
7
3

p + q = 6 and 2p q = 9

p = 5 , q = 1.

4. False.
Hint: As a + 5b = 10.
5. False, x = 4, y = 1 does not satisfy the second
equation.
From the graph, the lines intersect each other
at the point P(2, 1). Therefore, the solution
is x = 2, y = 1.
The lines meet the y-axis at the points
Q(0, 5) and R(0, 5).
9. Let the fixed charge and additional charge
for each day be ` x and ` y respectively.
Since Saritha paid ` 27 for a book kept for 7
days
...
x + 4y = 27
...(i)
Also, Susy paid ` 21 for the book kept for 5
days
...
x + 2y = 21
...(ii)
Subtracting equation (ii) from (i), we get
2y = 6 y = 3
Again substituting y = 3 in equation (ii),
we get
x = 15
Hence, the fixed charge is ` 15 and
additional charge for each day is ` 3.
OR
Son's age = 10 years, father's age = 40 years.
Hint: Let the present age of father and son
be x and y years respectively.
...
x + 5 = 3 (y + 5)
And
x 5 = 7 (y 5).
P A

O F

N E A R

6. No solution
Hint: 2x + 3y = 7, 6x + 9y = 11
2
3
7
Here, =
Parallel lines.
x
6
9
11
7. The given system of linear equations can be
written as
px + qy (p q) = 0
qx py (p + q) = 0
To solve the system for x and y, using the
method of cross-multiplication, we have

y
x
=
q( p + q) p( p q )
p( p + q) + q( p q)
1
=
2
p q2

x
1
y
=
=
2
2
2
2
p q
p q2
p q
2

x = 1, y = 1.

8. The given system of equations can be


written as
3x 4y 1 = 0
...(i)
6x 8y + 15 = 0
...(ii)
To draw the graph, we need atleast two
solutions for each of the equations (i) and
(ii), which are respectively given below:

E Q U A T

I O N S

...

41

3
2

3
2

11
2

x
6x 15
8

x = 11, y = 9
Hence, the required fraction is

3x 1
y=
4

y=

Let us draw the graph by using these points.

11
.
9

OR
` 6000, ` 5250
Hint: Let incomes of X and Y be 8x and 7x
respectively; and expenditures of them be
19y and 16y respectively.

8x 19y = 1250
...(i)
7x 16y = 1250
...(ii)

WORKSHEET 28

From the graph, it is clear that the lines are


parallel. Hence, the given system of
equations is inconsistent.
x
9. Let the fraction be
y
On adding 1 to each of numerator and
6
denominator, the fraction becomes
5
x
+
1
6
.
..
=
y +1 5

5x + 5 = 6y + 6

5x 6y = 1
...(i)
Further, on subtracting 5 from each of
numerator and denominator, the fraction
becomes
...

3
2

x5 3
=
y5 2

2x 10 = 3y 15
2x 3y = 5
...(ii)
Solving equations (i) and (ii), we get

42

1. (C) The condition that the given system of


equations has unique solution is represented by
a1
b1

.
a2
b2
2. (A) Multiplying first equation by 2 and the
other one by 3 and adding, we get
1
21.8x = 10.9 x =
2
1
Substituting x =
in any of the given
2
1
equations, we have y = .
3
1
1

x= , y= .
3
2
3. k = 6
Hint:

k3
3
k
.
= =
k
k
12

4. The condition that the given system of


equations represents parallel lines is

p2
p2 + 1
5
=

2
3
3p + 1

5p = 5 p = 1.

5. True.
The condition for parallel lines is
2 2 3
=

6
6
5
1
1
3

=
3
3
5
The condition holds. The lines are parallel.

M A T H E M A T

I C S X

6. x = a2 , y = b2
1
1
Hint: Put
= u and = v.
y
x
7. Given system of linear equations can be
written as:
(a b) x + (a + b) y (a2 2ab b2) = 0
(a + b) x + (a + b) y (a2 + b2) = 0
By cross - multiplication,
x
( a + b)( a2 + b 2 ) + ( a + b) ( a 2 2 ab b 2 )

y
( a b)( a + b ) + ( a + b) ( a2 2ab b2 )

1
( a b) ( a + b) ( a + b ) ( a + b)

y
x
1
=
=
2b ( a + b)
2b ( a + b)2
4ab 2

Hence, the solution of given system of


equations is

2ab
.
a+b
8. To draw graph of the equation, we need
atleast two solutions.
Two solutions of the equation
4x + 3y 24 = 0 are mentioned in the
following table:
x = a + b, y =

Observing the graph, we get the lines meet


each other pairwise in A(3, 4), B( 1, 4)
and C(9, 4).
Hence, the vertices of the triangle ABC so
obtained are A(3, 4), B(1, 4) and
C(9, 4).
1
Area of ABC = base height
2
=
9. ` 600, ` 700
Hint: Let cost price of trouser be ` x and
that of shirt ` y. Then

Similarly, two solutions of each of the


equations 2x y 2 = 0 and y + 4 = 0 are
respectively
x
y

125
110

x+
y = 1520

100
100

110
125

x+
y = 1535

100
100

and
x
y

O F

N E A R

25 x + 22 y = 30400

22 x + 25 y = 30700

OR
6 l of 50% and 4 l of 25%.

Using the tables obtained above, let us


draw the graph.
P A

1
10 8 = 40 sq. units.
2

Hint: Let x litres of 50% acid and y litres of


25% acid should be mixed.

E Q U A T

I O N S

...

43

50
25
40

x+
y=
( x + y)

100
100
100

x + y = 10

WORKSHEET 29
1. (C) x = 9, y = 6
Hint: x y = 3 and 2x + 3y = 36.
2. (A) Solving 3x 2y = 4 and 2x + y = 5, we
have x = 2, y = 1.
Now, substituting these values of x and y
in y = 2x + m, we have 1 = 2 2 + m
m = 3.
3p

3.

18
p

24

50
75

2
3

p = 3.

Hint: a2x b2y = ab(a + b), ax by = 2ab


Solving the equations, we get x = b, y = a.
26b
22a
6. x =
,y=
5
5

4bx + 3ay 2ab = 0


3bx + ay 8ab = 0.

7. 3x + 2y = 800,
12x + 8y = 3000;
Not possible
Hint: Let cost of 1 chair be ` x and that of
1 table be ` y.
3x + 2y = 800, 12x + 8y = 3000.

44

0.95x + 1.15y (x + y) = 7

0.05x + 0.15y = 7

...(i)

Case II. Selling price Cost price = Profit

1.05x + 1.10y (x + y) = 13

0.05x + 0.10y = 13

...(ii)

Solving equations (i) and (ii), we get


x = 100 , y = 80
Hence, actual prices of tea-set and lemonset are ` 100 and ` 80 respectively.
OR
The person invested ` 500 at the rate of
12% per year and ` 700 at the rate of 10%
per year.
Hint: Let the person invested ` x at the rate
of 12% per year and ` y at the rate of 10 %
per year

= 36 and 3
3

= 6 and 0 or 6 = 6.
5. x = b, y = a

Hint:

4. For inconsistency,
3

3
=

12

According to the question,


Case I. Selling price Cost price = Profit

2x = 3y

x + y = 10

8. Let the actual prices of tea-set and lemonset be ` x and ` y respectively

12x 10 y
= 130
+
100 100
6x + 5y = 6500

...(i)

12y 10x
+
= 134
100 100

5x + 6y = 6700
...(ii)
Adding and subtracting (i) and (ii), we get
x + y = 1200
...(iii)
x y = 200
...(iv)
and

9. Two solutions of 6y = 5x + 10 are:


x

Two solutions of y = 5x 15 are


x

Now, we draw the graph of the system on


the same coordinate axes.
M A T H E M A T

I C S X

3. For coincident lines


b
a1
c
= 1 = 1
c2
a2
b2
2

a b

7
3
=
4a b
a b3

a 5b = 0.

4. False, because the given pair of equations


has infinitely many solutions at k = 40 and
no solutions at k 40.
5. Given equations are
2y x. (x + y) = 1

(i) From the graph, we look that the two


lines intersect each other at A(4, 5).
(ii) The vertices of the triangle: A(4, 5);
B(2, 0); C(3, 0).
Height of ABC corresponding to the
base BC,
h = 5 units
and base,
b = BC = 5 units
1
Now, ar(ABC) =
bh
2
1
= 55
2
= 12.5 square units.

ASSESSMENT SHEET 5
1. (C) Let us check option (C).
2
5
5
2
x + y = ( 3) + ( 2) = 2 5 = 7
3
2
3
2
1
1
3x y = 3 ( 3) ( 2) = 9 + 1 = 8.
2
2

4x y = 42

2.

x y=2
x 2y = 8
+

y=6

...(i)
...(ii)

(Subtracting)
From (i) x = 4 x + y = 10.
P A

O F

N E A R

x + y=

1
2y x

...(i)

and (x + y)x y = 2
...(ii)
Substituting the value of x + y from equation
(i) in equation (ii), we get
1
y x
2

xy

=2

(2x y)x y = 21

(x y)2 = 1

xy=1

xy=1

...(iii)

or

x y = 1

...(iv)

Substituting x y = 1 and x y = 1 in
equation (ii), we get respectively
x+y=2
...(v)
1
and
x+y=
...(vi)
2
Solving equations (iii) and (v), we have
x=

1
3
;y= .
2
2

3
4
Solving equations (iv) and (vi), we have
Therefore, xy =

x=
Therefore, xy =
Hence,

E Q U A T

xy =

3
1
;y=
4
4
3
.
16

3
3
or
.
16
4

I O N S

...

45

6. Given equations can be written as


x
y
+
(a + b) = 0
a
b
x
y
+ 2 2= 0
2
a
b

Let us apply cross-multiplication method


to solve these equations.
y
1
x
=
=
2 1
b
1
1
2
a
1
2

+ 2 +
a a a
b b
b
ab 2 ba 2

a2 b 2
a2 y
b2 x
=
=
ab
b+a
a+b

Taking

a b
b2 x
=
b+a
a b

and

a2 b 2
a2 y
=
ab
a + b

2 2

x=

a2 b 2 ( a b)
2

b ( a b)

From the graph it is clear that the two lines


intersect each other at a point, P( 1, 1),
therefore, the pair of equations consistent.
The solution is x = 1, y = 1.

and y =

( a b) a2 b 2
2

a ( a b)

x = a2 and y = b2.

7. Given equations of lines are:


3x + y + 4 = 0

...(i)

and
6x 2y + 4 = 0
...(ii)
To draw the graphs of lines (i) and (ii), we
need atleast two solutions of each equation.
For equation (i), two solutions are:
x

For equation (ii), two solutions are:


x

8. Let the cost price of the saree and the list


price of the sweater be ` x and ` y
respectively. Now two cases arise.
Case I.
8
Sale price of the saree = x + x
100
108
=
x
100
10
Sale price of the sweater = y y
100
90
=
y
100
90
108

x+
y = 1008
100
100

108x + 90y = 100800

Case II.
Sale price of the saree = x + x

2
8
Let us draw the graphs of the lines (i)
and (ii).

46

...(i)

110 x
100

Sale price of the sweater = y y


M A T H E M A T

10
100

8
100

I C S X

Here,

92
y
100

a2 = 2p + 2q, b2 = p + 8q,
c2 = 2q + p 1

92
110
x+
y = 1028
100
100

110x + 92y = 102800 ...(ii)


Adding equations (i) and (ii), we get
218x + 182y = 203600 ...(iii)
Subtracting equation (i) from (ii), we get
2x + 2y = 2000
or
218x + 218y = 218000 ...(iv)
(Multiplying by 109)
Solving equations (iii) and (iv), we have
x = 600 and y = 400
Hence, the cost price of the saree is
` 600 and the list price of the sweater is
` 400.

ASSESSMENT SHEET 6
k
k
=
6
k+4
k
k = 2.
6

k
7
3
=

4k + 1
2k
12
7
3
=
k = 14.
2k
12

3. x = a and y = k must satisfy both the given


equations. Let us substitute these values of
x and y in
bx ay + 2ab = 0
b(a) ak + 2ab = 0

ak + ab = 0 k = b.
4. Yes, because
a1
b1
c1
1 2 3
a2 = b2 = c2 , i.e., 3 = 6 = 9 .
5. Given equations are:
4x + 5y = 2
(2p + 2q)x + (p + 8q)y = 2q p + 1
P A

O F

N E A R

For infinitely many solutions,


c
b
a1
= 1 = 1
c2
b2
a2

2
5
4
=
=
2q + p 1
p + 8q
2p + 2q

Taking

5
4
=
p + 8q
2p + 2q

10p + 10q = 4p + 32q

6p = 22q 3p = 11q ...(i)

Also, taking

2
4
=
2q + p 1
2p + 2q

1. (B) For infinitely many solutions,


13
=
39
13
Taking
=
39
2. For no solution,

a1 = 4, b1 = 5, c1 = 2,

4p 4q = 8q + 4p 4
q = 2p 1

Substitute q = 2p 1 in equation 3p = 11q to


get
11
p=
.
19
11
3
Hence, p =
,q=
.
19
19
6. Given system of equations is
43x + 67y = 24
...(i)
67x + 43y = 24
...(ii)
Adding (i) and (ii); and subtracting (i) from
(ii), we get respectively
110x + 110y = 0
and
24x 24y = 48
i.e.
x + y= 0
...(iii)
and
x y= 2
...(iv)
Adding equations (iii) and (iv); and
subtracting equation (iv) from (iii), we get
respectively
x = 1 and y = 1.
7. One of the given equations is
2x + y = 4
...(i)
Here, at
x = 0, y = 4
and at
x = 2, y = 0

E Q U A T

I O N S

...

47

Two solutions of equation(i) are given in


the following table:
x

Another given equation is


2x y = 4
...(ii)
Two solutions of equation (ii) are given by
the following table:
x

Let us draw the graph of the two equations


(i) and (ii) by using their corresponding
tables.

1
(4 + 4) 2
2

= 8 square units.
Thus, area of the triangle is 8 square units.
8. Let Aman had a total of x oranges; and he
made lot A of p oranges and lot B of
remaining x p oranges. There are two
cases now.
Case I.
2
Selling price of lot A = ` p
3
Selling price of lot B = ` (x p)
2
p + x p = 400
3
3x p = 1200

...(i)
Case II.
Selling price of lot A = ` p
4
Selling price of lot B = `
(x p)
5
4

p + (x p) = 460
5
4x + p = 2300
...(ii)
Add equations (i) and (ii) to get
7x = 3500

x = 500
Hence Aman had a total number of 500
oranges.

CHAPTER TEST
1. (C) A pair of linear equations is said to be
consistent, if the lines either intersect each
other at a point or coincide.
2. (C) 6, 36
From the graph, vertices of the triangle ABC
are A(0, 4), B(0, 4) and C(2, 0).
1
ar(ABC) = base height
2
=

48

1
AB OC
2

Hint: Let the son's age = x,


And
father's age = y

y = 6x
and
y + 4 = 4(x + 4)
Solve yourself.
3. The lines are coincident

1
8
3
=
=
k = 2.
k
6
16

M A T H E M A T

I C S X

4. Yes.
For consistency,
either

a
2a
b
2a
b

or
=
=
2a
4a
4a
2b
2b

Here only the relation

a
b
2a
=
=
,
2a
4a 2b

1
1
1
=
= holds.
2
2
2
The pair is consistent.

3 + 3y = 5y
3

y=
2
3
1
Thus,
x=
and y =
.
2
2
7. The given system of equations will have
infinite number of solutions if

i.e.,

21x + 47y = 110


47x + 21y = 162

5.

and

68x + 68y = 272 (Adding)


x + y= 4

...(i)

Subtracting the given equations from one


another, we get
26x + 26y = 52

x y= 2
Solve equations (i) and (ii) to get
x = 3, y = 1.
6. We are given
3
2xy
=
2
x+y
xy
3
and
=
2x y
10

...(ii)

...(i)
...(ii)

2xy
3
=
x+y
2

3x + 3y = 4xy
Now, taking equation (ii),

...(iii)

xy
3
=
2x y
10

6x + 3y = 10xy

...(iv)

Multiplying equation (iii) by 2 and adding


its result to (iv), we get
9y = 18xy
1

x=
2
1
Putting x =
in equation (iv), we get
2
P A

O F

N E A R

1
2
=
a+b2
a+b
1
a+b2
1
a+b2
ab

and 2a + 2b 4 = a + b

a + b a + b = 2 and a + b = 4
b = 1 and a = 3

Hence, the given system of equations will


have infinite number of solutions, if
a = 3, b = 1.

Taking equation (i),

1
=
ab
1
=
ab
2
=
a+b
a + b 2=

8. (i) Let fixed charge = ` x


and charges for a distance of 1 km = ` y
Now, According to question,
x + 12y = 89
...(i)
x + 20y = 145
...(ii)
We will solve equations (i) and (ii) by
elimination method.
Subtract equation (ii) from equation (i):
56
=7
8y = 56 y =
8
Putting value of y in equation (i), we get
x + 12(7) = 89
x = 89 84 = 5

x = 5; y = 7
For a journey of 30 km charge paid = x +
30y = 5 + 30(7) = 5 + 210 = ` 215.
(ii) Solution of pair of linear equations in two
variables.
(iii) Love towards environment.
9. To draw the graph of a line, we are required
atleast two solutions of its corresponding
equation.
At x = 0, 5x y = 5 gives y = 5

E Q U A T

I O N S

...

49

At x = 1, 5x y = 5 gives y = 0
Thus, two solutions of 5x y = 5 are given
in the following table:
x
y

Similarly, we can find the solution of each


remaining equation as given in the
following tables:
x + 2y = 1:
x

6x + y = 17:
x

Now, we will draw the graphs of the three


lines on the same coordinate axes.
From the graph, it is clear that the lines
form a triangle ABC with vertices A(1, 0),
B(3, 1) and C(2, 5)

qq

50

M A T H E M A T

I C S X

Chapter

TRIANGLES
WORKSHEET 33

1. (D) Observing the figure, we obtain


A = R, B = Q, C = P
ABC ~ RQP.

and

QT
QR
=
PQ
QS

[Using (i)]

1 = 1

(Common)

PQS ~ TQR

(SAS criterion)
Hence proved.

2. Q ABC ~ DEF
ar (ABC)
BC2
AB2
AC 2
=
=
2
2 =
2
ar (DEF)
EF
DF
DE

Taking

ar (ABC)
BC2
=
2
ar (DEF)
EF

64
BC2
=
2
121
(15.4)

BC =

BC = 11.2 cm.

6. 1 : 4
Hint: ar(DEF) =

1
ar(ABC)
4

ar( DEF)
1
= .
4
ar( ABC)
B

E
7. 13 m
Hint: Distance between tops = AD

64 15.4 15.4
121

AD =

AE2 + DE2

(5)2 (12)2

F
C

= 13 m.

3. Yes.
Here, 262 = 242 + 102 = 676

AC2 = AB2 + BC2


ABC is a right triangle.

8. Hint: Use Pythagoras Theorem.

4. In ABC, LM || CB
AM = AL
(i)
AB
AC
(Basic Proportionality Theorem)
Similarly in ADC,

9. Statement: If a line is drawn parallel to


one side of a triangle to intersect the other
two sides at distinct points, the other two
sides are divided in the same ratio.
Proof: ABC is a given triangle in which
DE || BC. DE intersects AB and AC at D
and E respectively.

(ii)
AN = AL
AD
AC
Comparing equations (i) and (ii), we have

AM
AN
=
.
AB
AD

Hence proved.
We have to prove

As 1 = 2

5.

PQ = PR
QT
QR
=
PR
QS

T R

A N G L E S

(i)
(Given)

AD
AE
=
CE
BD
Let us draw EM AB and DN AC. Join
BE and CD.

51

Now,

Also,

ar(ADE) =

1
base height
2

1
AD EM
2

...(i)

ar(ADE) =

1
AE DN
2

...(ii)

ar(BDE) =

1
BD EM ...(iii)
2

1
CE DN ...(iv)
2
Dividing equation (i) by equation (iii) and
equation (ii) by equation (iv), we have
ar (ADE)
AD
=
BD
ar(BDE)

...(v)

AE
ar(ADE)
=
CE
ar(CDE)

...(vi)

But
ar(BDE) = ar(CDE)
...(vii)
(Triangles are on the same base DE and
between the same parallels BC and DE)
Comparing equations (v), (vi) and (vii), we
have
AE
AD
=
.
CE
BD

2nd Part:
As
B

AB
AD + DB

AD

=
=
=
=

C
AC
AE + EC
AE

(... BD = EC)

AE
AD
=
EC
DB
By converse of Basic Proportionality
Theorem, DE || BC.
Hence proved.

= BC2 +
=9+

ar(CDE) =

and

AB
DC2 = BC2 + BD2 = BC2 +

2.

52

BC
AC
=
AC
DC

y2 = 16 4 y = 8 cm.

1
(AC2 BC2)
4

1
(25 9) = 9 + 4 = 13
4

13 cm.

3. Hint: As DE || BC
AE
AD
=
EC
DB

2x 1
2x + 5
=
.
x 3
x 1

4. DE || BC and DB is transversal

EDA = ABC
(Alternate interior angles)

Similarly, AED = ACB


Consequently, ADE ~ ACB
(AA similarity)

AD 2 ar (ADE)
=
ar (ABC)
AB 2

ar (ADE)
AD 2
=
2
153
9 AD

ar(ADE) = 17cm2.

5. No.
Here,

1
DP
5
=
=
PE
10
2

And

6
1
DQ
=
=
QF
18
3

DQ
DP

QF
PE

WORKSHEET 34
1. (A) BAC ~ ADC

DC =

Therefore, PQ is not parallel to EF.


6. Hint: Use Basic Proportionality Theorem.
M A T H E M A T

I C S X

Let us construct a PQR with Q = 90


such that
PQ =AB and QR = BC ...(ii)

7. As AB = BC = AC

1
BC
2
Using Pythagoras Theorem
AD BC BD =

AB2 = AD2 + BD2


1

AD2 = AB2 BC
2

3AB 2
4
4AD2 = 3AB2.

Hence proved

OR
Let ABCD be a rhombus
Since, diagonals of a rhombus bisect each
other at right angles,

AO = CO, BO = DO,
AOD = DOC
= COB = BOA = 90
Now, in AOD
AD2 = AO2 + OD2
...(i)
...(ii)
Similarly, DC2 = DO2 + OC2
CB2 = CO2 + BO2
...(iii)
...(iv)
and
BA2 = BO2 + AO2
Adding equations (i), (ii), (iii) and (iv),
we have
AD2 + DC2 + CB2 + BA2
= 2(DO2 + CO2 + BO2 + AO2)

BD 2 AC2 BD 2 CA 2
+
+
+
= 2

4
4
4
4
= BD2 + CA2.
Hence proved

1
BC
3
Use Pythagoras Theorem.

8. Hint: BD = DE = EC =

9. Statement: In a triangle, if square of the


largest side is equal to the sum of the
squares of the other two sides, then the
angle opposite to the largest side is a right
angle.
Proof: We are given a triangle ABC with
AC2 = AB2 + BC2
...(i)
We have to prove that B = 90
T R

A N G L E S

In PQR,
PR2 = PQ2 + QR2
(Pythagoras Theorem)
= AB2 + BC2
...(iii)
[From (ii)]
2
2
2
...(iv)
But
AC = AB + BC
[From (i)]
From equations (iii) and (iv), we have
PR2 = AC2

PR = AC
...(v)
Now, in ABC and PQR,
AB = PQ
[From (ii)]
BC = QR
[From (ii)]
AC = PR
[From (v)]
Therefore, ABC PQR
(SSS congruence rule)

B = Q
(CPCT)
But
Q = 90
.
..
B = 90.
Hence proved.
2nd Part
In ADC, D = 90
...
AC2 = AD2 + DC2 = 62 + 82
= 36 + 64 = 100
In ABC,
AB2 + AC2 = 242 + 100 = 676
and
BC2 = 262 = 676
Clearly, BC2 = AB2 + AC2
Hence, by converse of Pythagoras Theorem,
in ABC,
BAC = 90
ABC is a right triangle.

53

WORKSHEET 35

7. In ABC and AMP,


A = A
ABC = AMP = 90

1. (B)

ar ( ADE)
DE
=
ar ( ABC)
BC2

2 BC

ar(ADE) =
BC2

(i)

ABC ~ AMP,

(ii)

BC
CA
=
.
MP
PA
(... Corresponding sides of
similar triangles are proportional.)

81 = 36 cm2.

2. OAB ~ OCD

OA
OB
=
OC
OD

OB = 4

3
= 6 cm.
2

3x + 4
3 x + 19
AD
BE
=

=
x
x+3
DC
EC

3x2 + 19x = 3x2 + 4x + 9x + 12

6x = 12 x = 2.
4. No,
FED ~ STU
'
Corresponding sides of the similar triangles
are in equal ratio.

EF
DE
=
ST
TU

EF
DE

.
TU
ST
AP BQ
=
AO BO

(i)

AC || PR

AP CR
=
AO CO

(ii)

BQ
CR
From (i) and (ii) ,
=
BO
CO
BC || QR.
(By converse of BPT)

54

...

AC =

...

ar( ABE)
AB2
=
.
ar( ACD)
AC2

2a

As

2
ar (ABC)
=
ar ( AXY)
1

ABC ~ AXY

2
2
ar ( ABC)
AB

= ar ( AXY) =
1
AX

2 1
.
2

AB
2
BX
=

=
1
AX
AB

9. Hint: Prove converse of Pythagoras


Theorem.

WORKSHEET 36
1. (A)
In triangle ABC,

5. AB || PQ

6. 1 : 2.
Hint: Let AB = BC = a

(AA criterion)

8. Hint:
ar( AXY) = ar(BXYC)
2 .ar(AXY) = ar(BXYC) + ar( AXY)
A
= ar(ABC)

3. In ABC, to make DE || AB, we have to


take

(Common)

AD
6
2
=
=
DC
21 7

2
BE 18 14
=
=
7
EC
14

AD BE
=
DE || AB .
DC EC
2. (B) ABD ~ BCD

BD
AB
=
CD
BC

3.6
5.4
=
5.2
BC
5.4 5.2
BC =
3.6
= 7.8 cm.

M A T H E M A T

I C S X

EF2
ar (DEF)
=
ar (ABC)
BC2

3.

ar(DEF) = 54

16
= 96 cm2.
9

4. In ABC and ADE,


BAC = DAE
ACB = AED

AB =
Now,

AC 2 + BC 2 =

10 9
10
AQ
=
AQ =
6
6
9

AQ = 15 cm.
OR
Let the height of the tower be h metres

(Common angle)
(Each 90)

ABC ~ ADE

AO
AQ
=
BO
BP

(AA criterion)

25 + 144 = 13 cm

AC
BC
AB
=
=
AE
DE
AD

5
12
13
=
=
AE
3
DE

15
36
DE =
cm and AE =
cm.
13
13

ABC ~ PQR.

5. No.
Ratio of areas of two similar triangles
= Square of ratio of their
corresponding altitudes

8
BC
AB
12
=

=
40
QR
h
PQ
h=

12 40
= 60 metres.
8

8. Hint: As AOB ~ COD

9
6
3
= =
.
25
5
5

Hence, it is not correct to say that ratio of


6
areas of the triangles is .
5
6.

AE2 = AC2 + EC2

(i)

BD2 = DC2 + BC2

(ii)

Adding (i) and (ii), we get


AE2 + BD2 = AC2 + EC2 + DC2 + BC2
= (AC2 + BC2) + (EC2 + DC2)
AE2 + BD2 = AB2 + DE2.
Hence proved.
7. In AQO and BPO,
QAO = PBO
(Each 90)
AOQ = BOP
(Vertical opposite angles)
So, by AA rule of similarity,
AQO ~ BPO
T R

A N G L E S

2 CD

AB2
ar(AOB)
=
=
2
ar(COD)
CD
CD 2

4
.
1

9. Hint: Prove Pythagoras Theorem.


For 2nd Part:

AB2 = AD2 + BD2


(i)
Also
AC2 = AD2 + CD2
(ii)
From (i) and (ii),
AB2 AC2 = BD2 CD2
AB2 + CD2 = AC2 + BD2.
Hence proved.

WORKSHEET 37
1. (A)

M = 180 (L + N) (ASP)
= 180 (50 + 60) = 70

LMN ~ PQR
M = Q Q = 70.

55

2. (C) In KMN, as PQ || MN,


KP
KQ
=
PM
QN

KQ
KP
=
PM
KN KQ

KN
PM
1 =
KQ
KP

13
20.4
1 =
4
KQ

In right AOE,
AO2 = AE2 + OE2
From equations (i) and (ii), we have

13 17
20.4
=1+
=
4
KQ
4
20. 4 4
KQ =
17
KQ = 4.8 cm.

3. ABC ~ DEF.

AF2 + OF2 = AE2 + OE2


Similarly, we can find out that

...(iii)

BD2 + OD2 = BF2 + OF2

...(iv)

CE2

...(v)

and

2
QR 2
ar ( PRQ)
3 = 9 = 9 : 1.

=
=

ar ( BCA)
BC 2
1
1

5. True
Hint: Use Basic Proportionality Theorem
6. Hint:
Use: 1 = 2
3 = 4.

OB2 =
OD2 =
2
OB + OD2 =
=

D
O

OD2

Hence the result.


8. ABC ~ PQR

BC
AB
=
and B = Q
QR
PQ

1
BC
AB
2
=
and B = Q
1
PQ
QR
2

AB
BP
=
and B = Q
PQ QM

EO2 + EB2
OF2 + DF2
EO2 + EB2 + OF2 + DF2
EO2 + CF2 + OF2 + AE2
[' DF = AE, EB = CF]
= (EO2 + AE2) + (CF2 + OF2)
OB2 + OD2 = OA2 + OC2.
OR
Join OA, OB and OC
In right AOF,
AO2 = AF2 + OF2
...(i)

56

CD2

Adding equations (iii), (iv) and (v), we


arrive

7. Draw EOF || AD

OE2

AF2 + BD2 + CE2 = AE2 + BF2 + CD2.

4. Q ABC ~ PQR

...(ii)

(Q BD = DC and QM = MR)

ABD ~ PQM

AD
AB
=
.
PM
PQ

Hence proved.

9. Let the two given triangles be ABC and


PQR such that ABC ~ PQR

BC
AB
=
QR
PQ

M A T H E M A T

...(i)
I C S X

Further, in the question,


2
ar( %ABC)
BC
=

ar( %DEF)
EF

BC 2
64
=
15.4 15.4
121

Let us draw perpendiculars AD and PM


from A and P to BC and QR respectively.

ADB = PMQ = 90
...(ii)
Now, in ABD and PQM,
B = Q (QABC~PQR)
ADB = PMQ
[From (ii)]
So, by AA rule of similarity, we have
ABD ~ PQM
AD
AB
=
...(iii)

PM
PQ
From equations (i) and (iii), we get
BC
AD
=
QR
PM

...(iv)

1
BC AD
ar (ABC)
= 2
Now,
1
ar (PQR)
QR PM
2
1
BC BC
2
=
1
QR QR
2
[Using (iv)]
2

BC
=

QR
Similarly, we can prove that

ar (ABC)
AB
=

ar (PQR)
PQ

...(v)

BC =
=

1. (B) Ratio of areas of two similar triangles


= Ratio of squares of their
corresponding sides.
= 42 : 92 = 16 : 81.
2. (C) M = Q = 35
(Corresponding angles)
QR
PQ
=
MN
ML
(Ratio of corresponding sides)

MN = 5

AB
AD
=
AC
AE

AB = 21

5
2
AP
=
=
7.5
AQ
3

A N G L E S

2
4
BP
=
=
3
6
BR

AC
=
.
PR
Hence, the theorem.

5
= 15 cm.
7

4. Yes.

T R

12
= 10 cm.
6

3. In ABC, DE || BC

AC
ar( %ABC)
and
=
...(vii)

ar( %PQR)
PR
From equations (v), (vi) and (vii), we obtain
AB
BC
ar( %ABC)
=
=

ar( %PQR)
PQ
QR

8
15.4 = 11.2 cm.
11

WORKSHEET 38

...(vi)

64 15.4 15.4
121

Here,

AP
BP
=
AQ
BR

Hence, due to the converse of Basic


Proportionality Theorem, AB || QR.

57

5. Q DB BC and AC BC
DB || AC
Now, DBA = BAC
And, DEB = ACB

BDE ~ ABC

6.

(Alternate angles)
(Each 90)
(AA similarity)

DE
BE
=
(Corresponding sides)
BC
AC
AC
BE
=
.
Hence proved.
BC
DE

But as

AB = AC = BC
1
DE = BC.
2
(iii) Concept of similarity of two triangles and
mid-point theorem.
(iv) His ability to think rationally and taking
unbiased decision.

WORKSHEET 39

AX 2  2
=
2
AB
Hint: See Worksheet 35, Sol. 8.

7. Hint: AM =

1
AC
2
(Using mid-point theorem)

(ii) Yes, as DE P AC and DE =

1. (A) Let the length of shadow is x metres.


BE = 1.2 4 = 4.8 m
ABC ~ DEC

1
AB;
2

AB
BC
=
DE
EC

1
AC
2
Use Pythagoras Theorem.
AL =

3.6x = 4.32 + 0.9x.

8. Hint: Join AC and use Basic Proportionality


Theorem.
A

As

x=

2. (B) Here, (a)2 +

4.32
= 1.6 m.
2.7

3a

ar ( %ADF)
(DF)2
1
=
=
ar ( %ABC)
4
(BC)2

= a2 + 3a2

...(i)

According to the converse of Pythagoras


Theorem, the angle opposite to longest side
is of measure 90.

AD
2
AB AD
3
=

=
3
2
DB
AD

3.

ar(ADF) = ar(BDE) = ar(CFE) =


ar(DEF)

ar ( %DEF)
1
(i)
=
ar ( %ABC) 4

= 4a2 = (2a)2

9. As D and F are mid-points of AB and AC


respectively.
1
DF
1
DF P BC and DF = BC.
=
2
BC
2
Also, as ADF ~ ABC

4.8 + x
3.6
=
x
0.9

3
5
AB
AB
1=

=
2
2
AD
AD
DE || BC ABC ~ ADE

5
AB
BC
=
= .
2
DE
AD

D
B

58

4. No.
In PQD and RPD,

PDQ = PDR = 90
C

M A T H E M A T

I C S X

But neither PQD = RPD

PQD = PRD

nor

Therefore, PQD is not similar to RPD.


1

BC
BA
AC

=
=
DC
AC
AD

CA2

4
AL
=
5
DM
Ratio of corresponding heights is 4 : 5.

5. Hint: BAC ~ ADC

OR

= BC CD.

AD
5
5
AD
=

=
AB AD
4
DB
4

6.

5AB 5AD = 4AD


As DE || BC,

5
AD
=
AB
9

...(i)

...(ii)

[Using (i)]
Q DE || BC and DC is a transversal

EDC ~ BCD
(Alternate interior angles)
EDF = BCF

...(iii)

DEF = CBF

... (iv)

Similarly,
From equations (iii) and (iv), we have
DEF ~ CBF

(AA similarity)
2

DE
25
ar (DEF)
=
.
=
81
ar (CFB)
BC

[Using equation (ii)]


7.

In ADZ, XY || DZ

AY
AX
2
=
=
YZ
XD
3

... (i)

In YBC, BY || DZ
YZ
BD
1
(... BD = DC)

=
=
ZC
DC
1
2YZ = 2ZC
... (ii)
From (i) and (ii),
2ZC = 3AY
... (iii)
Now, AC = AY + YZ + ZC
8
3
3
= AY + AY + AY = AY
2
2
2
= 4AY
Therefore, AC : AY = 4 : 1. Hence proved.
8. 2 5 cm

1
1
BC; EB = AB
2
2
Use Pythagoras Theorem.

Hint: BD =

9. Statement: In a right triangle, the square


of the hypotenuse is equal to the sum of
the squares of the other two sides.
Proof: We are given a right triangle ABC
right angled at B.

Proof: Draw a ray DZ parallel to the ray


XY.

2YZ = 3AY

ADE ~ ABC
5
AD
DE

=
=
AB
9
BC

i.e.,

AL2
ar(ABC)
=
DM 2
ar( DEF)

AB = AC; DE = DF
AB
DE
=
=1
AC
DF

AB
AC
=
also A = D
DE
DF
ABC ~ DEF

T R

A N G L E S

We need to prove that AC2 = AB2 + BC2


Let us draw BD AC.
Now, ADB ~ ABC

59

AB
AD
=
AC
AB
or AD .AC = AB2

So,

(Sides are proportional)

N
P

...(i)

Also, BDC ~ ABC

Hint: W

CD
BC
So,
=
AC
BC
or CD.AC = BC2

8m
15 m

...(ii)

Adding equations (i) and (ii), we get


AD .AC + CD . AC = AB2 + BC2

3. 17 m

AC(AD + CD) = AB2 + BC2


AC.AC = AB2 + BC2

AC2 = AB2 + BC2.


Hence proved.

2nd Part:

Use Pythagoras Theorem and find OP.


4. Hint:
Let
AB =
AC =
BC =
.
..
a2 =

c
b
a
b 2 + c2

Also, ar(ABE) =

3 2
c
4

ar(BCF) =

3 2
a
4

AB2 = AD2 + BD2


= AD2 + (3CD)2
= AD2 + 9CD2

3 2
b .
4
5. See Worksheet - 36, Sol. 6.

= AD2 + CD2 + 8CD2

ar(ACD) =

= AC2 + 8CD2
= AC2 + 8

BC

'

2AB2 = 2AC2 + BC2.

CD =

BC
4

Hence proved.

WORKSHEET 40
1. (A)

AB
BC
AC
=
=
QR
PQ
PR
(Q ABC ~ PQR)

10
7
12
=
=
x
9
y

79
21
x=
=
12
4

and

y=

9 10 15
=
.
12
2

2. Required ratio =

60

16
4
= = 4 : 5.
25
5

6. Let ABCD be a quadrilateral of which


diagonals intersect each other at O.
It is given that
AO
BO
=
CO
DO
CO
AO
or
=
...(i)
DO
BO
In AOB and COD,
AOB = COD
(Vertically opposite angles)
CO
AO
=
DO
BO

[From (i)]

Hence, by SAS rule of similarity, we obtain


AOB ~ COD
BAO = DCO
i.e. BAC = DCA
These are alternate angles.
Therefore, AB || CD and AC is transversal
ABCD is a trapezium. Hence proved
M A T H E M A T

I C S X

OR
Hint:
As BAC = EFG ; ABC = FEG
and ACB = FGE

1
1
ACB = FGE
2
2

ACD = FGH
and
DCB = HGE

DCA ~ HGF
Similarly DCB ~ HGE.

7. Hint:

Perimeter of DEF
4
=
3 + 2 + 2.5
2
[Taking (ii) and (iv)]

Perimeter of DEF = 15 cm.


2. DE || BC

E
B

DE + EF + DF
AB + BC + CA
(iv)

Prove that AEB ~ DEC.

8. See worksheet 33, Sol. 9 (1st part).


2nd Part
Join EF and join BD to intersect EF at O.

x+2
x
=
x2
x 1

x2 4 = x2 x
x = 4.

3. KNP ~ KML
c
ac
x

=
x=
.
b+c
b+c
a
4. Hint:
A

P
D

Q AB || DC, and EF || AB,


AB || DC || EF
In ABD, EO || AB,
DE DO
=
...(viii)
AE
BO
(Basic Proportionality Theorem)
Similarly, in BCD,

CF
DO
=
...(ix)
BO
BF
Using equations (viii) and (ix), we obtain
the required result, i.e.,
AE BF
=
.
ED FC

Prove that ADL ~ CPD.


5. Hint: 2AP = PC AP =
Similarly, BQ =

1
BC
3

Use Pythagoras Theorem.


6. PQ || BC

AQ
AP
1
=
=
QC
PB
2

APQ ~ ABC
2

ar (ABC)
AB

=
= (3 )2 = 9
AP
ar ( APQ)

'

WORKSHEET 41
DF
EF
DE
=
=
AC
BC
AB
(i) (ii)
(iii)

1. (B)

T R

A N G L E S

1
AC
3

AB
AP

ar (ABC)
1=8
ar ( APQ)

61

ar (%APQ)
8
ar (BPQC)
1
=
= .
1
ar (BPQC) 8
ar (%APQ)

Ratio of areas of APQ and trapezium


BPQC is 1 : 8.
OR
Let the given square be ABCD.
Let us draw an equilateral triangle APB
and another equilateral triangle AQC on
the side AB and on the diagonal AC
respectively.

We need to prove
ar(APB) =
In right ABC,
AC =

1
ar (AQC)
2
AB2 + BC2

= AB 2
Now,

ar(APB) =

3
AB2
4

And

ar(AQC) =

3
AC2
4

...(i)
(Q AB = BC)

3
AB 2
4

3
AB2
2

62

ar(APB) =

Extend AD till E such that AD = DE and


similarly, PM = MN
Prove that ACE ~ PRN
1 = 2
...(i)
But
3 = 5,
(CPCT)
3 = 4 and 4= 6
.. .
5 = 6
...(ii)
Adding (i) and (ii),
1 + 5 = 2 + 6

BAC = QPR
.. .
ABC ~ PQR.
(By SAS)
8. Let us take two similar triangles ABC and
PQR such that ABC ~ PQR.
AB
BC
CA
...
=
=
...(i)
PQ
QR
RP
We need to prove

...(ii)

ar (ABC)
AB 2
BC 2
CA 2
=
=
=
ar(PQR)
PQ 2
QR 2
RP 2

...(iii)

Dividing equation (ii) by equation (iii), we


obtain

ar (APB)
=
ar (AQC)

7. Hint:

3
AB 2
1
4
=
2
3
AB 2
2

1
ar(AQC).
2
Hence proved.

Let us draw AM BC and PN QR.


Q
ABC ~ PQR

B = Q
...(ii)
In ABM and PQN,
B = Q
[From (ii)]
and M = N
(Each 90)
ABM ~ PQN
(AA criterion)

AB
AM
=
PQ
PN

M A T H E M A T

...(iii)
I C S X

From equations (i) and (iii), we have


BC
AM
=
...(iv)
QR
PN
Now,

And

1
base height
2
1
BC AM
=
2
1
ar(PQR) =
QR PN
2

ar(ABC) =

BC AM
Therefore, ar ( ABC) =
QR PN
ar ( PQR)

BC 2
QR 2

...(v)

[Using (iv)]
From results (i) and (v), we arrive

BC2
AB 2
ar ( ABC)
CA 2
=
=
=
.
2
2
ar ( PQR)
PQ
QR
RP 2
Hence the result.
Further, consider the question in the
following figure.

2. (D) Let the given areas be 2x and 3x.


Required ratio = 2x : 3x = 2 : 3 .
3. x = 11 or 8
Hint: Use

ar (AOB) AB 2
=
ar (COD) CD 2

1
ar(COD) = 84
2
= 21 cm2.

5. In right ADC,
AD2 = AC2 CD2
= (2CD)2 CD2
[Q AC = BC = 2 CD]
= 3 CD2.
6. Hint: BMDN is a rectangle.
DBMD ~ DDMC

DM
DN
=
DM2 = DN MC
MC
DM
Also, DBND ~ DDNA.

CD 1
=
'
AB 2

OBC ~ ODA

1. (A)

T R

A N G L E S

(AA criterion)

BE
GE
=
BC DC

GE
3x
=
2AB
3 x + 4x

6
AB
7
Similarly, DDGF ~ DDBA

WORKSHEET 42

DN
DM
=
DN2 = DM AN.
AN
DN

7. Let BE = 3x and EC = 4x.


In DBCD, GE || DC

DBGE ~ DBDC

OD
OC
=
.
OB
OA

4. True.
Geometrical figures which are equiangular
i.e., if corresponding angles in two
geometrical figure are same, are similar.

ABO = CDO and BAO = DCO


(Alternate angles)

AOB ~ COD
(AA rule)

OC
BC
OB
=
=
=2
OD OA
DA
BC = 2DA = 2 4 = 8 cm.

GE =

( DC = 2 AB)
...(i)

4
FG
4
=
FG =
AB ...(ii)
7
AB
7
Adding equations (i) and (ii), we get
6
4
GE + FG = AB + AB
7
7

63

10
AB
7
7 EF = 10AB.
EF =

Hence proved.
A

Draw AE BC

BE = EC = a
2
1
1
and BD = BC = a B
C
3
3
D E
Using Pythagoras Theorem
AD2 = AE2 + DE2
= AE2 + (BE BD)2
2

AD = AE2 + BE2 + BD2 2.BE.BD


1
AD2 = AE2 + EC2 + a
3

= a2 +

1
s BC s AM AM AO
2
=
=
=
.
1
DN OD
s BC s DN
2
4. Hint: Use concept of similarity.
5. Draw AP BC

1 1
2 a a
2 3

= AC2 +

ar ( ABC)
ar ( DBC)

8. Hint: Let AB = BC = AC = a

3. Hint: Draw AM BC
and DN BC
As AOM ~ DON

a2
a2

9
3

7 a2
a2 a2
=

9
9
3

9AD2 = 7AB2.

AB2 = AP2 + BP2


= AP2 + (BD + DP)2
2

AB = AP2 + BD2 + DP2 +2BD. DP


= AD2 + BD (BD + 2DP)
AB2 AD2 = BD CD.
[... BP = PC]
Hence proved.
6. Hint:

9. See Worksheet 39, Sol. 9 (1st part).


Hint: 2nd Part:
AC2 = AD2 + DC2
= AD2 + (3BD)2
= AD2 + 9BD2
= AD2 + BD2 + 8BD2
1

BC

4
2
2
2
2AC = 2AB + BC .

= AB2 + 8

From figure, show x =

ab
.
a+b

7. In MDE and MCB,


MDE = MCB

MD = MC

(Alternate angles)
(M is mid-point of CD)

DME = CMB
(Vertically opposite angles)

WORKSHEET 43
1 . (C) 294 cm2
Hint: Prove that OBP ~ OAQ.
2. 6 cm
Hint: Use AA-similarity to prove
AOB ~ COD.

64

MDE MCB,
DE = CB

M A T H E M A T

(ASA criterion)
(CPCT)
I C S X

AE AD = BC

AE = 2BC
...(i) (Q BC = AD)
Now, in LAE and LCB,

LAE = LCB (Alternate angles)

ALE = CLB
(Vertically opposite angles)
LAE ~ LCB
(AA criterion)
AE
LE

=
(Corresponding sides)
BC
BL
2BC
EL
=
[Using equation (i)]

BC
BL

EL = 2BL.
OR

Hence proved.

Also in BCD,
CM
DE
=
MB
EB
From (i) and (ii),
CE
DE
=
.
AE
EB

...(ii)

WORKSHEET 44
1. (C) BE2 =

4
3 2
a a2 = BE2
3
4

AB2 + BC2 + CA2


= a2 + a2 + a2
= 3a2
4
= 3 BE2 = 4 BE2.
3

60
cm
2. (B)
13

Hint:

Hint: Use Pythagoras Theorem.


3. x = 4
Hint: Use Basic Proportionality Theorem.

As AD is median
so,

AB2 + AC2 = 2(AD2 + BD2)

BC

AB2 + AC2 = 2 AD 2
4

2(AB2 + AC2) = 4AD2 + BC2


Similarly,

4. Hint: In ACD and ABC,


A = A
ADC = ACB = 90

...(i)

...(ii)
2(AB2 + BC2) = 4BE2 + AC2
2
2
2
2
2(AC + BC ) = 4CF + AB
...(iii)
Add (i), (ii) and (iii),
3(AB2 + AC2 + BC2)
= 4(AD2 + BE2 + CF2).
8. See Worksheet 33, Sol. 9 (1st part).
2nd Part: Draw EM || AB
M is a point on CB

EM || AB

ACD ~ ABC
AC2 = AB . AD

(i)

BCD ~ BAC
BC2 = BA. BD

(ii)

Applying (ii) (i) gives the result.


5. Let the given parallelogram be ABCD
We need to prove that
AC2 + BD2 = AB2 + BC2 + CD2 + DA2
Let us draw perpendiculars DN on AB and
CM on AB produced as shown in figure.

In ABC,
CM
CE
=
MB
AE

T R

A N G L E S

...(i)

65

In BMC and AND,


BC = AD (Opposite sides of a ||gm)
BMC = AND
(Each 90)
CM = DN
(Distance between
same parallels)

BMC AND
(RHS criterion)

BM = AN
...(i) (CPCT)
In right triangle ACM,
AC2 = AM2 + CM2
= (AB + BM)2 + BC2 BM2
= AB2 + 2AB . BM + BM2
+ BC2 BM2
= AB2 + BC2 + 2AB . BM ...(ii)
In right triangle BDN,
BD2 = BN2 + DN2
= (AB AN)2 + (AD2 AN2)
= AB2 2AB . AN + AN2
+ AD2 AN2
2
2
2
BD = AB + DA 2AB . AN

BD2 = CD2 + DA2 2AB . BM ...(iii)


[Using (i) and AB = CD]
Adding equations (ii) and (iii), we have
AC2 + BD2 = AB2 + BC2 + CD2 + DA2.
Hence proved.
6. Hint: AP || QB || RC
Use Basic Proportionality Theorem.
PQ2 = PR2 + QR2

7. (i)

(26)2 = (2x)2 + {2(x + 7)}2

676 = 4x2 + 4(x2 + 49 + 14x)

x + 7x 60 = 0
2

x + 12x 5x 60 = 0
(x 5)(x + 12) = 0
x = 5 or x = 12 (reject it)
x =5
PR = 2 5 = 10 km
QR = 2(5 + 7) = 24 km

66

(iii) Yes: as it will save time and fuel. Ravi is


innovative in his thoughts, so his rationality
and social responsibility is reflected here.
8. Let us produce AD to J and PM to K so that
DJ = AD and MK = PM.
Join CJ and RK.

In ADB and JDC,


AD = JD, ADB = JDC, BD = CD

ADB JDC
(SAS criterion of congruence)

AB = JC
...(i) (CPCT)
Similarly, we can prove that
PQ = KR
...(ii)
According to the given conditions, we have
AB
AD
AC
=
=
PQ
PM
PR

JC
KR

AJ
= 2
PK
2

AC
PR

[Using (i) and (ii)]

8x + 56x 480 = 0

x(x + 12) 5(x + 12) = 0

(ii) Pythagoras theorem

After construction of the highway the


distance travelled = 26 km
Distance saved = 34 26 = 8 km.

676 = 4x2 + 4x2 + 196 + 56x

Before construction of the highway the


distance travelled = 10 + 24 = 34 km

AC
PR

(SSS criterion of
similarity)
JAC = KPR
(Corresponding
angles)
i.e., DAC = MPR
...(iii)
Similarly, we can prove that
DAB = MPQ
...(iv)
Adding equations (iii) and (iv), we obtain

JC
AJ
=
KR
PK
AJC ~ PKR

M A T H E M A T

I C S X

BAC = QPR
...(v)
Thus, in ABC and PQR, we have
AC
AB
=
(Given)
PQ
PR
and BAC = QPR
[From (v)]
Therefore, ABC ~ PQR.
(SAS criterion of similarity)
Hence proved.
OR
ABE, ACE and ADE are right angled
triangles right angle at E each.

AB2 = AE2 + BE2


...(i)
2
2
2
AC = AE + CE
...(ii)
and AD2 = AE2 + DE2
...(iii)
Adding equations (i) and (ii), we get
AB2 + AC2 = 2AE2 + BE2 + CE2
= 2AE2 + (BD DE)2 + (CD + DE)2
= 2AE2 + BD2 2BD DE + DE2
+ CD2 + 2CD DE + DE2
= 2AE2 + BD2 2BD DE + DE2
+ BD2 + 2BD DE + DE2
(... BD = CD)
2
2
= 2AE + 2DE + 2BD2
=

2(AE2

DE2)

BC
2

1
BC2
2

[Using (iii)]
Hence proved.

WORKSHEET 45
1. (D) ABC ~ PQR
20
10
=
h
50

25 49
100
52
= 2 h2 =
49
100
h

h=

57
25 49
h=
= 3.5 cm.
10
100

3. Altitude AM divides base


BC in two equal parts. That
is BM = MC = 7 cm.
Using Pythagoras Theorem
In right ABM,
AM =
=

(25 + 7) (25 7)

25 2 7 2 =

32 18 = 24 cm.

4. (i) We know that diagonal of a square


2 side

=
In square AEFG, AF =

2 AG

...(i)

In square ABCD, AC = 2 AD
...(ii)
Using equations (i) and (ii), we obtain
AF
AC
=
.
AG
AD

(D is a mid-point of BC)
= 2AD2 +

2. (A) The ratio of similar triangles is equal to


the ratio of squares of their corresponding
altitudes.

...(iii)

(ii)
GAF = DAC (Each 45)
GAF GAC = DAC GAC

CAF = DAG
...(iv)
From equations (iii) and (iv), we have
ACF ~ ADG.
(SAS criterion)
5 . Hint: Q 1 = 2
PQ = PR

QT
QR
=
.
PQ
QS

6. Hint: Draw AM BC and DN BC.


7. Hint: Fig.
A

T R

h=
I

50 20
= 100 m.
10

A N G L E S

c
p
a

67

8. Hint: For 1st part: Prove Pythagoras Theorem.


For 2nd part: AC2 AB2
= (AD2 + CD2) (AD2 + BD2)
9. Hint: Let the
Then

DC = AB = x

4
3
x and AP = x
5
5
QRC ~ PRA.

See Worksheet 44, Sol. 5.

ASSESSMENT SHEET 7
1. (C) In ABC, PQ || BC

AQ
AP
=
QC
BP

2.4
2
=
BP = 3.6 cm
BP
3
AB = AP + BP = 2.4 + 3.6 = 6 cm.

BC 2
ar(ABC)
=
ar(DEF)
EF 2

2.

QC =

OR

1
BC AM
ar(ABC)
2
Now, ar(DBC) = 1
BC DN
2

9
BC
=

EF
4

3
BC
= .
2
EF

3. Draw AM BC and DN BC
Q
AMO = DNO = 90
and
AOM = DON

AO
.
DO

[Using (i)]

ar %ABC
AO
Therefore ar %DBC  DO Hence proved.

4. True, because BCD ~ CAD

CD2 = BD . AD.
5. PQ || BC and AB is transversal

APQ = ABC
...(i)
(Corresponding angles)
In ABC and APQ,
BAC = PAQ
(Common)
ABC = APQ
[From (i)]
so, by AA criterion of similarity,
ABC ~ APQ

ar( ABC)
AB2
ar(APQ) =
AP2
Subtracting unity from both the sides, we
have

ar( ABC) ar( APQ)


AB2
=
1
ar( APQ)
AP2

2
ar(trapezium BPQC)
AB
=
1
ar(APQ)
AP
...(ii)
It is given that

AP
1
PB
=

=2
AP
PB
2

68

AMO ~ DNO
(AA similarity)
AM
AO
=
...(i)
DN
DO

PB
+ 1= 2 + 1
AP
PB + AP
AB
=3
=3
AP
AP
2

AB

...(iii)

=9
AP
From equations (ii) and (iii), we have
M A T H E M A T

I C S X

ar(APQ)
1
=
ar(trapezium BPQC)
8

or

ar(APQ) : ar(trapezium BPQC)

2.

= 1 : 8.
6. See Worksheet 36, Sol. 6.
7. See Worksheet 33, Sol. 9 (1st part).
8. We are given two triangles ABC and PQR
such that ABC ~ PQR.
Draw perpendiculars AD and PM on BC
and QR respectively.

AB
BC
CA
=

.
QR
PQ
PR

30
15
x
6
=
x=
,x=
cm.
8
5
4
6+2

3. In ABC, LM || AB.
Using Basic Proportionality Theorem, we
have
2x
AC
BC
2x + 3
=

=
x3
BM
AL
x2

2x2 4x = 2x2 + 3x 6x 9
x = 9 x = 9.

4. False, because QPR 90.


5. Let the given right angled triangle be ABC
with C = 90 such that AC = b, BC = a and
AB = c.
We need to prove

AD 2
ar(ABC)
=
ar(PQR)
PM 2
In ABD and PQM,
ADB = PMQ = 90
ABD = PQM (... ABC ~ PQR)

ABD ~ PQM
(AA criterion of similarity)
AB
AD

=
...(i)
PQ
PM
(Corresponding sides)
We know that the ratio of areas of two
similar triangles is equal to ratio of squares
of their corresponding sides
AB 2
ar(ABC)
=
ar(PQR)
PQ 2
From equations (i) and (ii), we have

ar(ABC) AD
=
.
ar(PQR)
PM 2

...(ii)

Using Pythagoras Theorem, we have


AB2 = AC2 + BC2

c2 = b2 + a2
...(i)
Area of equilateral triangle drawn on side
BC

3 2
a
...(ii)
4
Similarly, areas of equilateral triangles
drawn on side AC and side AB are
respectively
=

ASSESSMENT SHEET 8
PR
PQ
QR
=
=
BC
CA
AB

T R

A N G L E S

3 2
b
4

...(iii)

3 2
c
4

...(iv)

Hence proved.
and

1. (A) PQR ~ CAB,

Sum of areas of equilateral triangles drawn


on the sides BC and AC
3 2
3 2
=
a +
b
4
4
[Adding (ii) and (iii)]

69

3 2
(a + b2)
4

3 2
c
4

[Using (i)]

= Area of equilateral triangle


drawn on hypotenuse AB.
Hence proved.

Proof: We are given a ABC in which


AC2 = AB2 + BC2
We need to prove ABC = 90.

6.

Let us construct a PQR such that


PQR = 90
and
PQ = AB
...(ii)
QR = BC
...(iii)
Using Pythagoras Theorem in PQR, we
have
PR2 = PQ2 + QR2

PR2 = AB2 + BC2


...(iv)
[Using equations (ii) and (iii)]
From equations (i) and (iv), we have
AC = PR
...(v)
Now, in ABC and PQR,
AB = PQ
(From (ii))
BC = QR
[From (iii)]
AC = PR
[(From (v))]
(SSS congruence)
So,
ABC ! PQR

ABC = PQR
(CPCT)
But
PQR = 90
(By construction)

ABC = 90.
Hence proved

ABC ~ PQR

1
BC
BC
AB
=
= 2
1
QR
PQ
QR
2

AB
BD
=
PQ
QM

...(i)

(a) In ABD and PQM,

AB
BD
=
QM
PQ

[From (i)]

ABD = PQM (QABC ~ PQR)


So, by SAS criterion of similarity, we have
ABD ~ PQM

AD
AB
=
PM
PQ

(b) Q ABD ~ PQM,


[From part (a)]

ADB = PMQ
180 ADC = 180 PMR
[From figure]

ADC = PMR.
Hence proved.
7. Converse of Pythagoras Theorem: In a
triangle, if square of one side is equal to the
sum of the squares of the other two sides,
then the angle opposite the first side is a
right angle.

70

...(i)

8.

BD = BE
...(i) (Given)
In OBD, AF OB and BD OB

AF || BD

OAF ~ OBD

OA
AF
=
OB
BD

AF
OA
=
BE
OB

...(ii) [Using (i)]

In AFC and BEC,


FAC = EBC
(Each 90)
FCA = ECB
(Vertically opposite angles)
M A T H E M A T

I C S X

So by, AA criterion of similarity,


AFC ~ BEC
AC
AF

=
...(iii)
BC
BE
Comparing equations (ii) and (iii), we have

4. Yes.
MQ = PQ PM
= 15.2 5.7 = 9.5 cm
NR = PR PN

OA
AC
=
OB
BC

= 12.8 4.8 = 8 cm

OC OA
OA
=
OB OC
OB

OA OB OA OC
= OB OC OB OA
(OA + OB) OC
= 2OA OB
Dividing both sides by OA OB OC, we
get
2
1
1
+
=
.
OC
OB
OA

PM
5.7
=
= 0.6
9.5
MQ

and

PN
4.8
=
= 0.6
NR
8

Clearly,

PN
PM
=
NR
MQ

MN || QR.
AOB ~ COD

5.

(AAA criterion of
similarity)

Hence proved.

CHAPTER TEST
1. (B) BC =

Now,

5 2 +122 = 13 cm

AO
BO
=
DO
CO
(Corresponding sides)

7x 9
9x 8
=
2x 1
3x

21x2 27x = 18x2 16x 9x + 8


3x2 2x 8 = 0 (x 2) (3x + 4)

ABD ~ CBA

AD
AB

=
AC
BC

AD =

1
=
2

2. (D)

16
40
=
2
25
50

1 : 2 = 16 : 25.
AE
AD
3.
=
EC
DB
1
1.5

=
EC
3
3

EC =
= 2 cm.
1.5

T R

x = 2.

6.

5 12
60
=
cm.
13
13
P12
P22

x = 2 or x =

A N G L E S

4
3

(Negative value rejected)

ABE ACD

AB = AC and AE = AD (CPCT)

Consider AB = AC

AD + DB = AE + EC

DB = EC

Also

AD = AE

...(i) (... AE = AD)


...(ii)
(Proved above)

Dividing equation (ii) by equation (i), we


have
AE
AD
=
EC
DB

...(iii)

71

Hence, in ABC
AE
AD
=
EC
DB

DE || BC

(Converse of Basic

Proportionality Theorem)

ADE = ABC and AED = ACB

ADE ~ ABC.

7. Hint:
PAC ~ QBC
RCA ~ QBA

x
AC
=
z
BC
y
AC
=
.
z
AB

8. Hint:
Draw MN || AD, passing through O to
intersect AB at M and DC at N.

1 = 2

(Corresponding
angles)
(Common)
(AA-criterion)

A = A

ADE  ABC
(ii) As ADE  ABC

ar %ADE

AD

ar %ABC

AD

=
2
2
AB

AB

[' ar(ADE) = ar(DECB) 2 ar(ADE) =


ar(DECB) + ar(ADE) 2 ar(ADE) =
ar( %ADE )
1
ar(ABC) ar %ABC = ]

1
2

AD
1
AD

1 =
1
AB
2
AB

1 2

Use Pythagoras Theorem for AOM, BOM,


CON and DON.

9. (i) As DE & BC.

2
AB AD


AB

BD
=
AB

2 1
2

2
2

AD  AB
AB

= 

BD
AB

2 2
2

Hence proved.
(iii) Concept of similarity of two triangles.
(iv) Honesty and rationality to divide his
land equally between his two children.
qq

72

M A T H E M A T

I C S X

Chapter

INTRODUCTION TO TRIGONOMETRY
WORKSHEET 50

=1+

116
4
A = 29.

A=

1 sin2 = 1

cos2 =

7.

1
2
6

24 2
252

sin R
1  sin 2 R

sin 60
4

( 3)

cos2 45

3 2

2
1

4
1
4
8 83
+

=
3
3
2
6

13
.
6

1
sin
+
cos cos

3
2

Adding and subtracting, we get respectively


2x = 120 and 2y = 60
i.e., x = 60 and y = 30.
9. cosec A =
sin A =

.
cos A =

6. True,

1
2

sin (x + y) = sin 90 and cos (x y) = cos 30


x + y = 90 and x y = 30

5. cot 25 + tan 41.

LHS =

8. sin (x + y) = 1 and cos (x y) =

24
25
24
49
1
= 25 +
=
+
=
= 7.
7
7
7
7
7
25
25

4. tan =

cot 30

72
252
7
cos =
25

Now, tan + sec =

4
2

24
24
sin2 =
25
25

cos 59
=1+1=2
cos 59

Hence, the given equation is valid.

x
x
2. 2 sin
= 1 sin
=
2
2
x
x
Q
sin
= sin

=
2
2
6

x=
x = 60.
3
3. sin =

sin 10
+ cos 59 sec 59
sin 10

1. (B) sin 3A = cos (A 26)


sin 3A = sin {90 (A 26)}

3A = 90 A + 26

cos 80
+ cos 59 cosec 31
sin 10

I O N

T O

1
=
cosec A

sin A
1
=
cos A
3

cot A =

1
=3
tan A

T R

1
10

1 sin 2 A =

tan A =

cos(90 10)
+ cos 59 cosec (90 59)
sin 10

I N T R O D U C T

10

1
=
10

I G O N O M E T R Y

3
10

73

sec A =

10
.
3

1
=
cos A

10. Hint: RHS =

sin C =

sin 2 A
1 cos2 A
=
1 cos A
1 cos A

(1 cos A) (1 + cos A)
1 cos A

7.

cos 60 + sin 30  cot 30


tan 60 + sec 45  cosec 45

1
1
+ 3
1 3
2
2
=
s
3
3 2 2

3 3
.
3

OR
Hint: LHS =

sin (1 2 sin 2 )


.
cos (2cos2 1 )

1. (A) sin ( + 36) = cos


sin ( + 36) = sin (90 )

+ 36 = 90

2 = 54

= 27.

2. (C)
Hint: Divide numerator and denominator
by cos .

5
25
sec2 =
4
16

sec2 1 =

25
1
16

tan2 =

9
16

tan =

1
sin R (1 sin R)
5. sec + tan =


cos R cos R
cos R

6. sin A =

74

1 sin R
1  sin R
2

b a
b+a b  a

cot R tan (90  R)  sec (90  R) cosec R


sin R cos (90  R) + cos R sin (90  R)
cot R cot R  cosec R cosec R
sin R sin R + cos R cos R

cot 2 R  cosec 2 R
sin 2 R + cos 2 R

= cosec2 1 cosec2
(Q sin2 q + cos2 q = 1)
= 1.
9. Draw ABC with
AB = BC = AC = a (say)
Draw AD BC

3
.
4
4. Hint: A = 30, B = 90, C = 60 .

3
3

8. Given expression

WORKSHEET 51

3. sec =

7
24
and cos C =
.
25
25

a
b
2

a
1 2
b

24
7
, cos A =
,
25
25

b a
.
ba

b a
b2  a2

BAD = DAC = = 30

and

BD = DC = a/2
sin =

sin 30 =

1
BD a/2
=
=
2
AB
a
1
.
2

1
tan R
tan R
10. LHS =

1
1  tan R
1
tan R
=

tan 2 R
1

tan R  1
tan R 1  tan R

tan 2 R 

tan R  1

M A T H E M A T

1
tan R

I C S X

1
tan R  1

1
tan 1

sec q = cosec 60
cos q = sin 60
q = 30
1
3
3
... 2cos2 30 1 = 2  1 =  1 =
2
2
4
5. Hint: sec 4A = cosec (90 4A).
4. Hint:

tan 3 R  1

tan R

(tan 1)(tan2 + 1 + tan )

tan

tan 2 R 1
sec 2 R
1 =
1
=
tan R
tan R

6. Hint: cos (90 q) = sin q,


sin (90 q) = cos q.
7.

= 1 + sec cosec = RHS

1 sin 2
(1 + sin )(1 sin )
=
(1 + cos )(1 cos ) 1 cos2

OR

cos A
 cos A
cot A cos A
sin A
=
LHS =
cos A
cot A + cos A
cos A
sin A

cos A

sin A

cos A

sin A

49
cos2
7
= cot2 = =
.
2
64
8
sin

OR

cosec 2 + cot 2
cosec 2 sec 2
=

1
1
cosec A 1
sin A
=
=
= RHS
1
cosec
A
+
1
+1
sin A

WORKSHEET 52
1. (A) Hint: tan 5 = cot 85; tan 25 = cot 65.
2. (D) Hint: (1 + sin q) (1 sin q)= cos2 q
1
=
.
sec 2 R

sec2

225
x1=
64
sec x =

17
8

sec2

1 + 2 cot 2
cot 2 tan 2

1 2s3
7
21
=
=
.
1
8
8
3
3
3

8. sin2 30 + sin2 45 + sin2 60 + sin2 90


2

2
3
1
1
2
= +
+ (1)
+
2
2
2

1 1 3
+ + +1
4 2 4

289
x =
64

5
1+ 2 + 3 + 4 10
= .
=
2
4
4

9. Hint:

8
17

LHS =

Now, sin x cos x = 1 cos 2 x cos x

15 8
64
8
=

289 17
17

7
=
.
17
I N T R O D U C T

cos x =

= 1

1 cot 2 R  1 tan 2 R

225
64

tan2 x =

8 tan x = 15

3.

1 cot 2 R cot 2 R

T O

(sec R  tan R)2


OR

Hint:
LHS =

I O N

1  sin R 1  sin R

1 + sin R 1  sin R

T R

cos A
sin 2 A

1  tan A sin A  cos A

I G O N O M E T R Y

75

cos 2 A
sin 2 A

cos A  sin A cos A sin A

cos 2 A  sin 2 A
cos A  sin A

WORKSHEET 53
1. (A) tan q =

Perpendicular
3
=
4
Base
BC =
=

32 42

25 = 5

4
Base
cos =
=
Hypotenuse
5

4
1
1  cos R
5 = 1.
=
4
1 + cos R
9
1+
5

=
=

(cot A + cosec A ) (cosec2 A cot2A )


cot A + 1 cosec A

(cosec A + cot A) [1  cosec A + cot A]


cot A  cosec A + 1

9. Given expression
=

2 cot 15
2 sin 68o

5 tan 75
cos 22o

2 cot (90  75)


2 sin (90  22)

cos 22o
5 tan 75

1 + cot R + cosec R
1
+ sec R =
cot R
cot R

2 = 90 9 3 = 81

5. cot R 

76

3 cot 70 cot 50 tan 50 tan 70


5

=2

= 27.

cos R
1  cos2 R

6. True
Hint: A6 + B6
= (A2 + B2) [(A2 + B2)2 3A2 B2].
7. Hint:
LHS =

2 tan 75
2 cos 22

5 tan 75
cos 22

tan 2 = tan [90 ( + 9)]

tan 50 tan 70

tan 2 = cot ( + 9)

3 tan 45 tan 20 tan 40 tan 50 tan 70


5

31 tan (90 70 ) tan (90 50 )

3. Hint: A + B = 90; A B = 30.


4.

cot A 1 + cosec A
cot A + 1 cosec A

= cosec A + cot A = RHS.

2. (C)
Hint:
1 + tan + sec
=1+

cosA  sin A 1
cosA sin A  1

Dividing numerator and denominator by


sin A, we get

= cos A + sin A.
10. Hint: 1 2sin2 q = 2cos2 q 1
= cos2 q sin2 q

8. LHS =

1 2sin cos + 1 + 2sin cos


.
(sin + cos )(sin  cos )

2
5

1
1

tan 50 tan 70
tan 70 tan 50
5

2
3 10  2  3
5
=
=
=1 .
5
5
5
5
10. Given expression

=2

= 8 3 cosec2 30. sin 60. cos 60. cos2 45.


sin 45. tan 30. cosec3 45.
M A T H E M A T

I C S X

= 8 3

1
2

sin 30

. sin (90 30).

cos (90 30) cos2 (90 45). sin 45.


sin 30
1
3
cos 30 sin 45

= 8 3

1
2

sin 30

2 tan 30
2. (C)
=
1 + tan 2 30

3. Hint: tan x =

cos 30. sin 30. sin2 45.

sin x =

sin 30
1

3
cos 30 sin 45

sin 45.

sin 30. sin 30 cos 30


=8 3
cos 30
sin 2 30

4.

tan2 = x 

1
1

2
2
16x
1
1

16 x 2 2

4x

tan = x 

1
.
4x

3 34
3 34

3 3+4
3 34

43 24 3
27 + 16 24 3
=
.
11
27 16

1 sin 2 A
1 cos 2 A

sin A
cos A

cos2 A sin 2 A
sin A cos A

AB = (4 x )2 (3x )2 = 7 x
BC
3x
3
=
=
.
AB
7x
7

I N T R O D U C T

225
64
161

=
.
289 289
289

1
1

sin A
cos A
=
sin A
cos A

3
,
4

let BC = 3x and CA = 4x

Now, tan A =

15
8
, cos x =
17
17

5. Q A + B + C = 180
C+A
180 B

LHS = cot
= cot
2
2
B
B
= cot (90 ) = tan
2
2
= RHS.
6. Yes.
7
Hint: Both sides =
.
25
7. LHS = (cosec A sin A) (sec A cos A)

WORKSHEET 54
1. (B) As sin A =

15
8

1
2
+1
2
3 = 3 +2 3 4
=
2 1
4+ 3 +2 3
+ +1
3 2

OR

sec2 1 = x 2

2
3
= 3 =
.
4
2
3

sin 30 + tan 45 cosec 60


sec 30+ cos 60 + cot 45

= 8 3 111= 8 3.

Hint: sec2 = x2

1
3

1
1+

sin2 x cos2 x =

sin 2 45 sin 45
sin 3 45

I O N

= sin A cos A
T O

T R

I G O N O M E T R Y

... (i)

77

RHS =

1
1
=
sin A cos A
tan A + cot A
+
cos A sin A

4
=0
3
3. Hint: Divide numerator and denominator
by sin A.
3
1+
1 + cot A
4 .
=
3
1 cot A
1
4
2
4. sec A =
sec A = sec 30 A = 30
3
A + B = 90 B = 90 30 = 60

Now, 5 sin 3 tan = 5 0.8 3

= sin A cos A
sin2 A

cos2 A

(Q
+
From equations (i) and (ii), we obtain
LHS = RHS.

...(ii)
= 1)

8. 7 sin2 + 3(1 sin2 ) = 4


Let sin = x
7x2 + 3 3x2 = 4
1
1
x=
4x2 = 1 x2 =
4
2
1
1
sin =
or sin =
2
2
1
sin =
is not possible as is acute.
2
3
cosec = 2
cos =
2
2
sec + cosec =
2 . Hence proved
3
9. 1
Hint: cos (40 + ) = sin {90 (40 + )}
= sin (50 )
and
cos 40 = sin 50.
10. LHS = m2 n2 = (tan + sin )2
(tan sin )2 = 4sin tan

...(i)

sin 2 
sin 2
cos2 

= 4 sin tan
From (i) and (ii), LHS = RHS.

Now, cosec B = cosec 60 =

36 8
64
+ 2

1. (A) Required value = 25
100
100
6

1
= 25
(192 + 216 400)
300
1
2
=
8= .
12
3

( 3 ) + 4
2

1
2
+ 5 0
+ 3
2
3

2+2

( 3)

3+2+4
= 9.
43
6. False
Hint: A = 30, B = 60.

sec cosec
sec + cosec

7.

.... (ii)

WORKSHEET 55

2
3

5. Given expression

= 4 sin sec 2 1

78

1  0.36 = 0.8

0.8
4
sin
=
=
0.6
3
cos

And tan =

sin A cos A
sin 2 A + cos 2 A

RHS = 4 mn = 4

1 cos 2 =

2. (C) sin =

1
1

cos sin = sin R  cos R


1
1
sin R + cos R
+
cos
sin

1
sin R
=
1
sin R + cos R
s
sin R

3
4 =
3
1+
4

sin

R  cos R
s

1 cot
1 + cot

1
.
7

M A T H E M A T

Hence proved.

I C S X

cos R  sin R
cot 1
=
.
cot + 1
cos R + sin R

8.

(Dividing numerator and denominator by


sin )
p
1
q
pq
=
=
.
p
p+q
+1
q
9. Given expression
2

= 1 + 1 2 cos 60
1
= 22 s = 2 1 = 1
2
OR
Given expression

cot B =

n
and sin A = m sin B
tan A

sin B =

1
sin A
m

m
sin A

...

cosec2 B cot2 B = 1

m2
n2
=1

2
sin A tan 2 A

m2  n2 cos2 A
=1
sin 2 A
m2 1 = (n2 1) cos2 A

m2  1
= cos2 A.
n2 1
Hence proved
OR

cos 58
sin 22
+
cos (90 22 )
sin (90 58 )
cos 38cosec (90 38)

tan 18 tan 35 tan 60 tan (90 18)


tan (90 35)

cos 58
sin 22
+
cos 58
sin 22

cos 58
sin 22
+
sin 32
cos 68

Consider an equilateral triangle PQR in which PS QR.


Since PS QR so PS bisects
P as well as base QR.
We observe that PQS is a
right triangle, right-angled at
S with QPS = 30 and PQS = 60.
For finding the trigonometric ratios, we need
to know the length of the sides of the
triangle. So, let us suppose PQ = x
Then , QS =

cos 38 sec 38
tan 18 tan 35 tan 60 cot 18 cot 35
1
tan 18 tan 35
cos 38
tan 18 tan 35 tan 60

1
x
QR =
2
2

and (PS)2 = (PQ)2 (QS)2


= x2

cos 38

= 2

I N T R O D U C T

6 3
3
=
.
3
3

10. tan A = n tan B

cos 38cosec 52

tan 18 tan 35 tan 60 tan 72 tan 55

1
2 3 1
=

3
3

sin 35
cos 55 2cos 60
=
+

sin 35
cos 55

=2

cosec B =

sin 35
cos 55

=
+

cos (90 35 )
sin (90 55)
2 cos 60

1
tan 60

sin 35 cos 55
=
+
2cos 60
cos 55 sin 35

=2

I O N

T O

T R

PS =

x2
3x 2
=
4
4

3x
2

I G O N O M E T R Y

79

x
1
QS
(i) cos 60 =
= 2 =
2
PQ
x

5. Given expression
2
1 2

1 4
1
2
= 4 + 3
 1

2 2

3x
2 = 3
2
x
x
QS
2 = 1 .
(iii) tan 30 =
=
PS
3
3x
2
PS
(ii) sin 60 =
=
PQ

1
1
= 4
+
16 4

WORKSHEET 56
1. (B) b2x2 + a2y2 = b2a2 cos2 + a2b2 sin2
= a2b2.
2. (A) A = 90 60 = 30
cosec A = cosec 30 = 2.
tan =

3. (C)

6.

12
5

1 + tan2 = 1 +

sec =

cos 27
sin 29

cos 90  29

sin (90 27 )

80

4
= 0.
2

9
17
8

=
= 2.
4
4
4

1
cos A
sin A
sin A
+
=
+
tan A
sin A
1 + cos A
1 + cos A

cos A + cos 2 A + sin 2 A


sin A 1 + cos A

1 + cos A
= cosec A
sin A (1 + cos A )

= 2.
7. Q

sin =

3
4

cosec =

4
3

7
9
=
4
16

Q cos =

1 sin2 =

7
4
and cot =
3
7

sec =

Now, LHS
2

1
4

= 1 + 12

13
5

sin 29
cos 27
2
4.
+
4 cos 45
cos
61
sin 63

1
3
3
+1
+3
4
2
4

12
52

13 12
25

5
5
=
= 5 = 25.
1
13 12

5
5 5

1 3
3 1
2 4

1 + sin R
1 + sin
cos R
sec + tan
Now,
=
=
1  sin R
1 sin
sec tan
cos R

cosec 2 cot 2
=
sec 2 1

7
7
=
= RHS.
9
3

16 7
9

9 9 = 9
9
16
1
7
7

M A T H E M A T

Hence proved.
I C S X

8. Hint: LHS =

1
1

1
cos A sin A

sin A sin A

xy
2

= sin A
x
+
y

1
sin A

1 cos A sin A

and

x y = cos2 A

sin 2 A 1 + cos A
=
sin A (1 cos A )

2. (A) 5

1 cos 2 A 1 + cos A
sin A (1 cos A )
cos A (1 cos A )
sin A (1 cos A )

Hint: (x + 1)2 = x2 + 52

= cot A.

Using a3 + b3 = (a2 + b2 ab) (a + b), we get


3

sin + cos
+ sin . cos
sin + cos

(sin + cos )(sin 2 + cos2 sin . cos )


sin + cos
+ sin q cos q
.
.
= 1 sin cos + sin cos = 1 .

9.

C = 180 A B = 180 120

3
1
1
3

= 1.
2
2
2
2

4. cos =

1
2

cos = cos 60

= 60
1
3

tan =

2 (1 + sin )(1 sin )

2 (1 + cos )(1 cos )

tan = tan 30

= 30.

Now, sin ( + ) = sin (60 + 30) = sin 90 = 1.

cos2
1 sin 2
=
=
= cot2
sin 2
1 cos 2

5. tan 1 tan 2.... tan 43 tan 44 tan 45


tan 46 tan 47..... tan 88 tan 89

2
225
= 15 =
.
8
64

= (tan 1 tan 89)(tan 2 tan 88)....(tan 43


tan 47)(tan 44 tan 46) tan 45

10. Hint: p2 1 = sec2 + tan2 + 2 sec tan 1


= 1 + tan2 + tan2 + 2 sec tan 1
= 2 tan (tan + sec )
Similarly p2 + 1 = 2 sec (tan + sec ).

= (tan 1 cot 1)(tan 2 cot 2)....(tan 43


cot 43)(tan 44 cot 44) tan 45
= (1) (1) .... (1) (1) tan 45
= (1 1 .... 1 1) tan 45
= 1 1 = 1.

WORKSHEET 57
1. (B)
Hint:

1
= tan 30
3

= 60
Now, sin A cos C + cos A sin C
= sin 30 cos 60 + cos 30 sin 60

(2 + 2 sin )(1 sin )


(1 + cos )(2 2 cos )
=

3. (A) tan A =

A = 30

OR
3

sin2 A + cos2 A = 1.

6. Given expression
x + y = 2 cot A
x y = 2 cos A

I N T R O D U C T

=
I O N

T O

T R

tan 50 + sec 50
+ cos 40 cosec 50
cot 40 + cosec 40

I G O N O M E T R Y

81

tan 50 + sec 50
=
+
cot (90  50) + cosec (90  50)
cos 40cosec (90 40)
1
tan 50 + sec 50
+ cos 40.
cos 40
tan 50 + sec 50
= 1 + 1 = 2.

7. LHS = tan (A B) = tan (60 30) = tan 30


1
.
3

=
RHS =

1
31
3
3 =
=
=
1
1
+
1
1+ 3 .
3
3

8. RHS =

1
= LHS.
3

2
3
2

sin 6  3 sin 2 
1
.
+
+1
cos6 
cos2  cos2

sin 6 R + cos6 R + 3 sin 2 R cos 2 R


cos6 R

sin 2 R cos2 R

cos6 R

= sec6 R = LHS. [' sin 2 R +cos2 R = 1]


OR
Hint: Numerator of
LHS = tan + sec (sec2 tan2 )
= (tan + sec ) (tan + sec ) (sec tan )
= (tan + sec ) (1 sec + tan ).
9. cos + sin = 2 cos
Squaring both sides, we get
cos2 + 2 cos sin + sin2 = 2 cos2
2 cos2 cos2 2 cos sin = sin2
cos2 2 cos sin = sin2
Adding sin2 to both sides, we have
sin2 + cos2 2 cos sin = sin2
+ sin2

82

2 sin Hence proved.

l tan + m sec = n
...(i) l
l tan m sec = n ...(ii) l
ll tan + ml sec = nl
l l tan m l sec = n l

(m l + ml) sec = nl n l

10. Hint:

sec =

Similarly, tan =

nl  n l
m l ml

nm mn
.
lm ml

WORKSHEET 58
1. (D) Given expression
=

Hence verified.

cos sin =

tan A  tan B
tan 60  tan 30
=
1 + tan A tan B
1 tan 60 tan 30

(cos sin )2 = 2 sin2

cos2 (90 70) + cos2 70


sec 2 (90 40) cot 2 40

+ 2 {cosec2 58 cot 58 tan (90 58)}


=

sin 2 70 + cos 2 70
cosec 2 40 cot 2 40
+ 2 (cosec2 58 cot2 58)

1
+ 2(1) = 1 + 2 = 3.
1
2. (A) sec 5A = cosec (A 36)
sec 5A = sec {90 (A 36)}

5A = A + 126 A = 21.
=

3. Given expression
= sin2 5 + sin2 10 ... + sin2 40 + sin2 45
+ sin2 50 + ... + sin2 80 + sin2 85 + sin2 90
1
= cos2 85 + cos2 80 + .... + cos2 50 +

+ sin2 50 + .... + sin2 80 + sin2 85 + (1)2


= (cos2 85 + sin2 85) + (cos2 80 + sin2 80)
1
+ .... + (cos2 50 + sin2 50) + + 1
2
1
= (1 + 1 + .... 8 terms) + + 1
2
1
1
=8+
+1= 9 .
2
2
M A T H E M A T

I C S X

[(a2 b2)2 (a2 + b2)2] tan2 + 4 a2b2 + 4 ab


(a2 b2) tan (a2 + b2)2 = 0

1
1 1
1
1
.
+ = + =1
2 2
2 2
2

4. tan 3x =

4a2b2 tan2 + 4ab (a2 b2) tan a4 b4


+ 2a2b2 = 0

45o
tan 3x = tan 45 x =
= 15.
3
5. cosec A =

1
2

2 sin A =
1 sin 2 A =

cos A =

4a2b2 tan2 + 4ab (a2 b2) tan


(a2 b2)2 = 0
4a2b2 tan2 4ab (a2 b2)tan
+ (a2 b2)2 = 0
[2ab tan (a2 b2)]2 = 0

2ab tan = a2 b2

1
1
=
2
2

tan A = 1, cot A = 1

Now,

2 sin 2 A + 3 cot 2 A

4 tan 2 A cos2 A

1
+31
2
1

41

2
=

9. LHS

4
= 2.
2

a2 + b 2 = 16 sin2 + 9 cos2 12 sin


cos + 16 cos2 + 9 sin2
+ 12 sin cos
= 16 + 9 = 25.

(1+ cot A + tan A )(sin A cos A )


sec 3 A cosec 3 A

cos A
sin A

+
1 +
(sin A cos A )
sin
A
cos
A

=
1
1

cos3 A sin 3 A

a cos + b sin = 4 ...(i) sin


a sin b cos = 3 ...(ii) cos
a cos sin + b sin2 = 4 sin
a sin cos b cos2 = 3 cos

b = 4 sin 3 cos
a = 4 cos + 3 sin

(sin A cos A + cos2 A + sin 2 A)(sin A cos A)


sin A cos A
(sin A cos A)(sin 2 A + cos2 A + sin A cos A)
sin 3 A cos3 A
= sin2 A . cos2 A = RHS.

10. m = cosec sin =

7. (a2 b2) sin + 2ab . cos = a2 + b2


Divide by cos
(a2 b2) tan + 2ab =

n = sec cos =

(a2 b2) tan + 2ab = (a2 + b2) . sec

= (a2 + b2). 1 tan 2 R

Now, LHS = m 2 n

Squaring both sides:


(a2 b2)2 tan2 + 4 a2b2 + 4 ab (a2 b2) tan
= (a2 + b2)2 (1 + tan2 )
= (a2 + b2)2 + (a2 + b2)2 tan2
I O N

Hence proved.

1
sin
sin
2

a2 b 2
cos R

I N T R O D U C T

a2  b2
.
2 ab

8. Hint: Use (a2 + b2)3 = a6 + b6 + 3a2b2(a2 + b2).

6. True
Hint:

Similarly,

tan =

T O

2
3

1
cos
cos R
sin 2 R
1 cos2 R
=
cos R
cos R
2
2 3

) + (mn )
2

cos4 sin 2 3
=
+
sin 2 cos

T R

cos
1 sin2
=
sin
sin

cos2 sin 4 3

sin cos

I G O N O M E T R Y

83

= cos 3

2
3

) (

+ sin 3

2
3

4. True, because LHS = tan 60 =

= cos2 + sin2 = 1 = RHS.


OR
LHS
= (1 + cot A cosec A)(1 + tan A + sec A)

= 1

=
=

cos A
1
sin A
1


1

sin A sin A
cos A cos A

sin A + cos A 1
cos A + sin A + 1

sin A
cos A

(sin A + cos A )2 12
sin A cos A
2

sin A + 2 sin A cos A + cos 2 A 1


sin A cos A

2 sin A cos A
=2
sin A cos A

Hence proved.

ASSESSMENT SHEET 9

x=
sin 30 =

y=

y3
AB
1

=
AC
2
20 3

(Given)

2 sin x cos y

=4

... (i)

2
cos 55o

= cos(90o  35o) +

\
^
cos 55o
cos(90o  35o)
2 cos 60

1
4

1
2
1
1
When sin x = , cos y =
2
2
1
1
When sin x = , cos y =
2
2

3. We know: sin = cos (90 ) so, given


expression

84

=4

2(sin2 x + sin2 x) = 1 sin2 x =

10 3
cm.
3

x cos y

2 sin 2 x cos 2 y

20
20 3
x=
cm
3
3

cos 55
cos 55
1
=
+
2

cos
55
cos
55

2
= 1 + 1 1 = 1.

(Given)

4
= 41
2(sin2 x + cos2 y) = 1
... (ii)
Substituting cos y = sin x from (i) in (ii),
we get

2. 25
Hint: tan 2 = cot ( + 15)
tan 2 = tan [90 ( + 15)].

2 sin x + cos y = 1
2 sin x + cos y = 20
sin x + cos y = 0
16sin

3
10
BC

=
2
x
AC

3.

5. 2 (sin6 A + cos6 A) 3 (cos4 A + sin4 A) + 1


= 2{(sin2 A + cos2 A)3 3sin2 A cos2A
(sin2 A + cos2 A)}
2
2
2
3{(sin A + cos A) 2sin2 A cos2 A} + 1
[Q (a + b)3 = a3 + b3 + 3ab(a + b)
and (a + b)2 = a2 + b2 + 2ab]
2
= 2(1 3sin A cos 2 A) 3(1 2sin2 A
cos2 A) + 1
2
[Q sin A + cos2 A = 1]
= 2 6 sin2 A cos2 A 3 + 6 sin2 A cos2 A
+1=0
6.

= RHS.

1. (A) cos 30 =

2
2 tan 30
3
RHS =
=
=
1
1 tan 2 30
1
3

3 and

sin x =

Hence, sin x =
cos y =

1
1
1
, cos y =
or sin x = ,
2
2
2

1
.
2

7. We know that
sin (90 ) = cos , tan (90 ) = cot ,
sec(90 ) = cosec
M A T H E M A T

I C S X

Now,

ASSESSMENT SHEET 10

sec 2 (90 ) cot 2


2(sin 2 25 + sin 2 65)

1. (A) Given expression


= sin 25 cos (90 25) + cos 25
sin (90 25)
2
2
= sin 25 + cos 25 = 1.

2 cos2 60 tan 2 28 tan 2 62


+
3(sec 2 43 cot 2 47)
=

sec 2 (90 ) cot 2

2cos2 60 tan 2 28 tan 2 (90 28)

3 sec2 43 cot2 (90 43 )

cosec 2 R  cot 2 R
2 sin 2 25o cos 2 25o

2cos2 60 tan 2 28 cot 2 28


3(sec 2 43 tan 2 43)

1
=
+
21
=

2 sin 2 25 + sin 2 (90 25 )

1
2 1
4
31

q=

2 2

3 +1

)(

3 1

3 3

4 2

2
2

cos A
1 + sin A
+
cos A
1+ sin A
2

=
... (iv)

1 +

sin A
cos A

cos 2 A + 1 + sin 2 A + 2sin A


1 + sin A
cos A

2 1 sin A

2 2 sin A
=
1

sin
A
cos
A

1 sin A
cos A

2
= 2 sec A
cos A
= RHS.

q
1) = 2(sin + cos )
2
q (p 1) = 2p.
Hence proved.
T O

cos2 A + 1 + sinA

(p2

I O N

3 1

3 2 6
.
8

5. LHS =

sin R cos R
2sin cos
sin R cos R

I N T R O D U C T

4. False, because cos2 23 sin2 67 = 0, 0 is


not a positive value.

1
1
+
cos
sin
sin R cos R
sin R cos R

From equations (iii) and (iv), we get


q (p2 1) =

8. Given equations are:


sin + cos = p
... (i)
sec + cosec = q
... (ii)
Squaring both the sides of equation (i),
we get
sin2 + cos2 + 2sin cos = p2
Subtract unity from both the sides to get
p2 1 = 2sin cos
... (iii)
Equation (ii) can be written as

4
2 1
2 tan 1
5
3
=
=
=
.
4
2 tan + 1
11
2 1
3
1
cos 45o
2
3.
=
2
sec 30o cosec 30
+2
3
1
1
2
3
=
=

2
2 1+ 3
2 1 3

1 1
2
+ = .
2 6
3

q=

2 sin R
cos R

2 sin cos
cos R
cos R
2.
=
cos R
2 sin R
2sin + cos

cos R
cos R

T R

Hence proved.

I G O N O M E T R Y

85

6. Let us construct a triangle ABC in which


AB = BC = AC = a (say). Draw AD BC.
AD bisects BC
a

BD = DC =
2
AD bisects BAC

= 30
In right angled ABD.
2
a
AD2 = AB2 BD2 = a2
2
2
2
a
3
a
= a2
=
4
4
3

AD =
a
2
Now, in ABD,
a
BD
tan 30 = 2
tan =
a
AD
3
2
1
tan 30 =
.
3
7. ( a2 b2) sin + 2ab cos = a2 + b2 (Given)
Divide both sides by cos to get
( a2 b2) tan + 2ab = (a2 + b2) sec
Squaring both sides, we get
(a2 b2)2 tan2 + 4a2 b2 + 4ab(a2 b2) tan
= (a2 + b2)2 sec2
(a2 b2)2 tan2 (a2 + b2)2 tan2 + 4ab
(a2 b2) tan (a2 + b2)2 + 4a2b2 = 0
(Q sec2 = 1 + tan2 )
2
2
2
4a b tan + 4ab (a2 b2) tan
(a2 b2)2 = 0
2
2
2
2
2
4a b x + 4ab (a b ) x (a2 b2)2 = 0
where x = tan
This is a quadratic equation in x.
Here, discriminant,
2

D = 16a2b2 a2  b2
 4 s 4a2b2 a2  b2

=0
x=

 4 ab( a2  b2 )  0
a2  b2

2ab
2 s (  4 a2 b 2 )

tan =

86

a2  b2
.
2 ab

Hence proved.

8. Since ABC is a acute angled triangle


so, A < 90, B < 90 and C < 90.
Also A + B + C = 180
... (i )
1
sin (A + B C) =
(Given)
2
sin (A + B C) = sin 30
A + B C = 30
... (ii )
Similarly, B + C A = 45
... (iii )
Add equations (ii ) and (iii ) to get
1o
2B = 75 B = 37
2
Subtract equation (ii) from equation (i) to get
2C = 150 C = 75
Subtract equation (iii) from equation (i) to get
2A = 135 A = 67
Thus, A = 67

1o
2

1o
1o
, B = 37
and C = 75.
2
2

CHAPTER TEST
1. (A) x =

sec
1
tan
and
=
2
x
2

2 x2

1
=2
x2

sec 2 R tan 2 R

= 2

2.

cos 2 20 + cos 2 70o


2 sin 59 sin 31o

tan 2 R

1
.
2

2
k

sin 2 70o cos2 70o


2 sin 59o cos 59o

sec 2 R

2
k

1 2
 k = 4.
2 k

3. sin4 + cos4 = 1 + 4k sin2 cos2


(sin2 + cos2 )2 2 sin2 cos2
= 1 + 4k sin2 cos2
2 sin2 cos2 ( 1 2k) = 0
1 2k = 0 k =

1
.
2

M A T H E M A T

I C S X

4.

tan2

tan = 4
+ 1 = 42 + 1
sec2 = 17

1
1
(tan2 + 2 sec2 ) =
(16 + 2 17)
10
10
= 5.
5. False.
Suppose A = 30 and B = 60
Then, LHS = tan (A + B) = tan (30 + 60)
= tan 90
LHS
= undefined
.... (i)
and RHS = tan A + tan B = tan 30
+ tan 60

9. LHS =

1
1+ 3
4
+ 3 =
=
3
3
3

RHS = a real number


.... (ii)
From results (i) and (ii), it is clear that the
given identity is false.

7.

cos 55 = cos (90 35) = sin 35


cos 70 = sin 20
and tan 5 = cot 85.

13 .
4

1
3
Hint: sin 30 =
= cos 60, sin 60 =
,
2
2
1
cos 45 =
= sin 45, sin 90 = 1.
2

Now,

sin6

cos6

... (i)

= sin + cos
2

3 sin2 cos2 (sin2 + cos2 )

a2 1
= 13 3
(1)
2
[Using equation (i)]

I N T R O D U C T

I O N

T O

1 + tan 2 + 2 sec tan

sec + tan 2 + 1 + 2 sec tan


tan 2 R + tan 2 R + 2 sec R tan R
sec 2 R + sec 2 R + 2 sec R tan R

2 tan (tan + sec )


2 sec (sec + tan )

tan
= tan cos
sec
sin
. cos = sin = RHS.
cos
Hence proved.
OR

sin

sin2 + cos2 + 2 sin cos = a2

2 sin cos = a2 1
a2 1
2

(sec

sin A sin B cos A cos B


+
cos A + cos B sin A + sin B

Squaring both sides.

sin cos =

sec 2 + tan 2 + 2 sec tan 1


sec 2 + tan 2 + 2 sec tan + 1

8. sin + cos = a

(sec + tan )2 1
(sec + tan )2 + 1

1
7
Hint:

Hence proved.

6.

2
4 3 (a 2 1)2
3 2
.
a 1 =
4
4

= 1

A sin B
sin A + sin B

cos A cos B
cos A cos B

cos

A + cos B
sin A + sin B

sin 2 A sin 2 B cos 2 A cos 2 B


cos A + cos B
sin A + sin B

sin

A cos 2 A)  (sin 2 B cos 2 B

cos

A + cos B
sin A + sin B

11
(cos A + cos B )(sin A + sin B )

= 0 which is an integer.

qq
T R

I G O N O M E T R Y

87

Chapter

STATISTICS
Now, l = 30, f1 = 32, f0 = 12, f2 = 20, h = 10
Use the formula:

WORKSHEET 62
1. (B) 21.1
Hint: 3 Median = Mode + 2 Mean.
2. Since the mode is 7
2k 1 = 7 k = 4.
3. In such case, mean will increase by 3.
New mean = 18 + 3 = 21.
4. x = 26
Hint: Mean =
5.

Class
interval
0-8
8-16
16-24
24-32
32-40
40-48

4f i x i
.
4f i

Frequency ( f )
8
10
16
24
15
7
N = 80

Cumulative
Frequency (cf )
8
18
34
58
73
80

N
80
= 40
=
2
2
In the cumulative frequency column, 58 is
just greater than 40.

For median class,

So, 24-32 is the median class.


N
Here, l = 24, cf = 34, f = 24,
= 40, h = 8
2
Using formula:
N

cf

Median = l + 2
f h

40 34
= 24 +
8 = 26
24

Hence, median of the given distribution is 26.


6. 36.25
Hint: Here maximum class frequency is 32.
So, the modal class is 30-40.

88

f1 f0
Mode = l +
h.
2 f1 f0 f2
7. Hint:
Production No. of Production yield
yield
farms
(in kg/ha)
(in kg/ha)
more than or
equal to

cf

50-55
55-60
60-65
65-70
70-75
75-80

2
50
100
8
55
98
12
60
90
24
65
78
38
70
54
16
75
16
100
80
0
For more than ogive, plot following points:
(50, 100), (55, 98), (60, 90), (65, 78), (70, 54),
(75, 16), (80, 0).

WORKSHEET 63
1. (B) median
2. 4
3. The given distribution can be represented as:
Marks obtained
No. of students
0-10
5
10-20
3
20-30
4
30-40
3
40-50
6
More than 50
42
Clearly, the frequency of the class 30-40 is 3.
4. Let us rewrite the given table with cumulative frequencies.
Class interval
f
cf
0-5
10
10
5-10
15
25
10-15
12
37
15-20
20
57
20-25
9
66
N = 66
M A T H E M A T

I C S X

These points are joined by a free hand


smooth curve to obtain a less than type ogive
as shown in the figure.

N = 66

N
=
2
Median class =
Modal class =
Required sum =

33
10-15
15-20
10 + 15 = 25.

5. In the given distribution, maximum class


frequency is 20, so the modal class is 40-50.
Here, lower limit of modal class: l = 40
Frequency of the modal class: f1 = 20
Frequency of the class preceding the modal
class: f0 = 12
Frequency of the class succeeding the modal
class: f2 = 11
Size of class: h = 10
Using the formula:

f1 f 0
Mode = l +
h
2 f1 f0 f2
20 12

= 40 +
10
2 20 12 11

= 40 + 4.70 = 44.70.
Hence, mode of the given data is about 45
cars.
6. Let us rewritten the table with class intervals.
Class interval

cf

36-38
38-40
40-42
42-44
44-46
46-48
48-50
50-52

0
3
2
4
5
14
4
3

0
3
5
9
14
28
32
35

Figure: Less than type ogive


To obtain median from the graph:
We first locate the point corresponding to
35
N
=
= 17.5 students on the y-axis. From
2
2
this point, draw a line parallel to the x-axis

to cut the curve at P. From the point P, draw


a perpendicular PQ on the x-axis to meet it
at Q. The x-coordinate of Q is 46.5. Hence,
the median is 46.5 kg.
Let us verify this median using the formula.
N cf
Median = l + 2

17.5  14
2
14

= 46 +

N = 35
We mark the upper class limits on x-axis
and cumulative frequencies on y-axis with a
suitable scale.
We plot the points (38, 0); (40, 3); (42, 5);
(44, 9); (46, 14); (48, 28); (50, 32) and (52, 35).
S T A T

S T LI C S

= 46 +

7
= 46 + 0.5
14

= 46.5 kg.
Thus, the median is the same in both
methods.

89

7. (i) By making the given data continuous, we get: a = 57, h = 3.


No. of mangoes
49.5-52.5
52.5-55.5
55.5-58.5
58.5-61.5
61.5-64.5

No. of boxes (fi)

Mid-points (xi)

15
110
135
115
25

51
54
a = 57
60
63

xi  a
h
2
1

ui 

f iui
30
110
0
115
50

0
1
2

fi  400

Mean = a + h

f i ui

fi

fi ui  25
= 57 + 3

25

400

= 57 +

75
! 57.19.
400

(ii) Step devitation method


(iii) Vikram Singh believes in quality serving, fruits will remian fresh and free from germs and
flies.

WORKSHEET 64
1. (C) mid-points of the classes.
2. (D) We have
Mode = 3 Median 2 Mean

45 = 3 Median 2 27
Median = 33.

Using the formula:


4f u
Mean = a + i i h
4fi
50 = 52 +

1.1 f1 0.9 f2 = 9.2


68 + f1 + f2 = 120

5 + 8+ 3+ 2
N
=
= 9.
2
2

4. Required number of athletes


= 2 + 4 + 5 + 71 = 82.

f1 + f2 = 52

...(ii)

Solving (i) and (ii), we obtain


f1 = 28
f2 = 24.

and

6. Let us convert the given data into less than


type distribution.

5.

Class
interval

Let us assumed mean be a = 52


Here,
h = 20

90

...(i)

But

3. 25-30
Hint:

2.8 1.1 f1 + 0.9 f 2


q 20
120

Lifetimes
(in hrs.)

0-20

10

less than 20

10

20-40

35

less than 40

45

40-60

52

less than 60

97

60-80

61

less than 80

158

80-100

38

less than 100

196

100-120

29

less than 120

225

M A T H E M A T

cf

I C S X

We mark the upper class limits along the


x-axis with a suitable scale and the cumulative frequencies along the y-axis with a
suitable scale. For this, we plot the points
A(20, 10), B(40, 45), C(60, 97), D(80, 158),
E(100, 196) and F(120, 225) on a graph paper.
These points are joined by a free hand
smooth curve to obtain a less than type ogive
as shown in the given figure.

Mean:
Q

Mean =

fi xi

fi
fi = 50 and fi xi = 7260

7260
= 145.20.
50
Hence, the mean is ` 145.20
Median:

Mean =

N cf
h
Median = l + 2

N
Q N = 50,
= 25, f = 14, cf = 12,
2
l = 120 and h = 20

Median = 120 +

25  12
20
14

= 120 + 18.57 = 138.57


Hence, the median is ` 138.57.
Mode:

f1  f0
Mode = l +
h
2 f1  f0  f2
Q l = 120, f1 = 14, f0 = 12
f2 = 8 and h = 20

Mode = 120 +

14  12

20
2 s 14  12  8

40
= 125
8
Hence, the mode is ` 125.
= 120 +

8. C.I.

Figure: Less than type ogive


7. The given distribution can be again
represented with the cumulative frequencies
as given below:

No. of consumers (fi)

(c.f.)

65-85

85-105

105-125

13

22 = c.f.

125-145

20 = f

42

145-165

14

56

Class interval

fi

xi

cf

fi xi

165-185

64

100-120
120-140
140-160
160-180
180-200

12
14
8
6
10
50

110
130
150
170
190

12
26
34
40
50

1320
1820
1200
1020
1900
7260

185-205

68

S T A T

S T LI C S

N = 68

N 68
=
= 34 c.f. just greater than 34 is 42.
2
2

Median class is 125-145.

91

As the maximum frequency is 30, the modal


class is 30-40.

N
c. f .
Median = l + 2
h
f

4.

34  22
= 125 +
20
20

= 125 + 12 = 137.
(ii) 20 + 14 + 8 + 4 = 46 families.
(iii) Since, Mr. Sharma is saving electricity so his
consumption is less, which means his
monthly bill will also be less. So, he believes
in saving and hence is responsible also.

1. (B) 30-40
Hint:

0-10
10-20
20-30
30-40
40-50
50-60
60-70

Frequency
(f)

Cumulative
Frequency

4
4
8
10
12
8
4

4
8
16
26
38
46
50

2. 45
Hint:
Draw a line parallel to the x-axis at the point
40
y=
= 20. This line cuts the curve at a
2
point. From this point, draw a perpendicular to the x-axis. The abscissa of the point
of intersection of this perpendicular with the
x-axis determines the median of the data.
3. The given distribution can also be represented
as follows:

92

fi

xi

fixi

1-3
3-5
5-7
7-10

9
22
27
17

2
4
6
8.5

18
88
162
144.5

fi = 75
Mean =

WORKSHEET 65

Class interval
(C.I.)

C.I.

Class interval

Frequency

0-10
10-20
20-30
30-40
40-50
50-60

3
9
15
30
18
5

fi xi
fi

fi xi = 412.5

412.5
= 5.5.
75

5. In the given distribution, the classes are in


the inclusive form. Let us convert them into
163  162
exclusive form by subtracting
, i.e.,
2
0.5 from lower limit and adding the same to
upper limit of each class.
Class interval

159.5-162.5

15

162.5-165.5

118

165.5-168.5

142

168.5-171.5

127

171.5-174.5

18

Here, the maximum frequency is 142.


l = 165.5, fl = 142, f0 = 118, f2 = 127, h = 3
Now,
f1 f0

mode = l +
h
2 f1 f0 f2

= 165.5 +

142  118

284  118  127

= 165.5 + 1.85 = 167.35


Hence, the modal height of the students is
167.35 cm.
6. The given data may be re-tabulated by the
following manner with corresponding
cumulative frequencies.
M A T H E M A T

I C S X

Heights (in cm.)


C.I.

ogive, take upper class limits and


corresponding cumulative frequencies; and
for drawing the more than type ogive take
lower class limits and corresponding
cumulative frequencies.

No. of girls Cumulative


( f)
frequency
(cf )

Below 140
140-145
145-150
150-155
155-160
160-165

4
7
18
11
6
5

4
11
29
40
46
51

ASSESSMENT SHEET 11
1. (A) Here, a = 25, h = 10.

N = 51

N
= 25.5.
2
This observation lies in the class 145-150.
Then l = 145, cf = 11, f = 18, h = 5
N

cf

Now,
median = l +
h
f

fi ui
x =a+h

fi

= 25 + 10

Now, N = 51. So,

25.5 11
= 145 +
5
18

= 149.03.
Hence, the median height of the girls is
149.03 cm.
7. C.I.

Sum of first 6 numbers = 6 32 = 192


Sum of last 6 numbers = 6 37 = 222
6th number = 192 + 222 385 = 29.
3. The modal class is 30-40.
h = 10, f1 = 32, f0 = 12, f2 = 20, l = 30.
f1 f0

Mode = l +
h
2
f
1 f0 f 2
32  12
10
64  12  20

= 30 +

xi

f ix i

10-12

11

77

12-14

12

13

156

= 30 + 6.25

14-16

18

15

270

= 36.25.

16-18

13

17

221

Mean mileage =

= 14.48 km/l.
(ii) No, the manufacturer is claiming mileage
1.52 km/l more than average mileage.
(iii) The manufacturer should be honest with
his customer.
8. 69.5.
Hint: Change the given distribution into
less than type and more than type
distributions. For drawing the less than type
S T A T

4. False,
Hint: N = 5 + 15 + 30 + 8 + 2 = 60
N

= 30
2

fixi = 724

4 f i xi
724
=
50
4 fi

S T LI C S

= 27.

2. Sum of 11 numbers = 11 35 = 385

fi

fi = 50

20

100

5.

xi

fi

f ixi

3
6
7
4
p+3
8

5
2
3
2
4
6

15
12
21
8
4p + 12
48

fi = 22

fixi = 4p + 116

93

Mean =

6.

6=

fi xi
fi
4 p 116
22

132 = 4p + 116
4p = 16

p = 4.

Class interval

Frequency
(f)

0-20
20-40
40-60
60-80
80-100

4
6
18
8
14

8. The cumulative frequency table for the given


data is given below:

The class corresponding to the maximum


frequency is 40-60. So, 40-60 is the modal
class.
f1 f0

Mode = l +
h
2 f1 f0 f2

Here, l = 40, f1 = 18, f0 = 6, f2 = 8 and h = 20


18 6

Mode = 40 +
20
2 18 6 8

= 40 +

12 s 20
= 50.91.
22

7. We notice classes are continuous. We form


cumulative frequency table by less than
method.
Marks
(C.I.)
0-10
10-20
20-30
30-40
40-50
50-60

Number Marks
of stuless
dents
than
5
8
10
9
6
7

10
20
30
40
50
60

cf

Point

5
13
23
32
38
45

(10, 5)
(20, 13)
(30, 23)
(40, 32)
(50, 38)
(60, 45)

On plotting these points on a graph paper


and joining them by a free hand smooth
curve, we get a curve called less than ogive.

94

Marks
(C.I.)

No. of
students
(f)

Cumulative
frequency
(cf )

0-10
10-20
20-30
30-40
40-50
50-60

10
f1
25
30
f2
10

10
10 + f1
35 + f1
65 + f1
65 + f1 + f2
72 + f1 + f2

N = 75 + f1 + f2
Clearly,
N = 75 + f1 + f2
But
N = 100

f1 + f2 = 25

... (i)

N
= 50.
2

The median is 32 which lies in the class


30-40.
So, l = 30, f = 30, cf = 35 + f1, h = 10.
Using the formula:

Median = l +

32 = 30 +

N
 cf
2
f

50  35  f1

30

M A T H E M A T

10

I C S X

2
15  f1
=
10
30

75 5f1 = 30

75  30
5
f1 = 9

f1 =

Substituting f1 = 9 in equation (i), we get


9 + f2 = 25 f2 = 16
Hence, f1 = 9 and f2 = 16.

5. Let us use the assumed mean method to find


the mean of the given data.
Marks No. of Class di =
(C.I.) students mark xi 35
( fi )
(xi)
0-10
10-20
20-30
30-40
40-50
50-60

4
6
8
10
12
30

x
n
x1
x
2 + .......... + n  x
k
k
k
k

(Dividing throughout by k)

x1 x2
xn
.........
k
k = x
k
k
n
(Dividing throughout by n)
Required mean =

x
.
k

2. The first ten prime numbers are:


2, 3, 5, 7, 11, 13, 17, 19, 23, 29.
Median =

11 13
24
=
= 12.
2
2

Mean =

3.

5 6 + 10 k + 15 6 + 20
10 + 25 5
15 =
6 + k + 6 + 10 + 5

4f i xi
4fi

445 10k
= 15
27 k
k = 8.

4. False, because the values of these three


measures depend upon the type of data, so
it can be the same.

30
20
10
0
10
20

fi = 70

ASSESSMENT SHEET 12
1. (D) x1 + x2 + ................ + xn = n x

5
15
25
35
45
55

f i di
120
120
80
0
120
600
fi di = 400

Here, assumed mean, a = 35


Now,

required mean = a +

fi di
fi

400
= 35 + 5.71 = 40.71.
70
6. Since mode = 36, which lies in the class
interval 30-40, so the modal class is 30-40.
f1 = 16, f0 = f, f2 = 12, l = 30 and h = 10.
= 35 +

Now,

mode = l +

f1  f0

2f  f  f

1
0
2

16  f
36 = 30 +
10
32  f  12

16  f
6
=
10
20  f

120 6f = 160 10 f
4 f = 40

f = 10.

7. 31.5 marks.
Hint:
Classes

No. of
students

Cumulative
frequency

0-10
10-20
20-30
30-40
40-50
50-60

5
8
6
10
6
6

5
13
19
29
35
41

Draw the ogive by plotting the points:


(10, 5), (20, 13), (30, 19), (40, 29), (50, 35)

S T A T

S T LI C S

95

N
= 20.5. Locate the
2
point on the ogive whose ordinate is 20.5.
The x-coordinate of this point will be the
median..

We plot the points given in this last table


on the same graph and join them by free
hand smooth curve to draw the cumulative
frequency curve by more than method (see
figure).

8. We prepare the cumulative frequency table


by less than method as given below:

Median: The two curves intersect each


other at a point. From this point, we draw
a perpendicular on the x-axis. The foot of
this perpendicular is P(375, 0). The abscissa
of the point P, i.e., 375 is the required
median.

and (60, 41). Here

CumuFre- Score lative


Scores quency less
fre(f)
than quency
(f)

Point

Hence, the median is 375.

200-250

30

250

30

(250, 30)

250-300

15

300

45

(300, 45)

300-350

45

350

90

(350, 90)

350-400

20

400

110

(400, 110)

400-450

25

450

135

(450, 135)

450-500

40

500

175

(500, 175)

500-550

10

550

185

(550, 185)

550-600

15

600

200

(600, 200)

We plot the points given in above table on a


graph paper and then join them by free
hand smooth curve to draw the cumulative
frequency curve by less than method.
Similarly for the cumulative frequency
curve by more than method, we prepare
the corresponding frequency table.

Scores

96

CumuFreScore lative
quency more
fre(f)
than quency
( cf )

Point

200-250

30

200

200

(200, 200)

250-300

15

250

170

(250, 170)

300-350

45

300

155

(300, 155)

350-400

20

350

110

(350, 110)

400-450

25

400

90

(400, 90)

450-500

40

450

65

(450, 65)

500-550

10

500

25

(500, 25)

550-600

15

550

15

(550, 15)

Figure: Less than and more than type


cumulative frequency curves

M A T H E M A T

I C S X

CHAPTER TEST

2
1
mode + mean
3
3
2
2
= mode mode + mean
3
3
2
= mode + (mean mode).
3

Median =

fi = N

1. (A) Let

(fixi x ) = fi xi N x

4f i xi

=N

 x

= N ( x x ) = 0.
2. Let us rewrite the given distribution in the
other manner.
Marks

No. of students

0-10
10-20
20-30
30-40
40-50
50-60

3
9
15
30
18
5

C.I.

xi

fi

800-820 810

40
 2
20

14

820-840 830

14

20
 1
20

14

840-860 850

19

0
0
20

860-880 870

15

20
1
20

15

880-900 890

40
2
20

18

Clearly, the modal class is 30-40.


3. 17.5
Hint: First, transform the given class-intervals
into exclusive form and then find the
cumulative frequency table.

N
= 28.5.
2

4.

Monthly income
(in `)

No. of
families

10000-13000
13000-16000
16000-19000
19000-22000
22000-25000
25000 or more

15
16
19
17
18
15

Hence, required number of families is 19.


5. No, because an ogive is a graphical representation of a cumulative frequency distribution.
6. Yes; as we know
mode = 3 median 2 mean
3 median = mode + 2 mean
S T A T

S T LI C S

f iu i

xi A
h

fi = 64

Here, N = 13 + 10 + 15 + 8 + 11 = 57

ui =

7.

fiui = 5

Let assumed mean be


A = 850
h = 20
u f
Mean = A + i i
fi

64

20

= 850 +

= 850 + 1.5625 = 851.5625.


Hence, the required mean is 851.5625.
f1 f0

Mode = l +
h
2
f
1 f0 f 2

8.

Here, l = 30, f1 = 45, f0 = 30, f2 = 12, h = 10

45 30
Mode = 30 +
10
90
30 12

= 30 + 3.125 = 33.125 marks.

97

9. (i)

Class intervals
(in daily pocket
allowances) (in `)

Frequency
(No. of children)
( fi )

Mid-points
of C.I.
(xi )

f ix i

11-13
13-15
15-17
17-19
19-21
21-23
23-25

7
6
9
13
x
5
4

12
14
16
18
20
22
24

84
84
144
234
20
110
96

fi = 44 + x

Mean =

4 f i xi
752 20 x
4 fi = 44 x

As Mean = ` 18 (given) 18 =

fixi = 752 + 20x

752 20 x
44 x

792 + 18x = 752 + 20x 40 = 2x x = 20.

(ii) Arithmetic mean of grouped data.


(iii) One shouldn't be spend thrift, but should save his money for future use.

qq

98

M A T H E M A T

I C S X

PRACTICE PAPERS

Practice Paper-1
SECTIONA
1. (D)

3. ...

4.

xy
1800
=
HCF( x , y)
12
= 150.
9 < 90
< 10
is an acute angle.
cos 9 = sin

a b

PRACTICE PAPER1

b
ab

(Comparing both)

k2 = 4k

k2 4k = 0
k = 4, 0.

1339 = 6k

k
k
13
=
=
k+4
6
39

k
13
=
k=2
k+4
39
3k = k + 4
k=2
k = 2, i.e., k = 2.

and
and
and
9. Yes.

262 = 242 + 102 = 676

Here,

AC = AB + BC
\DABC is a right triangle.
2

OR
No,
FED ~ STU
'
Corresponding sides of the similar triangles
are in equal ratio.

SECTION B

k2
=k
4

8. For infinitely many solutions,

a
a
Q

cos 9a = cos B
2

B a =
9a =
2
20

tan 5a = tan
= 1.
4

5. True, because out of any two consecutive


positive integers, one is even and the other
one is odd; and the product of an even and
an odd is even.
6. We know that the factors of a prime are 1
and the prime itself only.
Therefore, the common factor of p and q
will be 1 only. Hence, HCF (p, q) = 1.
7. No, if two zeroes are and of polynomial
x2 + kx + k, then
+ = k and . = k

k( k 4) = 0

LCM (x, y) =

a = k and a = k (when a = b)
a = 2k and a = k.
2

c1
a1
b
10
6
3
=
= 3, 1 =
= 3,
=
c
a2
b2
2
9
2
1

i.e., aa = bb cc
The given lines are parallel.
2. (B) D = Q and E = R
DDEF ~ DQRP (AA rule of similarity)
DE
EF
PQ
RP
.

10.

EF
DE
=
ST
TU

DE
ST

EF
TU
.

xi

fi

fixi

3
5
7
9
11
13

6
8
15
p
8
4

18
40
105
9p
88
52

Sf = p + 41 Sf x = 9p + 303
i

i i

99

Mean =

f i xi
fi

9p + 303
7.5 =
p + 41

7.5p + 307.5 = 9p + 303 1.5p = 4.5


\
p = 3.
SECTION C
11. On the contrary let us assume that 2 3
3 2 is a rational number. Then, we can
take coprime a and b such that
a
= 2 3 3 2
b

12

a2
= 12 + 18 12 6
b2
(Squaring both sides)
2
6 = 30 a 2
b

6=

30 b 2 a2
12 b 2

Since, a and b are integers, therefore, RHS


of this last equation is rational and so LHS
must be rational.
But this contradicts the fact that 6 is
irrational.
This contradiction has arisen due to

a2 = (6m + 5)2 = 36m2 + 60m + 25


= 36m2 + 60m + 24 + 1
= 12(3m2 + 5m + 2) + 1 = 6q + 1,
where q = 2(3m2 + 5m + 2).
Hence, a2 is of the form 6q + 1 or 6q + 3.
OR
As: 1032 = 408 2 + 216
...(i)
408 = 216 1 + 192
...(ii)
216 = 192 1 + 24
...(iii)
192 = 24 8 + 0
...(iv)
HCF = 24
From (iii)
24 = 216 192
= 216 [408 216] [Using (ii)]
= 2 216 408
= 2[1032 2 408] 408
[Using (i)]
24 = 1032 2 5 408
m = 2.

100

13. Since x 5 is a factor of


f (x) = x3 3 5 x2 + 13x 3 5 ,
so as f (x) may be rewritten.
f (x) = x3 3 5 x2 + 13x 3 5
= x3 5 x2 2 5 x2 + 10x

incorrect assumption that 2 3 3 2 is a


rational number.
So, we conclude that 2 3 3 2 is an
irrational number.
12. Let a be any odd positive integer. Then it
is of the form 6m + 1, 6m + 3 or 6m + 5,
where m is an integer.
Here, 3 cases arise.
Case I. When a = 6m + 1,
a2 = (6m + 1)2 = 36m2 + 12m + 1
= 12m (3m + 1) + 1
= 6q + 1, where q = 2m (3m + 1).
Case II. When a = 6m + 3,
a2 = (6m + 3)2 = 36m2 + 36m + 9
= 36m2 + 36m + 6 + 3
= 6(6m2 + 6m + 1) + 3 = 6q + 3,
where q = 6m2 + 6m + 1.
Case III. When a = 6m + 5,

+ 3x 3 5

= x2 x 5
2 5 x x 5

+ 3 x 5

= x 5

) (x

2 5x + 3

To find zeroes of f (x), put f (x) = 0.

( x 5 ) (x

x 5
x=
or

or

x =

2 5x + 3 = 0

= 0 or x2 2 5 + 3 = 0
5

2 5 20 4 1 3
2

x =

2 5 2 2
2
5 or x = 5 + 2

x =

5 or x =

MATHEMATICS X

Hence all the zeroes of f (x) are

5,

3 a2
...(v)
4
Also, in right-angled triangle APD,

5 +

2 and 5 2 .
14. The given pair of equations may be
rewritten as

i.e,

x+y
=
2 xy
1
1
+ =
y
x

AD2 = AP2 + DP2

10
2 2x y
;
=
3
xy
3
2
1
10
4
;
+
=
y
x
3
3

Adding this last pair, we get

3
= 2
y
Substituting y =
of last pair, we get

y = 32
3
in the first equation
2

AP2 =

16.

AD2 =

27 a 2 a 2
3a2
a2
+
=
4
36
36
[From (iv) and (v)]

36AD2 = 28a2

9AD

9AD2 = 7AB2.

= 7a2

Hence proved.

BAD = 90 CAD
(QBAC = 90)
= 90 (90 ACD)
(QAD C = 90)

1
1
2
1
4
4
+
=

=
x
x
3
3
3
3
2
1
1
=2
x=
x
2
3
1
Hence, x =
, y =
is the required
2
2
solution.
15. Let the length of each side of the given
equilateral triangle be a, then
AB = BC = CA = a
...(i)
a
BD =
...(ii)
3

BAD = ACD
...(i)
BDA = ADC = 90
...(ii)
Using equations (i) and (ii) in DABD and
DCAD, we have
DABD ~ DCAD (AA rule of similarity)
AD
BD
AD
=
(Corresponding parts)
CD
BD . CD = AD .
2

OR
Given: AB P CD; AB = 2CD
To find:

1 = 2
[Q Alternate interior angles]
3 =4
DAOB ~ DCOD
(AA Criteria)
2CD
AB
AB
AOB )
\ arar ((%%COD
=
=
=
CD
CD
) CD
As

Draw AP BC to meet BC at P. P will be


the mid-point of BC, that is
a
BP =
...(iii)
2
a a a
DP = BP BD = =
...(iv)
2 3 6
[Using (ii) and (iii)]
Now, in right-angled triangle APB,
a2
AP2 = AB2 BP2
AP2 = a2
4

PRACTICE PAPER1

ar (%AOB )
ar (%COD )

(Q AB = 2CD)
4
= .
1

101

cos sin
1
=
cos + sin
1+

17.

20. The given data is

C.I. 0-5

cos R sin R

cos R cos R
1 3

cos R sin R
1 3

cos R cos R

1 tan R
1
=
1 tan R
1+

tan

q =

\q = 60.

(1)2 (2 2)2 =

15

30

Q Mode = l +

AC =

10

80

40

20

From the table, maximum occuring


frequency is 80. So, modal class is 15-20.

BC
18. In ABC, tan A =
=
AB
2 2

5-10 10-15 15-20 20-25 25-30 30-35

1 8 = 3

f1  f 0
2 f1  f 0  f 2

Here, l = 15, f1 = 80, f0 = 30, f2 = 40, h = 5

\ Mode = 15 + 16080 30 30 40 5

250
= 17.78
90
Hence, modal size is 17.78 hectares.

= 15 +

OR

1
2 2
sin A = , cos A =
,
3
3

1
2 2
, cos C =
3
3
Now, sin A. cos C + cos A. sin C
sin C =

1
1
2 2
2 2
+

3
3
3
3
1
8
= + = 1.
9
9
=

19. True,
LHS =
=

cos 80
+ cos 59 cosec 31
sin 10

cos(90 10)
sin 10
+ cos 59 cosec (90 59)

sin 10
+ cos 59 sec 59
sin 10

=1+

cos 59
=1+1=2
cos 59

Hence, the given equation is valid.


102

Class
interval

Frequency
(f)

Cumulative
Frequency (cf )

0-8
8-16
16-24
24-32
32-40
40-48

8
10
16
24
15
7

8
18
34
58
73
80

N = 80
N
80
= 40
=
2
2
In the cumulative frequency column, 58 is
just greater than 40.

For median class,

So, 24-32 is the median class.


N
Here, l = 24, cf = 34, f = 24,
= 40, h = 8
2
Using formula:
N

cf

Median = l +
f h

40 34
= 24 +
8 = 26
24

Hence, median of the given distribution is 26.

MATHEMATICS X

Dividing by 4, we get

SECTION D
21. Let

f (x) = 3 2 x2 + 13x + 6 2
= 3 2 x2 + 9x + 4x + 6 2
(Spliting middle term)

1
1
2
+
=
y
x
24
Use equation (i) equation (ii),
1
1
2
4

y
y
24
15

= 3x( 2 x + 3) + 2 2 ( 2 x + 3)
= (3x + 2 2 ) ( 2 x + 3)
To find the zeroes of f (x), we have
3x + 2 2 = 0 or

2x + 3 = 0

2 2
3

x =
or
3
2
\ Zeroes of the given polynomial are
2 2
3
and
.
3
2
Now,sum of zeroes =

23 2 +

3
2

13
3 2

Coefficient of x
=
Coefficient of x 2

Product of zeroes

Constant term
3
2 2
6 2

=
=
.
Coefficient of x 2
2
3
3 2

Hence proved.
22. (i) Let speed of the train be x km/hr and
that of the bus be y km/hr.

Distance
= Time
Speed
Case I. According to question, we get

300 60
60
+
=4
y
x

60
15
+
=1
y
x

240
60
+
=4
y
x

4
1
1
+
=
...(i)
y
x
15

Case II. According to the given conditions,


we get
100 300 100
10
x 
= 4 +
y
60

100
200
25
+
=
x
y
6

PRACTICE PAPER1

... (ii)

85
3
2
=
=
y
120
120
120 2
y=
= 80 km/hr
3
Put y = 80 in equation (i), we get

1x =

1
1 43 1

=
=
60
60
15 20
x = 60 km/hr.
Hence, speed of the train = 60 km/hr
and
speed of the bus = 80 km/hr.
1
4
1
+
=
x 80
15

(ii) Solution of system of linear equations in


two variables.
(iii) By opting for public transport it depicts
that she is a responsibile citizen, so her
responsibility and rationality have been
depicted here.
OR
(i) Let l = length of the rectangle
b = breadth of the rectangle
According to question,
(l + 7)(b 3) = lb
(i)
(l 7)(b + 5) = lb
(ii)
From equation (i),
lb + 7b 3l 21 = lb
7b 3l = 21
(iii)
From equation (ii),
lb 7b + 5l 35 = lb
7b + 5l = 35
(iv)
Adding equations (iii) and (iv), we get
2l = 56
l = 28 m
Putting the value of l in equation (iii), we
get
b = 15 m.
l = 28 m, b = 15 m.
(ii) Solution of system of linear equations
in two variables.
(iii) Love for environment and human beings.

4
1
8
+ =
x
y 6

103

23. Table for values of x and y as regarding


equation 3x + y 5 = 0 is
x
y

0
5

1
2

Similarly table for equation 2x y 5 = 0 is


x
y

0
5

1
3

Let us draw the graph of lines using the


tables obtained above.

Comparing equations (i) and (ii), we get


1
1
AP MQ =
AQ NP
2
2
AP
NP
=
...(iii)
AQ
MQ
1
Further, ar( BPQ) =
PB MQ ...(iv)
2
1
QC NP ...(v)
And
ar( CQP) =
2
But triangles BPQ and CQP are on the
same base PQ and between the same
parallels PQ and BC, so their areas must
be equal.
i.e.,
ar( BPQ) = ar( CQP)

D
D

1
1
PB MQ =
QC NP
2
2
[Using equations (iv) and (v)]

PB
NP
=
QC
MQ

...(vi)

From, equations (iii) and (vi), we get


AP
PB
=
AQ
QC

The lines intersect y-axis at (0, 5) and (0, 5).


24. We are given a triangle ABC in which a
line PQ parallel to BC is drawn to intersect
the sides AB and AC at P and Q respectively.

AP AQ
=
QC
PB
Join PC and QB. Draw QM AB and PN
AC.

We need to prove ,

AP
AQ
=
.
QC
PB
Hence proved.
25. We are given two triangles ABC and PQR
such that ABC ~ PQR.
Draw perpendiculars AD and PM on BC
and QR respectively.

We need to prove

Now, area of a triangle


1
=
base height
2
1
ar( APQ) =
AP MQ
...(i)
2
1
Also ar( APQ)=
AQ NP
...(ii)
2

104

D
D

AD 2
ar(ABC)
=
ar(PQR)
PM 2
In ABD and PQM,
ADB = PMQ = 90
ABD = PQM (
ABC ~ PQR)
ABD ~ PQM
(AA criterion of similarity)
AB
AD
=
...(i) (Corresponding sides)
PQ
PM

\ D

...D

MATHEMATICS X

We know that the ratio of areas of two


similar triangles is equal to ratio of squares
of their corresponding sides

AB 2
ar(ABC)
=
...(ii)
ar(PQR)
PQ 2
From equations (i) and (ii), we have

ar(ABC)
AD 2
=
ar(PQR)
PM 2

Hence proved.

q + cos q = 3 (Given)
sin q + cos q + 2 sin q cos q = 3
(Squaring both the sides)

1 + 2 sin q cos q = 3

sin q cos q = 1
...(i)
sin R cos R
Now, tan q + cot q =

cos R sin R
2

sin 2 R cos 2 R
1
=
=
[Using (i)]

1
sin R cos R
= 1.
Hence proved.
27. Let us take LHS of the given identity.
tan 2 R
cos ec 2 R

tan 2 R 1

sin R
cos 2 R

sin 2 R
1
cos 2 R

sin 2 R
sin 2 R cos2 R

sin 2 R cos 2 R

sin 2 R cos 2 R
sin 2 R.cos 2 R

cos 2 R
sin 2 R cos2 R

PRACTICE PAPER1

...

4 m n = 4 (tan R sin R)(tan R  sin R)


= 4 tan 2 R  sin 2 R
[Q (A + B) (A B) = A2 B2]
=4

sin 2 R
 sin 2 R
cos 2 R

1
cos 2 R

1

= 4 sin sec2 1
= 4 sin

q tan q

From (i) and (ii),

1
sin 2 R

1
1

cos2 R
sin 2 R

sin 2 R cos 2 R
cos 2 R

1
sin 2 R

sin 2 R
cos 2 R

= 4 sin

1
sin R cos 2 R
sin R cos R
[ sin2 + cos2 = 1]
= RHS.
Hence proved.
OR
tan A = n tan B
n
cot B =
and sin A = m sin B
tan A

A
sinm A tann A = 1 m sinn cos
=1
A

m 1 = (n 1) cos A
m 1
= cos A.Hence proved

n 1
28. m n = (tan q + sin q) (tan q sin q)
= (tan q + sin q + tan q sin q)
(tan q + sin q tan q + sin q)
[Q A B = (A + B) (A B)]
= 2 tan q . 2 sin q
= 4sin q tan q
...(i)

sec 2 R cos ec 2 R
2

m
1
cosec B =
sin A
sin A
m
... cosec2 B cot2 B = 1
sin B =

26. sin

m2

.... (ii)
n2

= 4 mn .
29. Let us prepare the cumulative frequency
table by more than method as given
below:
Production
yeild
(in kg/ha)

fi

Production
yeild (in kg/ha)
more than
or equal to

50-55
55-60
60-65
65-70
70-75
75-80

2
6
8
14
15
5

50
55
60
65
70
75

Total

50

cf

Point

50
48
42
34
20
5

(50, 50)
(55, 48)
(60, 42)
(65, 34)
(70, 20)
(75, 5)

105

We plot the points mentioning in the table


such that lower class limits are on the
x-axis and the cumulative frequencies are
on the y-axis. By joining these points by
free hand smooth curve, We obtain more
than type ogive as shown in the adjoining
graph.

N = 60

x + y + 45 = 60
x + y = 15

... (i)

N
cf
Median = l + 2

q h

30 5 + x

28.5 = 20 +
10

20
On simplifying, x = 8

...(ii)

From equations (i) and (ii), we have


x = 8 and y = 7.
31. We will use the step-deviation method.
Marks

To obtain median from graph:


Draw a line parallel to x-axis passing

50
through y =
= 25. This line meets the
2
ogive at (68.2, 25).

\ Median = 68.2.
30. Given that median is 28.5. It lies in the class
interval 20-30, so 20-30 is the median class.
Let us prepare frequency distribution table.
C. I.

cf

0-10
10-20
20-30
30-40
40-50
50-60

5
x
20
15
y
5
x + y + 45

5
5+x
25 + x
40 + x
40 + x + y
45 + x + y

No. of

Class-

di

ui

fiui

d
= xi 45 = i
10

(C.I.)

students
(fi)

mark
xi

0-10

40

20

10-20

15

30

12

20-30

25

20

16

30-40

12

35

10

12

40-50

16

45

50-60

15

55

10

15

60-70

10

65

20

20

70-80

75

30

24

80-90

85

40

20

95

50

10

90-100

Sfiui

Sfi = 85

= 29

Let a = 45. Here h = 10


Mean = a +

fi ui

s
fi

= 45 +

29
10
85

= 45 + 3.41
= 48.41.

106

MATHEMATICS X

3 6a = 5a 5 and 9b = 10b 5

a = 8 and b = 5.
9. Yes, because PBC and PDE are similar

Practice Paper-2
SECTIONA
1. (C) ...

343
3

2 5 7

343
3

2 5 343
= 0.001.

1
1000

by SAS rule. As

PC 1
BP
=
= and BPC =
PE 2
DP

DPE.

2. (D) The condition for the line parallel is:

a1
c
b
3
2c 2
= 1 1
=

a2
b2 c2
2
5
1

15
.
4
ar(ABC) BC 2
3. As
=
ar(DEF)
EF 2
(Result on areas of similar triangles)

c =

OR
In AQO and BPO,
QAO = PBO

AOQ = BOP
(Vertically opposite angles)

4.

54
3
= 2
ar(DEF) 4
o

sin 70o
sin 70

E
2

sin 70

So, by AA rule of similarity,

cos (90o 70o )

ar(DEF) = 96 cm2.
2 cos k
=
2
cos

+2=

AQO ~ BPO

k
2

3=

k
2

k = 6.

SECTION B
5. False, because 3 or 2 3 is an irrational
and sum or difference of a rational number
and an irrational number is an irrational
number.
6. No, because prime factorisation of any
number of type 2n can not have 5 as one of
its prime factor.
7. False, since the discriminant is zero for k =
1
.
2
8. The condition for infinitely many solutions
a1
b
c
2b 1
3
( a + 1)
is a = 1 = 1 . =
=
2
c
b
1 2a
3b
5
2

(Each 90)

( a + 1)
2b 1
3
3
=
and =
1 2a
3b
5
5

PRACTICE PAPER2

AO
AQ
=
BO
BP

10 9
10
AQ
=
AQ =
6
6
9
AQ = 15 cm.

10. The empirical relationship among the three


measures of central tendency is:
Mode = 3 Median 2 Mean
= 3 55 2 50
= 165 100 = 65.
SECTION C
11. To prove that the pair of numbers (847,
2160) is coprime by using Euclids
algorithm, we have to prove that the highest
common factor of the pair is 1.
Since 2160 > 847

2160 = 847 2 + 466

Since the remainder 466 0.

847 = 466 1 + 381

Since the new remainder 381 0.

107


466 = 381 1 + 85
Since the new remainder 85 0.

381 = 85 4 + 41
Since the new remainder 41 0.

85 = 41 2 + 3
Since the new remainder 3 0.

41 = 3 13 + 2
Since the new remainder 2 0.

3 =21+1
Since the new remainder 1 0.

2 =12+0
Since, the remainder has now become zero,
the divisor at this stage is 1, the HCF of
847 and 2160 is 1.
OR
Let us assume, to the contrary, that 3 +
2 5 is rational.
So we can find coprimes a and b such that
a
3+2 5 =
b
a  3b
Rearranging,
5 =
2b
a and b are integers a 3b is an integer
a  3b

is rational number
2b

5 should be rational. But we


know that

is irrational. So our

assumption that 3 + 2 5 is rational is


wrong.
Hence 3 + 2 2 is irrational.
12. Let x be any positive integer.
Then it is either of the form 3q or 3q + 1 or 3q
+ 2.
Case I.
x = 3q
Cubing both sides, we get
x3 = (3q)3 = 27q 3 = 9m
(i)
3
m = 3q
Case II.
x = 3q + 1
Cubing both sides, we get
x3 = (3q + 1)3
= 27q3 + 1 + 3 (3q + 1) 3q
= 27q3 + 27q2 + 9q + 1
= 9q (3q2 + 3q + 1) + 1
108

= 9m + 1
m =
Case III.

q(3q2 +

(ii)
3q + 1).

x = 3q + 2

Cubing both sides, we get


x3 = (3q + 2)3
= 27q3 + 8 + 3 (3q + 2) 6q
= 27q3 + 54q2 + 36q + 8
= 9q (3q2 + 6q + 4) + 8
= 9m + 8;

(iii)

(3q2 +

m =q
6q + 4).
Thus, from equations (i), (ii) and (iii), it is
clear that cube of any positive integer is
either of the form 9m, or 9m + 1 or 9m + 8.
13.

distance
speed
Time taken by Abhay to cover one complete
We have time =

360
= 30 hours
12
Time taken by Ravi to cover one complete
round =

360
= 24 hours
15
Abhay and Ravi reach the starting point
respectively after 30 hours and 24 hours, and
their respective multiples. Therefore, they will
meet again at the starting point after the time
given by least common multiple of 30 hours
and 24 hours Let us determine the LCM of 30
hours and 24 hours.
30 = 2 3 5,
24 = 23 3 LCM = 23 3 5 = 120
Hence, the required time is 120 hours.

round =

14. Let and be the zeroes of 6x2 + x + k.

2 + 2 = ( + )2 2
2

1 k
1
k
= 2 =

36 3
6
6
But it is given that 2 + 2 =

25
36

k 24
25
1
k
=

=
k = 2.
3
36
36
36
3

MATHEMATICS X

OR
Since a = 2 is a zero of a3 3a2 10a + 24,
therefore a3 3a2 10a + 24 is divisible by
a 2. Further the obtained quotient will
provide the other two zeroes.

Add equations (i) and (ii) to get


PR2 + QR2 = 4PS2 2SM(PS SQ)

PQ
= 4

2
PR2 + QR2 = PQ2.

(Q PS = SQ)
Hence proved.

OR

AB
AC
AD
=
=
and
PQ
PR
PM
AD, PM are medians.

Given:
a2 a 12 = (a 4) (a + 3)
For other zeroes, put a 4 = 0 and a + 3 = 0
a = 3, 4
Thus, the other two zeroes are 3 and 4.
15. In right-angled triangle PQS,

To show:

ABC ~ PQR.

PS2 = PQ2 + QS2 QS2 = PS2 PQ2

QR 2
= PS2 PQ2
4

QR2 = 4PS2 4PQ2

(i)

In right-angled triangle PQR,


PR2 = PQ2 + QR2
=

PQ2

4PS2

4PQ2

PR2 = 4PS2 3PQ2.


16. Draw RM PQ.
In PRS, PSR > 90

[Using (i)]

Hence proved.

=
+
+ 2PS.SM ...(i)
[Using result on obtuse-angled triangle]
PR2

PS2

RS2

In QRS, QSR < 90

QR2 = RS2 + SQ2 2SQ.SM ...(ii)


[Using result on acute-angled triangle]
PRACTICE PAPER2

Construction: Extend AD up to D' such that


AD = DD' and extend PM up to M' such
that PM = MM'
Join BD', D'C; QM', M' R.
Proof: As AD = DD' and BD = DC and PM
= MM' and QM = MR
ABD' C and PQM' R are Pgm
AB = D' C and PQ = RM'
{Q Opposite sides of a Pgm are equal}
As it is given that:

AB
AC
AD
=
=
PQ
PR
PM

D C AC 2AD AD



RM PR
2PM PM
ACD' ~ PRM'
(SSS criteria)

1 = 2
(CPCT)

109

Similarly 3 = 4 1 + 3 = 2 + 4

BAC = QPR
Now : In ABC and PQR
AC
AB
=
PR
PQ

and

(i)

BAC = QPR
ABC ~ PQR

(ii)
(SAS)
Hence proved.

3
17.
sin =
4
Q sin2 + cos2 = 1

cos2 = 1 sin2

3
cos2 = 1
4

... (i) Given

cosec2 R cot 2 R

(tan 1) (tan 2 + 1 + tan )


tan (tan 1)
[Q a3 b3 = (a b) (a2 + b2 + ab]

= tan +

sin 2 + cos 2
+1
sin cos
= sec . cosec + 1

7
1
7
=
=
[Using (ii)]
16
9
3
1
7

= RHS.
Hence proved.
20. Let us convert the more than type
distribution to the normal distribution.

Hence proved.

3
1
, cos 60 = ,
2
2
2
sec 30 =
, cosec 30 = 2
3
3 + sin 2 60 + cosec2 30

We have sin 60 =

5 + cos2 60 + sec 2 30
2

3
2
3 +
+ (2 )
2
2

1 2
5+ +

2 3

3
+4
4
=
1 4
5+ +
4 3
3+

110

36 + 9 + 48
93
=
.
60 + 3 + 16
79

Marks

No. of students

0-20

20-40

40-60

20

60-80

15

80-100

We observe from the table that the value 20


is the maximum frequency. So, the modal
class is 40-60.

f1 f0
Now, mode = l +
h
2
f

f
1
0
2
Here,

l = 40, f1 = 20, f0 = 7, f2 = 15,


h = 20

Multiplying Num. and Deno. by 12,


=

1
sin cos
+1=
+
+1
tan
cos sin

sec 1
sec R 1
[... cosec2 cot2 = 1]
2

= RHS.
18.

tan 3 R  1
tan R tan R  1

[Using (i)]

7
16
sec2 =
... (ii)
16
7
Let us take LHS of the given equation.

cos2 =

LHS =

19. Consider, left hand side of the given


equation,
1
tan
tan
tan
cot
+
+
=
1
1 tan
1 cot 1 tan
1
tan
2
tan
1
=
+
tan 1 tan (1 tan )

20 7
Mode = 40 +
20
40 7 15

MATHEMATICS X

x2 x 7

260
18
= 40 + 14.44

= 40 +

x 2 1 x 4 x 3 8 x 2 ax b
x4 x2

 

= 54.44 marks.

x 3 7 x 2 ax b
x3 x

SECTION D
21. To solve a system of equations graphically
we need atleast two solutions of each
equation.
Two solutions of the equation 2x y = 2 are
given in the following table:
x

Two solutions of the equation 4x y = 8 are


given in the following table:
x

Let us draw the graph of the two given


equations.

 

7 x 2 ( a  1)x b
7 x2 7

( a  1)x (b  7)

Since, x4 + x3 + 8x2 + ax + b is divisible by


x2 + 1, therefore remainder = 0
i.e.,

(a 1)x + (b 7) = 0

or

(a 1)x + (b 7) = 0.x + 0

Equating the corresponding terms, we have


a 1 = 0 and b 7 = 0
i.e.,

a = 1 and b = 7.

(ii) Common good, Social responsibility.


23. Let the speed of rowing in still water and
the speed of the current be u km/hr and v
km/hr respectively.
The speed of rowing in downstream
= (u + v) km/hr.
The speed of rowing upstream
= (u v) km/hr.
Using the formula:

X'

Distance
= Time
Speed

According to first condition of the question,

Y'

From the graph, it is clear that the two lines


intersect each other at the point (3, 4).
Hence, the solution is x = 3, y = 4.
22. (i) First we divide x4 + x3 + 8x2 + ax + b by
x2 + 1 as follows:

PRACTICE PAPER2

12
18
+
=3
... (i)
u+v u v
According to second condition of the
question,

40
36
+
=8
... (ii)
u+v u v
1
1
Let us put,
= x and
= y such
u+v
uv
that equations (i) and (ii) reduce to
18x + 12y = 3
... (iii)
And 36x + 40y = 8
... (iv)

111

Equations (iii) and (iv) form a pair of linear


equations.
Multiply equation (iii) by 2 and subtract
the result from equation (iv) to get
1
16y = 2 y =
8
1
Substitute y =
in equation (iv) to get
8
1
36x = 8 5 x =
12
1
1
Q x=
u + v = 12 Q y =
u+v
uv

uv =8
This last system gives u = 10 and v = 2
Hence, the speed of the rowing in still water
= 10 km/hr and the speed of the current
= 2 km/hr.
ar(AXY) = ar(BXYC)

24.

2.ar(AXY) = ar(BXYC) + ar(AXY)


= ar(ABC)
A

As

BC
AB
=
and B = Q
QR
PQ

AB
2BD
=
and B = Q
PQ
2QM

BD
AB
=
and B = Q
QM
PQ

AB
AD

PQ
PM
26. m = cosec sin
(Given)
1 sin 2
1
m =
sin =
sin
sin
cos2
m =
... (i) (Q 1 sin2 = cos2 )
sin
cos 4
m2 =
... (ii) (Squaring)
sin 2
Further, n = sec cos
(Given)
1 cos2
1
cos n =
n =
cos
cos
ABD ~ PQM =

sin 4
sin 2
... (iii) n2=
... (iv)
cos
cos2
Multiplying equations (ii) and (iii), we get

n =

ABC ~ PQR

2
ar (ABC)
=
ar ( AXY)
1
ABC ~ AXY
2
2
ar ( ABC)
AB

= ar ( AXY) =
1
AX

1
AX
AB
2

=
2
AB
1
AX
1
1
AB  BX
BX

=
1
=
2
2
AB
AB
2 1
BX

=
.
AB
2
25. Hint:

m 2n =

cos 4
sin 2

sin 2
= cos3
cos

(m2n)2/3 = cos2
... (v)
Multiplying equations (i) and (iv), we get
n2m =

sin 4
cos2

cos2
= sin3
sin

(n2m)2/3 = sin2

... (vi)

Adding equations (v) and (vi), we get


(m2n)2/3 + (n2m)2/3 = cos2 + sin2
i.e., (m2n)2/3 + (n2m)2/3 = 1. Hence proved.
27. Given equations are: sin + cos = p ... (i)
and sec + cosec = q
... (ii)
Squaring both the sides of equation (i),
we get sin2 + cos2 + 2sin cos = p2
Subtract unity from both the sides to get
p2 1 = 2sin cos

112

... (iii)

MATHEMATICS X

Equation (ii) can be written as


1
1
q =
+
cos
sin
sin R cos R

q =
sin R cos R

... (iv)

From equations (iii) and (iv), we get


q (p2 1) =

sin R cos R
2sin cos
sin R cos R

q (p2 1) = 2(sin + cos )


q (p2 1) = 2p.
Hence proved.
OR
a
1
=
x
sin
1
b
and y = b tan
=
tan

As x = a sin

LHS =
=

a2
x

b2
y

1
sin 2

1 cos 2

1
2

sin

1
tan 2

cos2
sin 2

sin 2

B + C
180 A
1
=
= 90 A
2
2
2
A
B+C

A
sin
= sin 90 = cos
2
2
2

B
C
+

A
sin2
(i)
= cos2
2
2
A
B+C
Now, sin2
+ sin2
2
2
A
A
= sin2
+ cos2
2
2
[Using (i)]
= 1.
Hence Proved.
29. The cumulative frequency table for the
given data is given below:

PRACTICE PAPER2

No. of students
(f)

Cumulative
frequency (cf )

0-10
10-20
20-30
30-40
40-50
50-60

10
f1
25
30
f2
10

10
10 + f1
35 + f1
65 + f1
65 + f1 + f2
72 + f1 + f2

N = 75 + f1 + f2
Clearly,
N = 75 + f1 + f2
But
N = 100

f1 + f2 = 25

... (i)

N
= 50.
2
The median is 32 which lies in the class
30-40.
So, l = 30, f = 30, cf = 35 + f1, h = 10.
Using the formula:
N

 cf
2

Median = l +
h
f

sin 2
sin
= 1 = RHS
Hence proved.
28. As A + B + C = 180
B + C = 180 A
2

Marks
(C.I.)

32 = 30 +

50  35  f1

30

10

2
15  f1
=
75 5f1 = 30
10
30
75  30

f1 =
f1 = 9
5
Substituting f1 = 9 in equation (i), we get
9 + f2 = 25 f2 = 16

Hence, f1 = 9 and f2 = 16.


30. Let us prepare the cumulative frequency
distribution from the given distribution.
Class
interval (C.I.)

Frequency
( f)

Cumulative
frequency (c f )

0-10
10-20
20-30
30-40
40-50
50-60

5
x
20
14
y
8
N = 47 + x + y

5
5+x
25 + x
39 + x
39 + x + y
47 + x + y

113

Since 27 is the median, therefore, 20-30 is


the median class.

l = 20, cf = 5 + x, f = 20, h = 10

Given: N = 68
Using the formula:
N
 cf

Median = l + 2
f

68

5x
2

27 = 20 +
10

20

29 x
29 x
2
= (27 20) 2 x = 15
From the table, N = 47 + x + y
But
N = 68
(Given)
\ x + y + 47 = 68
15 + y + 47 = 68 (Substituting x = 15)
y = 6. Thus, x = 15, y = 6.

27 = 20 +

f u
14
Mean = A + h i i = 60 + 20
50
fi
= 60 + 5.6 = 65.6
Hence, the required arithmetic mean is 65.6.
31. Let us convert the given data into less than
type distribution.
Class
interval

Lifetimes
(in hrs.)

cf

0-20

10

less than 20

10

20-40

35

less than 40

45

40-60

52

less than 60

97

60-80

61

less than 80

158

80-100

38

less than 100

196

100-120

29

less than 120

225

OR
Class

Mid-

Freq-

ui =

interval

values

uency

(xi)

(fi)

10-30

20

40
 2
20

10

30-50

40

20
 1
20

50-70

60 = A

12

0
0
20

70-90

80

20

20
1
20

20

90-110

100

40
2
20

110-130

120

60
3
20

xi A

f iu i

fi = 50

fiui = 14

Let assumed mean be A = 60


Here, h = 20
114

MATHEMATICS X

We mark the upper class limits along the


x-axis with a suitable scale and the
cumulative frequencies along the y-axis
with a suitable scale. For this, we plot the
points A(20, 10), B(40, 45), C(60, 97), D(80,
158), E(100, 196) and F(120, 225) on a graph
paper. These points are joined by a free
hand smooth curve to obtain a less than
type ogive as shown in the given figure.
Practice Paper-3
SECTIONA
1. (D) There are infinitely many real numbers
of both types rational and irrational
between

3 and

5.

2. (B) For infinite number of solutions:

5
3
=
, i.e., k = 6.
10
k
DABC ~ DDEF

3.

B = E =180 (40 + 65) = 75.


4.

AC2 = BC2 AB2


AC2 = 2 1 = 1
AC = 1

BC
2
=
AB
1
= 2.

cosec C =

SECTION B
5. 8 can be rewritten as 23n. Clearly, the prime
factor of 8n is only 2. To end with the digit 0,
one of the prime factors of 8n must be 5.
Hence, 8n cannot end with the digit zero for
any n N.
n

PRACTICE PAPER3

6. LCM=

First number Second number


HCF

96 404
= 24 404 = 9696.
4
7. True, because we find the remainder zero
when 3x4 + 5x3 7x2 + 2x + 2 is divided by
x2 + 3x + 1.
=

8. Infinite number of solutions because the


system obeys the following condition:

3
3
a1
b
1
c
= 1 = 1 , i.e.,
=
=
.
9
9
3
b2
a2
c2
9. In DAOD and DCOB,
DO 1
AO
=
=
OB
2
OC

AOD =COB
DAOD ~ DCOB
and

AD
1
1
4
=
=
BC
BC 2
2

BC = 8 cm.
OR
Let the height of the tower be h metres
DABC ~ DPQR

8
BC
AB
=
12 =
40
PQ
QR
h
12 40

h =
= 60 metres.
8
10. Since the maximum frequency is 8, so
modal class is 4-8.
\ l = 4, f1 = 8, f0 = 4, f2 = 5, h = 4

f1 f0
Now, mode = l +
h
2 f1 f0 f2
84
= 4 +
4 = 4 + 2.29
16 4 5
= 6.29.

115

SECTION C
11. Let us assume on the contrary that 2 is
a rational number. Then 2 can be written
a
as 2 = , where a and b are coprime
b
and b 0.
a2
(Squaring)
b2

a2 = 2b2
... (i)
2
a is divisible by 2
...(ii)
a is divisible by 2
...(iii)
[If a prime (here 2) divides d2, then the
same prime divides d, where d is a positive
integer.]

a = 2c a2 = 4c2
... (iv)

2=

From (i) and (iv), we get


4c2 = 2b2 b2 = 2c2
b2 is divisible by 2
b is divisible by 2
....(v)
From results (ii) and (v), we have a and b
both are divisible by 2.
But this contradict the fact that a and b are
coprime. This contradiction has arisen
because of our incorrect assumption that
2 is a rational number. Thus, we conclude
that 2 is an irrational number.
OR
(i) The given fraction can be written as
43
43 5
=
= 0.0215
3
2 5
10 4
Hence, the given number terminates after
four places of decimal.
(ii) The given fraction can be written as
4

359
2 4 359
5744
= 5
=
= 0.05744
5
25
2 55
100000
Hence, the given number terminates after
five places of decimal.
12. (i) LCM of 2, 3, 4, 5, 6 = 60
\ Required number = 60p + 1; p is
positive integer
= (7 8 + 4)p + 1
= (7 8p) + (4p + 1)
116

Now, this number is to be divisible by 7.


Since 7 8p is always divisible by 7; so,
we must choose the least value of p
which will make 4p + 1 divisible by 7.
Putting p = 1, 2, 3, 4, 5 etc. in succession
we find that p = 5.
\ Required number 60p + 1
= 60 5 + 1 = 301
(ii) LCM of two real numbers.
(iii) Kindness and love towards society.
13. p(t) = t2 15
To obtain zeroes of p(t), put p(t) = 0
i.e., t2 15 = 0
t2

15

=0

(t +

15

)(t

15 = 0

t = 15 , 15

So, zeroes of p(t) are 15 and

15

Sum of zeroes = 15 + 15 = 0
=

0
Coefficient of t
=
1
Coefficient of t 2

Product of zeroes = 15 15 = 15

Constant term
15
=
.
1
Coefficient of t
Hence verified.
14. The given system of equations can be
rewritten as
4x + 3y 48 = 0
40x 6y 192 = 0
Applying the method of cross multiplication to solve the system.
y
1
x

=
=
768 + 1920 24 120
576 288
y
1
x

=
=
1152 144
864
=

1152
864
and y =
144
144
x = 6 and y = 8.

x =

15. We are given a square of side length a.


Then length of its diagonal will be a 2 .
We know that area of an equilateral
3 2
triangle of side length x is
x .
4
MATHEMATICS X

\ Area of the equilateral triangle described


of the side of the square.
4
3 2
A
a ... (i) a2 =
3 side
4
And area of the equilateral triangle
described on the diagonal of the square

Aside =

Adiagonal =

3
3 2
a
(a 2 )2 =
4
2

3 4

A side [Using (i)]



2
3

1
Aside = Adiagonal . Hence proved.
2
16. In the figure drawn,
AB || DC and DAED ~
DBEC.
D ADC and D BDC
both are on the same base DC and lie
between same parallels AB and DC.
So, ar(DADC) = ar(DBDC)
ar(DAED) + ar(DDEC)
= ar(DBEC) + ar(DDEC)

ar(DAED) = ar(DBEC)
(i)
ar(%AED) (AD)2
=
Now,
ar(%BEC)
(BC)2
(Q DAED ~ DBEC)

1 =

(AD )2
[From (i)]
(BC )2

AD = BC.
17. Given expression
1 1
= 2
2 2

Hence proved.

{( 3 ) + (1) }
2

2
+ 3

3
1
4
1
= 2 + (3 + 1) + 3
16 16
3
1
1
= 4+4= .
4
4
18. Consider left hand side of the given
equation.
LHS = (cosec A sin A) (sec A cos A)
1

sin A
=
sin A

PRACTICE PAPER3

cos A

cos
A

1 sin 2 A 1 cos2 A
.
sin A
cos A

cos2 A sin 2 A
.
= sin A cos A
sin A cos A
Also, taking right hand side,

1
1
=
sin A cos A
tan A + cot A
+
cos A sin A
sin A cos A
=
= sin A cos A
sin 2 A + cos2 A
Hence, LHS = RHS.
OR
Draw D ABC with
AB = BC = AC = a (say)
RHS =

Adiagonal =

Draw AD ^ BC
\ BAD = DAC = q = 30
and BD = DC = a/2

\ sin q=

BD a/2
=
AB
a

1
1
sin 30= .
2
2

19. Given equations are: sin q + cos q = p ... (i)


and sec q + cosec q = q
... (ii)
Squaring both the sides of equation (i),
we get sin2 q + cos2 q+ 2sin q cos q = p2
Subtract unity from both the sides to get
p2 1 = 2sin q cos q
... (iii)
Equation (ii) can be written as
1
1
q =
+
cos
sin
sin R cos R

q =
... (iv)
sin R cos R
From equations (iii) and (iv), we get
sin R cos R
2sin q cos q
sin R cos R
q (p2 1) = 2(sin q + cos q)
q (p2 1) = 2p.
Hence proved.

q (p2 1) =

117

20. Let us use step-deviation method to obtain


the mean.
C.I.

fi

xi

di

ui =

di
h

d i ui

Similarly, y3 = cosec A sin A


2

= xi a
0-20
20-40
40-60
60-80
80-100

17
p
32
24
19

10
30
50
70
90

40
20
0
20
40

Sfi
= 92+p

2
3
x = sin A
cos A

=
2
1
0
1
2

34
p
0
24
38

Sfi ui
= 28 p

Here a = 50, h = 20
fi ui
Mean = a +
h
fi
28 p
50 = 50 +
92 + p

28 p = 0 \ p = 28.
OR
Since mode = 36, which lies in the class
interval 30-40, so the modal class is 30-40.

1 sin A
1
sin A =
sin A
sin A
1

2
3
y = cos A
sin A

LHS = x2 y2 (x2 + y2) = x4y2 + x2y4


1

sin 2 A 3

=
cos A

16  f
36 = 30 +
10
32  f  12
16  f
6
=
10
20  f

120 6f = 160 10 f

4f = 40
f = 10.
SECTION D
21. We have
x3 = sec A cos A
1
1 cos2 A
cos A =
=
cos A
cos A

118

2
cos A 3 +
sin A

2
sin A 3
cos A

2
cos A 3
sin A

2
2

3
3
= sin A cos A
cos A sin A

sin 2 A 3 cos 2 A 3
+
cos A sin A

\ f1 = 16, f0 = f, f2 = 12, l = 30 and h = 10.


f1  f0
Now, mode = l +
h
2 f1  f0  f2

sin 8 A cos 4 A 3

+
=
4
2
cos A sin A

) (

= sin 6 A 3 + cos6 A
= 1 = RHS.

sin 4 A cos8 A 3

2
4
cos A sin A

)3 = sin2A + cos2 A

Hence proved.

x
.
y
On adding 1 to both the numerator and
x
4
the denominator of , it becomes
y
5
x +1
4
i.e.
,
= , i.e., 5x + 5 = 4y + 4
y +1
5
i.e., 5x 4y = 1
...(i)

22. Let the original fraction be

On subtracting 5 from both the numerator


and the denominator of

1
x
, it becomes
y
2

MATHEMATICS X

x5
1
= , i.e., 2x 10 = y 5
y5
2
i.e., 8x 4y = 20
...(ii)
Subtracting equation (i) from equation (ii),
we get
3x = 21 x = 7
Substituting x = 7 in equation (i), we get
5 7 4y = 1 y = 9
7
Hence, the required fraction is .
9
OR
Let incomes of X and Y be 8x and 7x
respectively; and their expenditures be 19y
and 16y respectively.
We know that:
Income Expenditure = Savings
... 8x 19y = 1250
(i)
and 7x 16y = 1250
(ii)
Comparing equations (i) and (ii), we have
8x 19y = 7x 16y or x = 3y (iii)
Substituting this value of x in equation (i),
we get 24y 19y = 1250

5y = 1250 y = 250
Substituting this value of y in equation (iii),
we get
...
x = 3 250 = 750.
Now,
8x = 8 750 = 6000
and
7x = 7 750 = 5250
hence Xs income is 6000 and Ys income is
5250.
23. Let us make the table for the values of x and
corresponding values of y to the equation
2x + y 8 = 0

i.e.,

x
y

2
4

4
0

Similarly, for the equation x y 1 = 0


x
y

4
3

Now, AM = 3 units and BC = 8 + 1 = 9 units.


1
BC AM
... ar(DABC) =
2
1
27
=
93=
sq. units
2
2
Hence, x = 3, y = 2; area = 13.5 sq.units.
24. Pythagoras Theorem:
In a right triangle, the square of the
hypotenuse is equal to the sum of the
squares of the other two sides.
Proof: We are given, a DABC in which A
= 90

3
2

Let us draw the graph.


From the graph, the lines intersect each
other at the point A(3, 2). Therefore, the
solution is x = 3, y = 2.
The lines intersect the y-axis at B(0, 8) and
C(0, 1).
PRACTICE PAPER3

To find the area of the shaded portion, that


is, DABC, draw perpendicular AM from A
on the y-axis to intersect it in M.

We need to prove BC2 = AB2 + AC2.


Draw AD ^ BC
(see figure)
In DABC and DDBA,
ABC = DBA
(Common)
119

So,

BAC = BDA
DABC ~ DDBA

(Each 90)

(A A criterion of similarity)

AB
BC
=
AB
BD
or
AB2 = BD . BC
...(i)
Similarly, DABC ~ DDAC
AC
BC
Therefore,
=
DC
AC
AB2 = DC . BC
...(ii)
Adding equations (i) and (ii), we get
BD . BC + DC . BC = AB2 + AC2
or (BD + DC) . BC = AB2 + AC2
or
BC . BC = AB2 + AC2
(Q BC = BD + DC)
or BC2 = AB2 + AC2.
Hence proved.
OR
Converse of Pythagoras Theorem: In a
triangle, if square of one side is equal to
the sum of the squares of the other two
sides, then the angle opposite the first side
is a right angle.
Therefore,

Proof: We are given a DABC in which


AC2 = AB2 + BC2
...(i)
We need to prove ABC = 90.
Let us construct a DPQR such that PQR =
90 and PQ = AB
...(ii)
QR = BC
...(iii)
Using Pythagoras Theorem in DPQR, we
have
PR2 = PQ2 + QR2

PR2 = AB2 + BC2


...(iv)
[Using equations (ii) and (iii)]
From equations (i) and (iv), we have
AC = PR
...(v)
Now, in DABC and DPQR,
AB = PQ
(From (ii)]
BC = QR
[From (iii)]
120

AC = PR
[(From (v)]
@ DPQR (SSS congruence)
\
= PQR
(CPCT)
But
= 90
(By construction)
\
= 90.
Hence proved.
25. Let the given parallelogram be ABCD
We need to prove that
AC2 + BD2 = AB2 + BC2 + CD2 + DA2
Let us draw perpendiculars DN on AB and
CM on AB produced as shown in figure.
So,

DABC
ABC
PQR
ABC

In DBMC and DAND,


BC = AD (Opposite sides of a ||gm)
BMC = AND
(Each 90)
CM = DN
(Distance between same parallels)
\ DBMC @ DAND
(RHS criterion)

BM = AN
...(i) (CPCT)
In right triangle ACM, AC2 = AM2 + CM2
= (AB + BM)2 + BC2 BM2
= AB2 + 2AB . BM + BM2 + BC2 BM2
= AB2 + BC2 + 2AB . BM
...(ii)
In right triangle BDN,
BD2 = BN2 + DN2
= (AB AN)2 + (AD2 AN2)
= AB2 2AB . AN + AN2 + AD2 AN2
BD2 = AB2 + DA2 2AB . AN

BD2 = CD2 + DA2 2AB . BM ...(iii)


[Using (i) and AB = CD]
Adding equations (ii) and (iii), we have
AC2 + BD2 = AB2 + BC2 + CD2 + DA2.
Hence proved.
26. Asa, b are zeros of p(x) = x2 px + q.
a + b = p;
ab = q

MATHEMATICS X

B 2 C2

C2 B 2
(B 2 C 2 )2  2B 2C 2
B 2C 2

\ Consider
=

=
=

B 4 C4
B 2C 2
(B 2 C 2 )2  2q 2
q

(B C)2  2BC  2q 2

q2

p 4 4q 2  4 p 2 q  2q 2
q2
p

q2

p 4 2q 2  4 p 2 q

q2

4p
2 = RHS.
q

sec2

or

( p 2  2 q )2  2 q 2

or 1 + tan2

=x+

Hence proved.

1
4x

q = x + 41x

... (i) (Given)


2

1
1
+
= x2 +
2
2
16 x

or

tan2

q = x2 +

or

tan2

q = x 41x

or

tan

(Squaring)

1
1

2
2
16x
2

q = + x 41x

... (ii)

Add equations (i) and (ii) to get


sec

q + tan q = x +

or x +

1
1
+x
4x
4x

1
1
1
x+
= 2x or
.
4x
4x
2x
Hence proved.

28. LHS = (1 + cot A cosec A)


(1 + tan A + sec A)

= 1

(sin A + cos A )2 12
sin A cos A
2

sin A + 2 sin A cos A + cos 2 A 1


sin A cos A

2 sin A cos A
= 2 = RHS. Hence proved.
sin A cos A
29. We prepare the cumulative frequency table
by less than method as given below:

Marks

Freq.

Marks
less than

cf

Point

0-10
10-20
20-30
30-40
40-50
50-60
60-70
70-80

4
10
16
22
20
18
8
2

10
20
30
40
50
60
70
80

04
14
30
52
72
90
98
100

(10, 4)
(20, 14)
(30, 30)
(40, 52)
(50, 72)
(60, 90)
(70, 98)
(80, 100)

sec

27.

cos A
1
sin A
1


1

sin A sin A
cos A cos A

sin A + cos A 1
cos A + sin A + 1

sin A
cos A

PRACTICE PAPER3

We take marks on the x-axis and


cumulative frequency on the y-axis and
then plot the points mentioned in the table.
On joining these points by free hand
smooth curve, we get less than ogive.
Further, we prepare the cumulative
frequency table by more than method as
given below:
Marks

Freq.

Marks
more than
or equal to

cf

Point

0-10
10-20
20-30
30-40
40-50
50-60
60-70
70-80

4
10
16
22
20
18
8
2

0
10
20
30
40
50
60
70

100
96
86
70
48
28
10
2

(0, 100)
(10, 96)
(20, 86)
(30, 70)
(40, 48)
(50, 28)
(60, 10)
(70, 2)

We will plot the points mentioned in this


table on the same graph. On joining these
points by free hand smooth curve, we get
more than ogive.
121

Mean: Mean =

fi
Q
S fi = 50 and Sfi xi = 7260
7260
\ Mean =
= ` 145.20.
50
N cf

Median: Median = l + 2
h
f

105
100
95

(0, 100)
(10, 96)

(80, 100)

(70, 98)

90

(60, 90)
(20, 86)

85

less than ogive

80
75
70

(50, 72)

(30, 70)

N
= 25, f = 14, cf = 12,
2
l = 120 and h = 20

N = 50,

65
60

\ Median = 120 + 25  12 20 = ` 138.57

55
50

fi xi

(40, 52)

14

(40, 48)

45

Mode: Mode = l +

40
35
30

(30, 30)

(50, 28)

20

more than ogive


(20, 14)

5
0

(10, 4)

(60, 10)
(70, 2)

(39, 0)

10 20 30 40 50 60 70 80 90 100

Median: The abscissa of the point of


intersection of both the ogives
determines the median of the given
distribution. To find such abscissa, we
draw a perpendicular from the point of
intersection of both the ogives to the
x-axis, which meet the axis at (39, 0).
Hence the required median is 39 marks.
30. The given distribution can be again represented with the cumulative frequencies as
given below:
Class interval

fi

xi

cf

fi xi

100-120
120-140
140-160
160-180
180-200

12
14
8
6
10

110
130
150
170
190

12
26
34
40
50

1320
1820
1200
1020
1900

50
122

14  12

20 = `125
2 s 14  12  8

31. We prepare cumulative frequency table


from the given data.

15
10

l = 120, f1 = 14, f0 = 12, f2 = 8 and h = 20

\ Mode =120 +

25

f1  f0

2f  f  f

1
0
2

7260

C.I.
( f)

Frequency

Cumulative
Frequency (cf )

0-8
8-16
16-24
24-32
32-40
40-48

8
8
10
18
16
34
24
58
15
73
7
80
N = 80
N
Here, N = 80 \
= 40
2
Cumulative frequency just more than 40 is
58. So 24-32 is the median class.
\ l= 24, cf = 34, f = 24, h = 8

N cf

Median = l + 2
f

48
40 34
= 24 +
= 26.
8 = 24 +
24
24
MATHEMATICS X

Practice Paper-4
SECTIONA
1. (D)

2 3
2 3

2 3
2 3
s
2 3
2 3

2 32 6
= 2 6 5
23
As 2 6  5 is irrational, it has nonterminating, non-repeating decimal form.

2. (C) 2 + k = 1 and p + 3 = 2, i.e., k = 1


and p = 1. So, p + k = 2.
3.

= k[x2 (5 + 2)x + ( 5)(2)]


= k(x2 + 3x 10), k being a real number.
f (x) is not a unique polynomial as k is any
real number.
8. Condition for infinite number of solutions:
a1 b1
c1
2k 3 2k 1 4(k  1)
i.e.,
c2
a2 b2
3
4
3
5
i.e., k = .
2
9. No. In AOB and DOC, AOB = DOC
(Vertically opposite angles)

= =

AD
AE
1
=
=
BD
3
CE
DE BC

||

AO OB
5 3
5
6
as
, i.e., x
x
x
3 5
3 10
DO OC
Therefore, AOB is not similar to
DOC.
10. Yes. Let us take left hand side of the given
equation.

DADE~DABC \
7.5x =

3.5
14

DE
AD

BC
AB

x=

15
cm
8

LHS =

3
3 3
4
= 0.
2
8
SECTION B

\ HCF (x, y) is

PRACTICE PAPER4

cos 2 R

sin 2 R
cos 2 R

= sec2 + tan2 = (1 + tan )2 + tan2


= 1 + 2 tan2 = RHS.
OR
A + B + C = 180

= cot (90

6. The required highest number will be the


HCF of 120, 224 and 256.
120 = 23 3 5; 224 = 25 7; 256 = 28
Therefore, HCF = 23 = 8.
7. Let the required polynomial be f(x).
Then f (x) = k[x2 (sum of zeroes) x
+ product of zeroes]

\ LHS = cot C +2 A = cot 1802 B


Q

5. As p2 and p3 are common factors.

p2n p3l

cos 2 R

4. If x = 30,
3 cos 30 4 cos3 30
=3

1 + sin 2 R

B
B
) = tan = RHS.
2
2

SECTION C
11. Let us assume to the contrary that 3  4 5
is rational. Then we can take integers a and
b 0 such that
a
=3 4 5,
b
a
3b  a
i.e., 4 5 = 3
i.e., 5 =
b
4b

123

RHS of this last equation is rational as a


and b are integers, but LHS of it is
irrational. This is an incorrect statement
due to our wrong assumption that 3  4 5
is rational.
So, we conclude that 3  4 5 is irrational.
12. Any positive integer is either of the form
3q, 3q + 1 or 3q + 2.
There are 3 cases now:
Case I. When n = 3q, n + 1 = 3q + 1
and n + 2 = 3q + 2.
Here, only n is divisible by 3.
Case II. When n = 3q + 1, n + 1 = 3q + 2
and n + 2 = 3q + 3 = 3(q + 1)
Here, only n + 2 is divisible by 3.
Case III. When n = 3q + 2,
n + 1 = 3q + 3 = 3(q + 1)
and n + 2 = 3q + 5 = 3(q + 1) + 2
Hence only n + 1 is divisible by 3.
OR
Let x be any positive integer. Then x can be of
the form: 5m, 5m + 1, 5m + 2, 5m + 3, 5m + 4
If x = 5m
x2 = 25m2 = 5(5m2) = 5q.
x2 = 25m2 + 1 + 10m
If x = 5m + 1
2
= 5(5m + 2m) + 1 = 5q + 1
x2 = 25m2 + 4 + 20m
If x = 5m + 2
2
= 5(5m + 4m) + 4 = 5q + 4
If x = 5m + 3
x2 = 25m2 + 9 + 30m
2
= 5(5m + 6m + 1) + 4 = 5q + 4
If x = 5m + 4
x2 = 25m2 + 16 + 40m
2
= 5(5m + 8m + 3) + 1 = 5q + 1
Square of x can be of the form: 5q, 5q +
1 or 5q + 4.
13. By the division algorithm,
Dividend = Divisor Quotient
+ Remainder
6x3 + 8x2 3x + 8
= g(x) (3x + 4) + 6x + 20

g(x) =
=

124

6 x 3 8 x 2  9x  12
3x 4

(3 x 4)(2 x 2  3)
3x 4
g(x) = 2x2 3.
are zeros of p(x) = x2 px + q.

g(x) =

a, b
a + b = p;
ab = q
2
\ Consider BC2

14. As

C2
B2
2
2 2
2 2
(B C )  2B C
B 2C 2

B 4 C4
B 2C 2
(B 2 C 2 )2  2q 2

=
=

q2

=
=

(B C)2  2BC  2q 2

q2

p 4 4q 2  4 p 2 q  2q 2

q2
p4
q2

4p2
2 = RHS.
q

( p 2  2 q )2  2 q 2

q2

p 4 2q 2  4 p 2 q
q2

Hence proved.

OR
Let the present ages of father and his son
are x years and y years respectively.
According to the given conditions:
x + y = 65
After 5 years, the fathers age
= (x + 5) years
After 5 years, the sons age = (y + 5) years
Therefore, x + 5 = 2 (y + 5)
i.e.,
x 2y = 5
Thus, the required pair of linear equations is
x + y = 65.... (i) and x 2y = 5.... (ii)
Subtracting equation (ii) from equation (i),
we get 3y = 60
y = 20
Substituting y = 20 in equation (i), we get
x + 20 = 65
x = 45
Thus, present age of father = 45 years and
present age of his son = 20 years.

15. Let us draw MN parallel to AB, which


passes through P. So, AM = BN and
DM = CN.

2x 2 (3 x 4)  3(3x 4)
3x 4
MATHEMATICS X

From right-angled triangles APM, BPN,


CPN, DPM; we have respectively
PA2 = PM2 + AM2

.... (i)

PB2

BN2

.... (ii)

CN2

.... (iii)

PD2 = PM2 + DM2

.... (iv)

PC2

PN2

PN2

From equations (i) and (ii),


PA2 PB2 = PM2 PN2
From equations (iii) and (iv),
PC2 PD2 = PN2 PM2

.... (vi)

Add equations (v) and (vi) to get


PA2 + PC2 = PB2 + PD2. Hence proved.
16. Let median AD passes through the point
O on PQ.
To prove: PO = QO

Proof: In APO and ABD,

PAO = BAD

(Common angle)

and

APO
AOP

=
=

ABD
ADB

(Corresponding angles)

\DAPO ~ DABD (By AAA similarity)


PO
AO
BD
=
...(i)
AD
Similarly, in DAQO and DACD,
AO
QO
=
...(ii)
CD
AD

PO = QO (... BD = CD)

Hence, median AD also bisects PQ.


Hence proved.
OR
Let the given parallelogram be ABCD
We need to prove that
AC2 + BD2 = AB2 + BC2 + CD2 + DA2
Let us draw perpendiculars DN on AB and
CM on AB produced as shown in figure.

\D

@D

17.

cos R  sin R 1  3

cos R sin R 1 3
Using componendo and dividendo, we get
cos R  sin R cos R sin R 1  3 1 3
=
cos R  sin R  cos R  sin R 1  3  1  3
2
2 cos R
1
=
cot =
 2 sin R  2 3
3

q
cot q = cot 60 q = 60.

18. LHS =
=2

PRACTICE PAPER4

||

From equations (i) and (ii), we have

PO
QO
=
BD
CD

In BMC and AND,


BC = AD
(Opposite sides of a gm)
BMC = AND
(Each 90)
CM = DN
(Distance between same parallels)
BMC
AND
(RHS criterion)
BM = AN
...(i) (CPCT)
In right triangle ACM,
AC2 = AM2 + CM2
= (AB + BM)2 + BC2 BM2
= AB2 + 2AB . BM + BM2 + BC2 BM2
= AB2 + BC2 + 2AB . BM
...(ii)
In right triangle BDN,
BD2 = BN2 + DN2
= (AB AN)2 + (AD2 AN2)
= AB2 2AB . AN + AN2 + AD2 AN2
2
BD = AB2 + DA2 2AB . AN
BD2 = CD2 + DA2 2AB . BM
...(iii)
[Using (i) and AB = CD]
Adding equations (ii) and (iii), we have
AC2 + BD2 = AB2 + BC2 + CD2 + DA2.
Hence proved.

.... (v)

cos R cos R sin R sin 4 R


sec4 2 cosec2 + cosec4

sec2

125

= 2(1 + tan2 ) (1 + tan2 )2

So,

q) + (1 + q
= (1 + tan2 q) (2 1 tan2 q)
(1 + cot2 q) (2 1 cot2 q)
= 1 tan4 q (1 cot4 q)
= cot4 q tan4 q = RHS. Hence proved.
2(1 +

cot2

19. We know that


cosec (90 ) = sec
cot (90 ) = tan
tan (90 ) = cot

q
q
q

Now,

=
=

cot2

q
q
q

Median = l +

N
 cf
2
f

= 3000 +

250  200
500
115

= 3000 + 217.39 = 3217.39 hours.

21. To draw a line, we need atleast two


solutions of its corresponding equation.

tan 10 .tan 20. tan 60.tan 70 .tan 80


sec . sec tan .tan + sin 2 55
+ sin 2 (90 55)
tan 10 tan 20 . 3 .tan (90 20)
tan (90 10)

sec 2 R  tan 2 R sin 2 55o cos 2 55o


tan 10o .tan 20o . 3 cot 20o s cot 10o
1 1

SECTION D

sec .cosec (90 ) tan .cot (90 )


+ sin 2 55 + sin 2 35

tan 10o s

cf = 200, f = 115, l = 3000.

)2

1
1
.tan 20o .
s 3
tan 10o
tan 20o

Two solutions of
Two solutions of
2x y = 4
xy=1
From the graph, the two lines intersect
each other at the point A(3, 2).
x = 3 and y = 2
Shaded region is ABC.
Height of ABC = 3 units
And its base = BC = 3 units.

20. We prepare the cumulative frequency table


for the given data.
Lifetimes
(in hrs.)

Frequency
(f )

Cumulative
frequency (cf )

1500-2000
2000-2500
2500-3000
3000-3500
3500-4000
4000-4500
4500-5000

24
86
90
115
95
72
18

24
110
200
315
410
482
500

N = 500
Here, h = 500
Q

126

N = 500,

\ N2 = 250.

\ ar(DABC) = 21 3 3 = 29

square units.

MATHEMATICS X

22. Given system of linear equations may be


written as
bx + ay ab (a + b) = 0
b 2x + a2y 2 a 2b 2 = 0

To find g(x), we proceed as following:


x2 2x 1
3x  5 3x3 x2  7 x  5
3x3  5x2

Applying cross-multiplication, we get

6x2  7 x  5
6 x 2  10 x

 2 a3 b 2 a 4 b a 3 b 2
y
1
=
= 2
2 3
 2 a b a 2 b 3 ab 4
a b  ab 2
4 x 3 2 = 4 y2 3 = 2 1 2
a ba b
ab  a b
a b  ab
(i)
(ii)
(iii)
Taking (i) and (iii),
a3 b ( a  b)
a 4 b  a3 b 2

 a2
x= 2
ab ( a  b)
a b  ab 2
Taking (ii) and (iii),

ab3 (b  a) 2
b
ab (a  b)
a2b  ab2
Thus, the required solution is x = a2, y = b2.
23. (i) Let p(x)= Total Relief Fund
g(x) = Number of families who
received Relief Fund
q(x) = Amount each family received
r(x) = Amount left after distribution
When the polynomial p(x) is divided by
a polynomial g(x) such that q(x) and r(x)
are respectively the quotient and the
remainder, the division algorithm is
p(x) = g(x) . q(x) + r(x)
(i)
According to the question,
p(x) = 3x3 + x2 + 2x + 5
q(x) = 3x 5
and r(x) = 9x + 10
Substituting these values of p(x), q(x) and
r(x) in the equation (i), we get
y=

ab4  a2b3

3x3 + x2 + 2x + 5
= g(x) (3x 5) + 9x + 10

(3x 5) g(x) =

3x3

x2

+ 2x + 5 9x 10

= 3x3 + x2 7x 5

g(x) =

PRACTICE PAPER4

3x 3 x 2  7 x  5
3x  5

3x  5
3x  5

0
Thus, g(x) = x2 + 2x + 1.

(ii) Common good, Accountability, social


responsibility.

24. We are given a ABC in which a line


PQ BC intersects the sides AB at P and
AC at Q.

||

AP AQ
.

PB QC
AB and PN AC.

We need to prove
Draw QM

Join PC and BQ.


Proof: Area of a triangle
1
Base Height
2

1
AP QM
2

ar( APQ) =

1
AQ PN
2
AP QM = AQ PN

AQ
QM
=
... (i)
AP
PN
Since BPQ and CQP are on the same
base PQ and between the same parallels
PQ and BC, therefore, their areas should
be equal.
i.e., ar( BPQ) = ar( CQP)

1
1
PB QM = QC PN
2
2

QM
QC
=
PB
PN

... (ii)

127

Adding equations (iii) and (iv), we obtain

Form equations (i) and (ii), it is clear that

QC
AP AQ
AQ
=
i.e.,
=
PB
AP
PB
QC

BAC = QPR
Thus, in DABC and DPQR, we have

Hence proved.
25. Let us produce AD to J and PM to K so
that DJ = AD and MK = PM.
Join CJ and RK.

AC
AB
=
PR
PQ

...(v)

(Given)

QPR
Therefore, DABC ~ DPQR.

and BAC =

[From (v)]

(SAS criterion of similarity)


Hence proved.

q sin q = sin1

m = cosec

26.

sin

cos
1 sin2
=
sin
sin

n = sec

In ADB and JDC,


AD = JD,

ADB = JDC, BD = CD

DADB @ DJDC
AB = JC

Now, LHS = m 2 n

PQ = KR
...(ii)
According to the given conditions, we have

= cos 3

AB
AD
AC
=
=
PQ
PM
PR

= cos2

(SSS criterion of similarity)

JAC = KPR

(Corresponding angles)

i.e.,

DAC = MPR

...(iii)

cot x

27.

1 tan x

LHS =

128

...(iv)

2
3

+ sin 3

2
3

q + sin2 q = 1 = RHS.

tan x
1 cot x

cot x
1 tan x

cos x

= 1 + sec x . cosec x

tan x
1 cot x

sin x

sin x
cos x
=
+
sin x
cos x
1
1
cos x
sin x
cos x
cos x
sin x
sin x
=
+

sin x cos x sin x cos x sin x cos x

cos2 x
sin 2 x
+
sin x (cos x sin x)
cos x (sin x cos x)

cos3 x sin 3 x
sin x . cos x (cos x sin x)

Similarly, we can prove that

DAB = MPQ

2
2 3

) + (mn )

Similarly, we can prove that

2
3

cos4 sin 2 3 cos2 sin 4 3


=
+

sin 2 cos sin cos2

...(i) (CPCT)

AJ
JC
AC
= 2 =
PK
KR
PR
2
[Using (i) and (ii)]
JC
AJ
AC
=
=
KR
PK
PR
AJC ~ PKR

1
cos
cos R

sin 2 R
1 cos2 R
=
cos R
cos R

(SAS criterion of congruence)

q cos q =

MATHEMATICS X

(cos x sin x)(cos2 x cos x . sin x sin 2 x)


sin x . cos x (cos x sin x)
1 cos x . sin x
1
=
=
+1
sin x . cos x
sin x . cos x
= 1 + sec x . cosec x = RHS.
Hence proved.
28. It is given that sin + cos = a
Squaring both sides, we get
sin2 + cos2 + 2 sin cos = a2
But
sin2 + cos2 = 1
.... (i)
2
1 + 2 sin cos = a
2 sin cos = a2 1
a2  1
sin cos =
.... (ii)
2
Cubing both sides of equation (i), we get
sin6 +cos6
+ 3 sin2
cos2
(sin2
2
+ cos ) = 1
=

q
q

q
q
q
q

q
q
q

29. We prepare a table for less than series with


corresponding cumulative frequencies and
points.
Marks

Freq.

Marks
less
than

Cumulative
Freq.

Point

0-10
10-20
20-30
30-40
40-50
50-60

5
8
6
10
6
5

10
20
30
40
50
60

5
13
19
29
35
40

(10, 5)
(20, 13)
(30, 19)
(40, 29)
(50, 35)
(60, 40)

q
q
q
q
q
sin6 q + cos6 q + 3 (sin q cos q)2 = 1
[Using (i)]

sin6 q + cos6 q + 3 a

 1

=1

( a2  1)2
4
2
2
4  3( a  1)
=
.
4
Hence proved.

sin6

q + cos6 q = 1 3

sin6

q + cos6 q
OR

Consider
a sin3 + b cos3 = sin . cos
a sin . sin2 + b cos . cos2
= sin . cos
b cos . sin2 + b cos . cos2
= sin . cos
(... a sin = b cos )
b sin2 + b cos2 = sin
b (sin2 + cos2 ) = sin
b = sin
...(i)
Again,
a sin = b cos
a . b = b cos [From (i)]
a = cos
...(ii)
Now, squaring and adding equations (i)
and (ii), we get
b2 + a2 = sin2 + cos2
a2 + b2 = 1.Hence proved.

a
a
a
a
a
a a
a
a
a
a a
a
a
a
a
a

PRACTICE PAPER4

a
a
a

a
a

a
a

We take upper limits on the x-axis and


cumulative frequencies on the y-axis. Then
we plot the points on the graph paper. By
joining these points by free hand smooth
curve, we obtain less than ogive as shown
in the above figure.
OR
A more than type distribution from the
given distribution is given below:
129

Production yield (in kg/ha)


More
More
More
More
More
More

than
than
than
than
than
than

or
or
or
or
or
or

equal
equal
equal
equal
equal
equal

to
to
to
to
to
to

Cumulative
freq.

50
55
60
65
70
75

100
98
90
78
54
16

To draw more than type ogive, we mark


lower limits from the given table on
x-axis and cumulative frequency from the
above table on y-axis with suitable
scale(s). We plot the points (50, 100), (55,
98), (60, 90), (65, 78), (70, 54) and (75, 16).
Now, we join these points with free hand
smooth curve as shown. in the adjoining
figure.

Fig. : More than type ogive.


30.
Class

Mid-

Freq. ( fi )

interval values

ui =

xi A

fiui

50-70

60 = A

12

0
0
20

70-90

80

20

20
1
20

20

90-110

100

110-130

120

(xi )
10-30

20

30-50

40

130

40
  2
20
20
  1
20

10
8

Sfi = 50

40
2
20
60
3
20

6
6

Sfiui = 14
MATHEMATICS X

Let assumed mean be A = 60


Here,
h = 20

a2 = 4, b2 = 3, c2 = 5

f u
14
Mean = A + h i i = 60 + 20
50
fi
= 60 + 5.6
= 65.6
Hence, the required arithmetic mean is
65.6.
31. We convert the given data of less than
type to the normal distribution.
Marks

fi

xi

di = xi 25

fi di

0-10
10-20
20-30
30-40
40-50

5
11
19
30
15

5
15
25
35
45

20
10
0
10
20

100
110
0
300
300

Now,

Therefore, 3p + q = 12.
3. Consider 3A = 90\ A = 30
So, sin 30 cos 2 30 = sin 30 cos 60
=

1 1

= 0.
2 2

DE + EF + FD
EF
=
BC
AB + BC + CA

4.

Sfidi = 390

Sfi = 80

4f d
390
Now, mean = a + i i = 25 +
80
4fi
= 25 + 4.875 = 29.88 marks.

Practice Paper-5
SECTIONA
1. (B) Let us assume that x + y is rational
number and let x + y = z; when z is
rational.
x + y + 2 xy = z2 2 xy = z2 x y
z2 x y
xy =
2
which given a contradiction as LHS is
irrational but RHS is rational.

x + y is an irrational number.
2. (C) Coincident lines is given by

b1
a1
c
=
= 1
b2
a2
c2
a1 = p, b1 = q, c1 = 4

PRACTICE PAPER5

4
(3 + 2 + 2.5)
2
Perimeter of DDEF = 15 cm.

Perimeter of DDEF =

Let us use the assumed mean method with


assumed mean a = 25.

Here,

p
q 4
p q
= =
= = 0.8
3 5
4
4 3
p = 3.2, q = 2.4

SECTION B
5. True, because product of an even number
and an odd number is an even number.
6. Going in opposite direction to the factor
tree, we obtain
(i) 2 330 = 660 and (ii) 2 165 = 330
7. For infinite number of solutions,
a+b
c
2a
a1
b
18
= 1 = 1 i.e.,
=
=
,
5
2
a2
c2
9
b2
i.e., a = 2 and a + b = 10 i.e., a = 2, b = 8.
8.

BC
AB
CA
1
=
=
=
DABC ~ DDFE
EF
DF
DE
2
B =F. But B = 60, so, F = 60.

9. Let p(x) = ax2 + bx + c


So, y = ax2 + bx + c should be satisfied by
(1, 0), (0, 3) and (4, 0)
Therefore, 0 = a b + c; 3 = c;
0 = 16a + 4b + c

c = 3, a =

9
3
,b= .
4
4
131

3 2 9
x x 3
4
4
3 2

p(x) = (x 3x 4)
4
This is the required expression.
Hence,

p(x) =

10.
xi

15

17

19

20 + p

23

fi

5p

fixi 30

51

76 100p + 5p2 138

Sfi =15 + 5p
Sfixi = 295
+100p + 5p2

Mean =

295 + 100p + 5p2


fi xi
20 =
15 + 5p
fi

300 + 100p = 295 + 100p + 5p2


5p2 = 5 p = 1.
But frequency cannot take negative value.
So, p 1. Hence, p = 1.
OR
Relation among mean, median and mode
is given by the following impirical formula:
Mode = 3 Median 2 Mean
SECTION C
11. Let a be any positive integer. We know
that any positive integer is either of the
form 2q or 2q + 1 for some integers q.
\
a = 2q or 2q + 1
Case I. When a = 2q, a + 1 = 2q + 1.

a (a + 1) = 2q 2q + 1
2q (2q +1) = 2r, where r = q (2q + 1)
So, a (a + 1) is divisible by 2.
Case II. When a = 2q + 1,
a + 1 = 2q + 2 = 2 (q + 1)
\
a (a + 1) = 2 (2q + 1) (q + 1)
where r = (2q + 1) (q + 1)
So, a (a + 1) is divisible by 2.
Hence, multiplication of any two
consecutive positive integers is divisible
by 2.
12. Ram, Ravi and Nitin will meet next after
the time given by the LCM of 5 days, 24
days and 9 days.
132

Now, we find out the LCM of 5, 24 and 9.


5 = 5; 24 = 23 3; 9 = 3 3
\ LCM = 23 3 3 5 = 360
They met last on Sunday. So, it will be
Sunday after 7n days, where n is a natural
number.
So, it will be Sunday after 357 days.
Therefore, it will be Wednesday after 360
days. Hence, they will meet on next
Wednesday.
OR
We represent 6, 72 and 120 in their prime
factors.
6 = 2 3, 72 = 23 32, 120 = 23 3 5
Now,
HCF = 2 3 = 6
And
LCM = 23 32 5 = 360.
13. Let zeroes are a, b, g
Let
ab = 8
... (i)
Also we know a + b + g = 9
... (ii)
ab + ag + bg = 26
abg = 24 \ 8 (g) = 24 g = 3
From (ii), a + b = 6
... (iii)
From (i) , b =

\ Use if in (iii) we get a + B = 6


a2 6a + 8 = 0
(a 4) (a 2) = 0 a = 4 a = 2.
If
a = 4 b = 2 and if a = 2

b = 4 \Zeroes are 2, 4 and 3.


1
1
= u and
= v in the given
xy
x+y
system of equation, we get
10u + 2v = 4
15u 5v = 2
i.e.,
5u + v = 2
... (i)
15u 5v = 2
... (ii)
Multiply equation (i) by 5 and add the
result to (ii)
1
40u = 8 or u =
5
1
Substitute
u =
in equation (i)
5
v =1

14. Put

MATHEMATICS X

1
and v = 1 give x + y = 5
5
and
xy =1
On solving, we get x = 3 and y = 2
Hence, x = 3, y = 2 is the required solution.

u=

15. In DABC, DE || BC
Q
DABC ~ DADE

ar (ABC )
AB
=
...(i) D
ar (ADE )
AD 2
B
Again, DE || BC
Q ar (DADE) = ar ( BCED)
\ ar (DABC) = 2 ar (DADE)

4AN2 + 4 CM2 = (4AB2 + AB2) + (BC2 + 4BC2)


= 5AB2 + 5BC2
4(AN2 + CM2) = 5 (AB2 + BC2) = 5 AC2
[From (i)]
Hence proved.

Adding equations (ii) and (iii), we get

cos 58
cos 18o cosec 52o
17. 2

3
tan 15o tan 60o tan 75o
sin 32

cos (90 32 )
= 2

sin 32

ar (ABC )
=2
ar (ADE )

... (ii)

From equations (i) and (ii), we get

AB
2
=
1
AD
Let

sin 32
= 2
2
sin 32

AB =

2 x and AD = x,
then from the figure,

BD =
Now,

BD
=
AB

2x x =
2 1 x

2 1 x

=21

2x

RHS =

16. In right-angled DABC,


AB2 + BC2 = AC2 ...(i)
(By Pythagoras Theorem)
In DABN,

1
2

1 tan 2
1 tan 2 30
=
1 + tan 2
1 + tan 2 30

Thus, LHS = RHS.


18. LHS =

1
BC2
4

4 AN2= 4AB2 + BC2


Similarly, in DCBM,
4 CM2 = AB2 + 4 BC2
PRACTICE PAPER5

1
= 2 1 = 1.
3
OR

(... N is the mid-point of BC)


= AB2 +

1
1
2
1
1

=
3= 3= 2 = 1
=
2
1
4
4
2
1
1+
1+

3
3
3

AN2 = AB2 + BN2


BC

sin 52o

sin 52o

1
cot 75o 3

cot 75o

LHS = cos 2q= cos (2 30) = cos 60 =

2
2 2
=
.
2
2

2 1

AB2

cos 90o 52o


cosec 52o

tan 90o 75o


s 3 s tan 75o

=
...(ii)
=

Hence proved.

tan + sec 1
tan sec + 1

tan + sec sec 2 tan 2

tan sec + 1

(tan + sec ) (sec + tan )(sec tan )


tan sec + 1

...(iii)
133

tan sec + 1

= tan q + sec q =

21. Given that p(x) = a(x2 + 1) x(a2 + 1)


i.e.,
p(x) = ax2 (a2 + 1)x + a
To find zeroes of p (x), put p(x) = 0.

sin
1
+
cos
cos

1 + sin
=
= RHS.
Hence proved.
cos
19.
p2 q2 = (a cot q + b cosec q)2
(b cot q + a cosec q)2
= a2 cot2 q + 2ab cot q cosec q + b2 cosec2 q
b2 cot2 q 2ab cot q cosec q a2 cosec2 q
= a2 (cosec2 q cot2 q)
+ b2 (cosec2 q cot2 q)
= a2 (1) + b2 (1) = b2 a2.
20. Since, the maximum frequency is 41, so
the modal class is 10000-15000.
\ l = 10000, f1 = 41, f0 = 26, f2 = 16, h = 5000

f1  f0
Now, mode = l +
uh
2 f1  f0  f2
= 10000 +

41 26

s
82 26 16

5000

75000
40
= 10000 + 1875 = 11875.
Thus, the monthly modal income is ` 11875.
OR
Let a = 50; h = 20
= 10000 +

C.I.

fi

xi

0-20
20-40
40-60
60-80
80-100

4
10
28
36
50

10
30
50
70
90

128

xi a
h
2
1
0
1
2

ui =

fi ui
8
10
0
36
100

Sfi ui = 118

fi ui
118
x = a + h f = 50 + 20 128
i

295
= 50 +
= 50 + 18.4 = 68.4.
16
134

SECTION D

(tan + sec )(1 sec + tan )

i.e.,
i.e.,
i.e.,
i.e.,
i.e.,

ax2 + a a2x x
(ax2 a2x) (x a)
ax (x a) 1 (x a)
(x a) (ax 1)
x

=
=
=
=
=

0
0
0
0
1
a,
a

1
are the zeroes of p(x).
a
2
1 a 1
Sum of zeroes = a =
a
a

Thus, a and

a2 1

Coefficient of x
=
a
Coefficient of x 2
1
a
Product of zeroes = a =
a
a
Constant term
=
.
Hence proved.
Coefficient of x 2
OR
Let
p(x) = 3x3 + x2 + 2x + 5
q(x) = 3x 5
r(x) = 9x + 10
g(x) = ?
We know p(x) = q(x) . g(x) + r(x)
3x3 + x2 + 2x + 5 = (3x 5) g(x) + 9x + 10
3x3 + x2 + 2x + 5 9x 10 = g(x) . (3x 5)
=

3x 3 x 2  7 x  5
= g(x)
3x  5
Consider
x2 2x 1
3x  5 3x3 x2  7 x  5

3x3  5x2

6x2  7 x  5
6 x 2  10 x

3x  5
3x  5

g(x) = x2 + 2x + 1.
MATHEMATICS X

22. Let the actual length be x and breadth be y.


Then according to question,
xy 9 = (x 5) (y + 3)
...(i)
and xy + 67 = (x + 3) (y + 2)
...(ii)
Simplifying equations (i) and (ii), we have
3x 5y = 6 ...(iii) and 2x + 3y = 61 ...(iv)
On solving (iii) and (iv), x = 17, y = 9
Hence, length of rectangle is 17 units and
that of breadth is 9 units.
23. Table for values of x and y corresponding
to equation 4x 5y 20 = 0 is
x
y

24. Statement: In a triangle, if square of the


largest side is equal to the sum of the squares
of the other two sides, then the angle
opposite to the largest side is a right angle.
Proof: We are given a triangle ABC with
AC2 = AB2 + BC2
...(i)
We have to prove that B = 90
Let us construct a DPQR with Q = 90
such that, PQ = AB and QR = BC ...(ii)

Similarly for the equation 3x + 5y 15 = 0


x
y

Let us draw the graphs for the two equations.

As the graphs of the two lines intersect


each other at the point A(5, 0), the required
solution is x = 5, y = 0.
The graphs intersect the y-axis at B (0, 3)
and C(0, 4). Therefore, the coordinates of
vertices of the triangle ABC are A(5, 0),
B(0, 3) and C(0, 4).
Hence the answer: x = 5, y = 0 and (5, 0),
(0, 3), (0, 4).

PRACTICE PAPER5

In DPQR, PR2 = PQ2 + QR2


(Pythagoras Theorem)
2
= AB + BC2[From (ii)] ...(iii)
But AC2 = AB2 + BC2 [From (i)] ...(iv)
From equations (iii) and (iv), we have
PR2 = AC2

PR = AC
...(v)
Now, in DABC and DPQR,
AB = PQ
[From (ii)]
BC = QR
[From (ii)]
AC = PR
[From (v)]
Therefore, DABC @ DPQR
(SSS congruence rule)
B = Q
(CPCT)
But Q = 90
...
B = 90.
Hence proved.
OR
We are given two triangles ABC and
PQR such that DABC ~ DPQR.
Draw perpendiculars AD and PM on BC
and QR respectively.

135

AD 2
ar(ABC)
=
ar(PQR)
PM 2
In DABD and DPQM,
ADB = PMQ = 90
ABD = PQM
(...DABC ~ DPQR)
\ DABD ~ DPQM
(AA criterion of similarity)
AB
AD
PQ =
(Corresponding sides) ...(i)
PM
We know that the ratio of areas of two
similar triangles is equal to ratio of squares
of their corresponding sides
We need to prove,

AB 2
ar(ABC)
=
...(ii)
ar(PQR)
PQ 2
From equations (i) and (ii), we have

ar(ABC)
AD 2
=
.
ar(PQR)
PM 2

Hence proved.

25. Given: DABC and DDBC are on the same


base BC and O is the point of intersection
AD and BC

or

ar (%ABC)
AM
=
ar (%DBC)
DN

or

ar (%ABC)
AO
=
Proved.
ar (%DBC)
DO

'

AM
DN

AO
DO

26. We have
q sin q = p and p cos q = q
q
p
sin q =
and cos q =
p
q
6

q = sin6 q + cos6 q
(i) p +
q p
= (sin2 q)3 + (cos2 q)3
= (sin2 q + cos2 q)3 3 sin2 q . cos2 q
(sin2 q + cos2 q)
[... a3 + b3 = (a + b)3 3ab (a + b)]
= (1)3 3 .

2
p2 . q .
1= 1 3 = 2.
2
q2 p

Hence proved.
(ii)

ar (%ABC) AO
=
ar (%DBC) DO
Construction: Draw AM ^ BC and DN ^ BC
Now in DAMO and DDNO,
AMO = DNO
[Each 90]
To prove:

q6

[AA Similarity]

AM
AO
=
DN
DO
[Ratio of corresponding sides of
similar triangles]

1
s BC s AM
ar (%ABC)
2
Now,
=
ar (%DBC)
1
s BC s DN
2

136

q6

= 2

p q12
12

p 6 q6
p6
+ q12 = 2p6 q6

= 2

p12 + q12 + 2p6 q6 = 0 (p6 + q6)2 = 0


p6 + q6 = 0.

Hence proved.

27. We have to prove


sec + 1
sec 1
+
= 2 cosec q
sec 1
sec + 1

[Vertically opposite angles]

DMOA ~ DNOD

p12

AOM = DON
\

LHS
=

sec R  1 sec R  1 sec R 1 sec R 1


s
s
+
sec R 1 sec R  1 sec R  1 sec R 1

(sec 1)2

sec 2 1

(sec 1)2
tan 2

(sec + 1)2

sec 2 1

(sec + 1)2
tan 2
MATHEMATICS X

Marks

sec R 1
sec R 1

tan R
tan R

No. of
students
7
10
23
51
6
3

0-10
10-20
20-30
30-40
40-50
50-60

2
cos
sec 1 + sec + 1 2 sec
=
=
=
sin
tan

tan
cos
2
=
= 2 cosec q = RHS.
sin
Hence proved.
28. Consider an equilateral triangle PQR in
which PS ^ QR. Since PS ^ QR so PS
bisects P as well as base QR.
We observe that DPQS is a right triangle,
right-angled at S with QPS = 30 and
PQS = 60.
For finding the trigonometric ratios, we
need to know the length of the sides of
the triangle. So, let us suppose PQ = x

Marks
less than
10
20
30
40
50
60

cf

point

7
(10, 7)
17 (20, 17)
40 (30, 40)
91 (40, 91)
97 (50, 97)
100 (60, 100)

More than type cumulative frequency table:


Marks
No. of
Marks
cf
point
students more than
or equal to
0-10
7
0
100 (0, 100)
10-20
10
10
97 (10, 97)
20-30
23
20
83 (20, 83)
30-40
51
30
60 (30, 60)
40-50
6
40
9
(40, 9)
50-60
3
50
3
(50, 3)
Y
100

(60, 100)

(0, 100)
(50, 97)

(10, 97)

95

(40, 91)

90
85

Less than
than ogive
ogiv
e
Less

(20, 83)

80

Then , QS =

1
x
QR =
2
2

and (PS)2 =(PQ)2 (QS)2 = x2

75
70
2

x
3x
=
4
4

65
60

3x
2

55

3x
2 = 3
2
x
x
1
QS
(ii) tan 30 =
= 2 =
.
3
PS
3x
2

45

PS =

PS
(i) sin 60 =
=
PQ

29. To draw the less than type and more than


type ogives, we prepare the cumulative
frequency table by less than and more than
methods as given below:
Less than type cumulative frequency table:
PRACTICE PAPER5

(30, 60)

50

(30, 40)

40
35
30
25
20
15

(20, 17)

More than
than ogive
ogiv
e
More
(40, 9)

10
5
0

(10, 7)
(50, 3)

(32, 0)
10

20

30

40

50

60

137

We plot the points as given in both of the


tables on a graph, taking marks on the
x-axis and the cumulative frequencies on the
y-axis. On joining these points by free hand
smooth curve, we obtain the less than and
more than type ogives as shown in the figure.
Median: The abscissa of the point of
intersection of the two ogives determines
the median of the given data. To obtain

the coordinates of this point of intersection,


we draw a perpendicular from this point
on the x-axis. The abscissa of the foot of
this perpendicular is the required median.
Here the coordinates of the foot of the
perpendicular are (32,0), where 32 is the
approximate value.
Hence, the required median is nearly 32
marks.

30. (i) By making the given data continuous, we get: a = 57, h = 3


No. of mangoes

No. of boxes (fi)

Mid-points (xi)

49.5-52.5
52.5-55.5
55.5-58.5
58.5-61.5
61.5-64.5

15
110
135
115
25

51
54
a = 57
60
63

ui =

xi  a
h

fiui

2
1
0
1
2

30
110
0
115
50

S fi = 400
Q

4fi ui

4 fi

\ Mean = a + h

= 57 + 3

25

400

75
= 57 +
400

@ 57.19.
(ii) Step deviation method
(iii) Vikram Singh believes in quality
serving, fruits will remain fresh and free
from germs and flies.
31. First, we prepare the cumulative frequency
table as given below:
Class
interval

Frequency
(f )

Cumulative
frequency (cf )

85-100
100-115
115-130
130-145

11
9
8
5

11
20
28
33

N = 33

138

S fiui = 25
N = 33

N
= 16.5
2
Cumulative frequency just greater than
16.5 is 20. So, median class is 100-115.

\ cf = 11, f = 9, l = 100, h = 15
N

cf
2

Now, median = l +
u h
f

16.5

= 100 +

11

s 15

= 100 + 9.17
= 109.17
Hence, the median speed is 109.17 km/hr.

oo

MATHEMATICS X

NOTE
_____________________________________________________________________________________
_____________________________________________________________________________________
_____________________________________________________________________________________
_____________________________________________________________________________________
_____________________________________________________________________________________
_____________________________________________________________________________________
_____________________________________________________________________________________
_____________________________________________________________________________________
_____________________________________________________________________________________
_____________________________________________________________________________________
_____________________________________________________________________________________
_____________________________________________________________________________________
_____________________________________________________________________________________
_____________________________________________________________________________________
_____________________________________________________________________________________
_____________________________________________________________________________________
_____________________________________________________________________________________
_____________________________________________________________________________________
_____________________________________________________________________________________
_____________________________________________________________________________________
_____________________________________________________________________________________
_____________________________________________________________________________________
PRACTICE PAPER5

139

NOTE
_____________________________________________________________________________________
_____________________________________________________________________________________
_____________________________________________________________________________________
_____________________________________________________________________________________
_____________________________________________________________________________________
_____________________________________________________________________________________
_____________________________________________________________________________________
_____________________________________________________________________________________
_____________________________________________________________________________________
_____________________________________________________________________________________
_____________________________________________________________________________________
_____________________________________________________________________________________
_____________________________________________________________________________________
_____________________________________________________________________________________
_____________________________________________________________________________________
_____________________________________________________________________________________
_____________________________________________________________________________________
_____________________________________________________________________________________
_____________________________________________________________________________________
_____________________________________________________________________________________
_____________________________________________________________________________________
_____________________________________________________________________________________
140

MATHEMATICS X

You might also like